★東大京大入試作問者になったつもりのスレ★

このエントリーをはてなブックマークに追加
1
理系で数学が得意な高校生が25〜50分で解ける問題を
考えてうぷするスレ。前スレが落ちてたので新しく立てますた。

★東大入試作問者になったつもりのスレ★
http://science.2ch.net/test/read.cgi/math/1000592003/
2:03/02/25 18:26
ちなみに今日が前期入試一日目だというのは全くの偶然でございまつ。
3132人目の素数さん:03/02/25 18:26
ぷっ
4132人目の素数さん:03/02/25 18:26
ヴォケが。
5132人目の素数さん:03/02/25 18:28
>860 名前:大学への名無しさん[] 投稿日:03/02/25 07:58 ID:Jk+eqVdA
>彼女の家に泊まってしまい三時間くらいしか寝れませんですた(((( ;゚Д゚)))ガクガクブルブル

逝ってよし。
6132人目の素数さん:03/02/25 18:29
>860 名前:大学への名無しさん[] 投稿日:03/02/25 07:58 ID:Jk+eqVdA
>彼女の家に泊まってしまい三時間くらいしか寝れませんですた(((( ;゚Д゚)))ガクガクブルブル

どうせ気を紛らわす為のセクースとかしたんだろヴォケが。落ちちまえ(ゲラゲラ
75-6:03/02/25 18:31
………………………………ハァ(;´Д`)=3 漏れ、なにやってんだろ…
8132人目の素数さん:03/02/25 18:38
(゚∀゚)アヒャヒャヒャヒャヒャヒャヒャヒャヒャヒャヒャ
9132人目の素数さん:03/02/25 18:44
えーまた入試考えなきゃなんねえのかよ、面倒だな、逃げてえ
10132人目の素数さん:03/02/25 20:07
>>1
もう1日目終わったっちゅうに。
・・・今更どうしろと。俺はやだ。考えないぞ!
11132人目の素数さん:03/02/26 03:10
π>3.05を証明しなさい
>>11
どこまでπに対する知識を使用したら良いのかわからない。
有名な級数展開式などを使用して良いのか?とか。
極端な話、「π=3.14159…故にπ>3.05」が駄目かどうかも不明。
13132人目の素数さん:03/02/26 03:52
>>12
君採用!
14132人目の素数さん:03/02/26 09:54
そもそもπの定義にも依る。
>>14
言えるね。π=3という定義をしているところだってあるかもしれない。
聖書にだって載ってるし。
e^π>21

π<3.15789

π>3.05

近似値問題は少しずつ簡単になってゆく…

後期は「0.6 =< Σ[k=2〜∞](1-k/√(k^2-1)) < 0.7」みたいな問題が出てくるといいなぁ…
17世直し一揆:03/03/09 14:42
<血液型A型の一般的な特徴>(見せかけのもっともらしさ(偽善)に騙されるな!!)
●とにかく気が小さい(神経質、臆病、二言目には「世間」(「世間」と言っても、同じA型を中心とした一部の人間の動向に過ぎないのだが・・・)、了見が狭い)
●他人に異常に干渉し、しかも好戦的でファイト満々(キモイ、自己中心、硬直的でデリカシーがない)
●妙に気位が高く、自分が馬鹿にされると怒るくせに平気で他人を馬鹿にしようとする
(ただし、相手を表面的・形式的にしか判断できず(早合点・誤解の名人)、実際にはた
いてい、内面的・実質的に負けている)
●本音は、ものすごく幼稚で倫理意識が異常に低い(人にばれさえしなければOK!)
●権力、強者(警察、暴走族…etc)に弱く、弱者には威張り散らす(強い者にはへつらい、弱い者に対してはいじめる)
●あら探しだけは名人級でウザイ(例え10の長所があってもほめることをせず、たった1つの短所を見つけてはけなす)
●基本的に悲観主義でマイナス思考に支配されているため性格がうっとうしい(根暗)
●単独では何もできない(群れでしか行動できないヘタレ)
●少数派の異質、異文化を排斥する(差別主義者、狭量)
●集団によるいじめのパイオニア&天才(陰湿&陰険)
●悪口、陰口が大好き(A型が3人寄れば他人の悪口、裏表が激しい)
●他人からどう見られているか、人の目を異常に気にする(「〜みたい」とよく言う、
世間体命)
●自分の感情をうまく表現できず、コミュニケーション能力に乏しい(同じことを何度
も言ってキモイ)
●表面上協調・意気投合しているようでも、腹は各自バラバラで融通が利かず、頑固(本当は個性・アク強い)
●人を信じられず、疑い深い(自分自身裏表が激しいため、他人に対してもそう思う)
●自ら好んでストイックな生活をしストレスを溜めておきながら、他人に猛烈に嫉妬
する(不合理な馬鹿)  
●後で自分の誤りに気づいても、強引に筋を通し素直に謝れない(切腹するしかない!)●自分に甘く他人に厳しい(自分のことは棚に上げてまず他人を責める。包容力がなく冷酷)
●男は、女々しいあるいは女の腐ったみたいな考えのやつが多い(例:「俺のほうが男
前やのに、なんでや!(あの野郎の足を引っ張ってやる!!)」)
>>16はティムカス以下
1916:03/03/12 21:13
>>18
ふ、ふざけんな!評価低すぎんぞヴォケ!!
20132人目の素数さん:03/03/13 20:14
21132人目の素数さん:03/03/13 20:29
1(1)はクソ問だが(2)は激ムズの予感・・・
これからやってみるか。
問題3における(2)の存在意義は時間・問題数調整以外にあるのだろうか…?
2番連分数知ってるとかなり有利な気が。
>>21に聞きたいんだけど、1の(1)はどうやるの?
まさかテイラー級数で剰余項を評価して・・・はなしだよね?
むー。今年は京大後期の4番が一番好きだな。
問題
高校レベルの問題…
必要条件に含まれる集合の要素の数は
十分条件に含まれる集合の要素の数と等しいか又は
それより大きい事を証明せよ。
28132人目の素数さん:03/03/14 03:28
>>27
これって、定義だと思ってた。
証明できるんや。
小さいと仮定すると
十分条件の定義より
成り立たないものが出てくるからじゃだめですか?
30 ◆njG9nIUngc :03/03/16 15:20
今年は円周率の評価だから来年は三角関数かなとか思ったので次の問題どうぞ。

[問い]
(1) cos4θを cosθ の整式で表わせ。
(2) cos11°>0.98 を証明せよ。
(1)があると簡単すぎでは?
32132人目の素数さん:03/03/16 17:02
cos4θ=cos2θ*cos2θ−sin2θ*sin2θ
   =(2cos^2θ-1)^2−4(1-cos^2θ)*cos^2θ
   =8cos^4θ-8cos^2θ+1
(注意)cos^4θ=(cosθ)^4

(2)
θ=11°とおくと
cos44°=8(cos11°)^4-8(cos11°)^2+1>1/√2
整理して
8√2(cos11°)^4-8√2(cos11°)^2+√2-1>0
解の公式を用いてとくと
(cos11°)^2=(2±√(2+√2))/4≒0.961(プラスのほう)
よって
cos11°≒0.9803>0.98    
33132人目の素数さん:03/03/16 17:16
問い
関数f(x)=x^xには極小値が存在することを示し、またそのときのxの値を求めよ。

…簡単すぎるか。
34 ◆njG9nIUngc :03/03/16 18:21
>>31
自分でも思った。やっぱ(2)だけにしとくべきだったか・・・。

こちらはヒントなしで。これ自信作です。

[問い]
行列Aを
A=[[0, -1], [1, 2cosαπ]]
で定める。A^n=A となるような2以上の自然数nが存在するための
αのみたすべき必要十分条件を求めよ。
>>34
Jordan標準形を考えておしまい…おもしろくない
36 ◆njG9nIUngc :03/03/16 18:38
>>35
いや・・・ここ大学入試問題を考えるスレなんですけど・・・。
高校範囲で考えなきゃ。
行列ツマンネ
38132人目の素数さん:03/03/16 18:40
33が解けないんですが
>>38
ぶんびー
40132人目の素数さん:03/03/16 18:42
>>34 固有方程式は使用不可能ですか?
41 ◆njG9nIUngc :03/03/16 19:24
>>40
昔の指導要領にはあったような気がするが、確か今はないから微妙。
使わなくても出来るとだけいっておこう。
ねねね、ねえ、みんな、
こんなことして悲しくならない?
悲しいと思っている人は正常かも・・・。
43132人目の素数さん:03/03/16 20:06
>>33
f(x)の対数を取ると、
log||f(x)|| = log(x^x) = xlogx
両辺を微分すると、
||f(x)'||/f(x) = (x)'logx+x(logx)' = logx+1
よってf(x)' = f(x)(logx+1) = (x^x)(logx+1)
log(1/e) = -1、つまりlog(1/e)+1 = 0
x = 1/eのとき、f(x)' = 0、よって極値。

>>34
計算がめんどくさいので2cosαπ = tとでも置く。||t||≦2
A = | 0 -1|
.   | 1 t |
…バトンタチキボン…ゴメソ
||t||≦2 を何に使うのか興味津々(w
4543:03/03/16 21:52
>>44
特に意味はないよ。…消せばよかったなぁ。

>>42
貴方のためにも私たちのためにも帰ってください
46132人目の素数さん:03/03/17 20:29
http://school2.2ch.net/test/read.cgi/kouri/1047893003/l50
★数学を『考えて解ける』様になるスレ★


アドバイス降臨きぼんぬ
47 ◆CGMTP7bY0Q :03/03/18 14:07
>>34のヒントを[メール欄]に書きました。
4834=47 ◆CGMTP7bY0Q :03/03/18 14:08
あれ? トリップ変わっちゃった。まあ、いいや。
>>47-48
ハミルトン・ケーリーの定理により

 A^2 - 2cosαπ・A - E = 0

また、x^n を x^2 - 2cosαπ・x - 1 で割った時の商を Q(x)、余りを px + q とすると、

 x^n = (x^2 - 2cosαπ・x - 1)Q(x) + px + q

が成立。x^2 - 2cosαπ・x - 1の解を g 、 h とすると、

 g^n = pg + q 、 h^n = ph + q

である。よって、

 p = (g^n - h^n)/(g + h)
 q = gh(g^n + h^n)/(g + h)

である。AとEは可換なので、

 A^n = (A^2 - 2cosαπ・A - 1)Q(A) + ((g^n - h^n)/(g + h))A + gh(g^n + h^n)/(g + h)E

である。つまり、

 A^n = |        0               (g^n - h^n)/(g + h)       | + | gh(g^n + h^n)/(g + h)          0        |
      | -(g^n - h^n)/(g + h)   2cosαπ・(g^n - h^n)/(g + h).  | . |          0         gh(g^n + h^n)/(g + h) |

    = (g^n + h^n)/(g + h).| gh       1      | = | 0   -1   . |
                     | -1  2cosαπ + gh .|  .| 1 2cosαπ |

A^nを求めてみますた。間違いがあったら教えてクリ
50 ◆CGMTP7bY0Q :03/03/18 19:51
最初が違う。

A^2-2(cosαπ)A+E=O

だよ。
51132人目の素数さん:03/03/19 00:33
(-)÷(-)=(+)になることを証明せよ
(・)(・)=オパーイになることを証明せよ
>>49
参加賞問題は別として、東大や京大がそんな答え丸見えの問題を出すとは思えない

いや、もしかしたらそんな日が来るのかもな(w
1985年生まれが多いと思われる2004年度入試・・・を想定。
でも適当に作ったのでちょっと簡単すぎ。(3)はこじつけすぎ。

2004のn乗を1985で割った余りをP(n)とおく。
(1) P(1)を求めよ
(2) P(2)を求めよ
(3) {√P(4)}*(1/2)を求めよ
55132人目の素数さん:03/03/19 12:45
e^(1/e) > √2 を証明せよ。
5655:03/03/19 12:47
ごめん、恐ろしく簡単だったね。
>>52
それはケムンパスでやんす. よってその命題は偽.
 
59132人目の素数さん:03/03/21 01:12
半径rの球の体積が4πr^3/3になることを証明せよ
>>59
一見よさげだが、ちょっと簡単すぎ。積分するだけだから
>>55
確かに、道を間違えなければ恐ろしく簡単
運がよければ3分でおしまい、運が悪ければ以下(ry
試験をやると点差は十分につきそう?
ただ、eと√2 を適当に近似して不等式をごにょごにょすれば、満点は力技で十分ゲットできるな…
計算嫌いの漏れは電卓なしでは力技はパス(w

出題者の予想した(であろう…あれでしょ?)答えを見るとバカでもわかるだろうけど、意外に点数の
取れない奴の多い(≒白紙)問題の典型かも
だけど、>>30のようなとってつけたような、対策が余裕で可能で一本道な(例え誘導尋問がなくても)
不等式よりはよほどか面白いと思う

出題者のレベルの違いを露骨に感じるよ(w
62132人目の素数さん:03/03/21 02:55
どうして円周率が「およそ3」になったのか、GHQの上陸から説明せよ
63132人目の素数さん:03/03/21 03:36
nを2以上の自然数とする。a_i (i=1,..,n) を自然数とし、a_1 != a_2 とする。
以下の方程式の解の個数を求めよ。

a_1^a_2 = a_2^a_3 = .... = a_n^a_1
64132人目の素数さん:03/03/21 19:47
>>55
e^(1/e) だろ? 2.7182^(1/2.7182) = 0.36789
これは √2 = 1.41421よりも明らかに小さくないか?

なんか大きな間違いをしてる気がするので教えてキボンヌ
>>64
ゴメン嘘電卓の使い方間違えてた
>55
「2<e」は使っていいの?
>>66
それは簡単に示せるし、端折らない方がいいと思う。
>>55
nを自然数、x>0として、f_n(x)=e^x-(1+Σ[m=1,n]x^m/m!)とする。
まずf_n(x)>0をnについての数学的帰納法で示す。
(@)n=1のときe^xは単調増加だから
   f_1(x)=e^x-1>e^0-1=0より成り立つ。
(A)n=k(≧1)のときf_k(x)>0を仮定すると
   f_{k+1}'(x)=f_k(x)>0よりf_{k+1}(x)は単調増加。
   ∴ f_{k+1}(x)>f_{k+1}(0)=0
ゆえに数学的帰納法によりf_n(x)>0が示された。
n≧2として
e^x>1+1/e+Σ[m=x,n]1/(m!e)^m
  >1+1/e+Σ[m=x,n]1/[{(1+2+…+m)/m}e^m]
  =1+1/e+Σ[m=x,n]1/{(m+1)e/2}^m  (∵相加平均>相乗平均)
  >1+1/e+{2/(3e)}^2
  >1+0.36+0.06
  =1.42
  >√2

解答用紙に書くと減点されるが、証明は出来た。
今気付いたが、途中からのΣの中身がおかしい。正しくは
Σ[m=2,m]
なのでよろ。
正しくなってなかった罠。Σ[m=2,n]でよろ。つってくる…∧||∧
7155:03/03/21 22:32
おつかれー。

f(x) = x^(1/x), (x > 0) を考える。
log f(x) = log x / x
f'(x) = x^(1/x) * (1 - log x)/x^2
x^(1/x) は常に正だから、f'(x)=0 は、x=e のときのみ。
x=e で最大値を取ることもf'(x)の挙動から分かる。

即ち、任意の正数 a に対して、e^(1/e) > a^(1/a) 。
a = 2 を入れれば、題意が示される。

ついでに、類題〜。 pi^(1/pi) > √2 を証明せよ。

# pi^(1/pi) > 3^(7/22) = 2187^(1/22) > 2048^(1/22) = √2
# で証明できちゃうんでこれでもあまりいい問題でないけど。
>>71
なるほど! 感動した。
確率は苦手なのでちゃんと解ける問題になってるかどうか分からんけど。

次のような2種類のサイコロがある。
(A)・1から3の目が出る確率はいずれも等しい
   ・4から6の目が出る確率はいずれも等しく、それぞれ1の目が出る確率の2倍である
(B)・1から5の目が出る確率はいずれも等しい
   ・6の目が出る確率は、1の目が出る確率のk倍である(ただし、kは自然数)

このようなサイコロが1個ずつあるとして、次のような手順でゲームを行う。
1・先攻は2個のサイコロからどちらか好きなほうを選んで振る
2・後攻はその結果を見て、相手と同じサイコロか残っているサイコロかのどちらかを選んで振る
3・出た目の大きい方を勝ちとする

今、先攻が4を出したとする。後攻はどちらのサイコロを選ぶほうが有利か。
ただし、サイコロは見た目などでどちらが(A)でどちらが(B)かを区別することはできない
(先攻も自分がどっちを振っているのか分からない)。また、kの値は既知であるとする。
>>73
kが2以下で変えないほうがよく、3以上で変えたほうがいい
7573:03/03/25 01:11
>74
正解。
6だけじゃなく「1と6が出やすい」とかしたほうが
面白かったかも、と今更考えてたり。
サイコロを2個振って出た目の積をpとする。ある実数の定数kがあって、
||p-k||を得点とする。得点の期待値が最小となるようにkを定めよ。
77132人目の素数さん:03/03/25 02:46
63の問題書き直すね。反応ないから。

nを2以上の自然数とするとき、
a_1^a_2 = a_2^a_3 = ... = a_n^a_1
を満たす自然数の組 (a_1, a_2, ..., a_n) の中で a_1 ≠ a_2 なるものの個数を
f(n) とするとき、f(n) を求めよ。
>>73
強引に解いたんだが、それしかないのかな。
7973:03/03/26 12:55
>78
たぶんね。
もし実際に入試で出すとしたら、50^2から100^2くらいまでの範囲の
テーブルを用意したほうがいいのかも。
手で開平計算できる受験生は少ないみたいだし、できるできないで
解ける解けないが分かれてしまうのは望ましくないし。
80132人目の素数さん:03/03/28 11:38
e>2.71を証明せよ。
81132人目の素数さん:03/03/28 15:15
e^x > 1 + x + x^2/2 + x^3/6 + x^4/24 + x^5/120 を証明してから
x = 1 を代入する以外に解く方法はなさそうな感じ。
82132人目の素数さん:03/03/28 15:55
ピカソは親友カサヘマスの恋人を寝取り、それを知ったカサヘマスは自殺した。
これを三角関係という。つまり三角関係はなりたたない。
三次元上に1991の点をとりそれぞれの点は最短距離の点にむけ
矢印型のラブビームをおくっている。ラブビームを受けない星がゼロである確率を求めよ。
83132人目の素数さん:03/03/28 16:04
しかも二点間の距離はすべてちがっているものとする
84132人目の素数さん:03/03/28 16:11
>82
ゼロ

世の中には必ずもてない奴がいる
これ自明
85132人目の素数さん:03/03/28 16:14
http://www.pink-angel.jp/betu/linkvp2/linkvp.html
★☆★☆★☆★☆★☆★☆★☆★☆★☆★☆★
86132人目の素数さん:03/03/28 16:59
e>(1+0.001)^1000>1+1+0.999/2!+0.999*0.998/3!+0.999*0.998*0.997/4!+0.999*0.998*0.997*0.996/5!
各項は 前の項*(1-0.001*n)/(n+1) > 前の項/(n+1)-0.001
1/2-0.001=0.499, 0.499/3-0.001>0.165, 0.165/4-0.001>0.040, 0.040/5-0.001=0.007
e>合計2.711
87132人目の素数さん:03/03/28 17:15
東工大85年
1)
a^2-2b^2=+1 または a^2-2b^2=-1
2)
a+√2b>0
をみたす任意の整数 から得られる実数 全体の集合を G とする. 1より大きい G の元のうち最小のもの
を u とする.
(問い1)
u を求めよ.
(問い2)
整数 n と G の元 g に対し, gun は G の元であることを示せ.
(問い3)
G の任意の元 g は適当な整数 m によって, g=um と書かれることを示せ.

ペル方程式を題材に出題するのも面白い
88132人目の素数さん:03/03/28 17:50
★☆★☆★☆★☆★☆★☆★☆★☆★☆★☆★

http://www5b.biglobe.ne.jp/~ryo-kyo/osu.html

↓ おすすめゲーム ( フリーソフト )
http://my.vector.co.jp/servlet/System.FileDownload/download/ftp/0/278022/pack/win95/game/table/pachinko/majo_.lzh

★☆★☆★☆★☆★☆★☆★☆★☆★☆★☆★
89132人目の素数さん:03/04/06 12:27
複素数平面上に、中心が原点と重なるようにアナログ時計が置かれている
(3時の位置が実軸の正の部分に重なるように置く)。
また、aは24未満、bは60未満の非負整数とする。
(1)時計がa時b分を指しているとき、長針の指す複素数と短針の指す複素数の
偏角をそれぞれ求めよ。
(2)長針の指す複素数と短針の指す複素数の積が実数になる時刻を求めよ。
90132人目の素数さん:03/04/07 23:56
このスレで発表されてるような問題って、
みなさんどうやって考えるんですか?

>>90
そこらの問題集から良問をパクる
92132人目の素数さん:03/04/08 00:04
パクってるだけなんすか?
大学入試を作っている先生たちはどうやって考えるんですかね。
結構簡単に思いつくもんなんでしょうか。
特に作為的なパズルみたいな問題は
作ろうとしないと作れないと思えるのですが。
93132人目の素数さん:03/04/08 00:53
他の問題を解いているときに思いつくなぁ。
俺も>>93に同じ。
問題を作ろうとして作れるんじゃなく、問題に取り組んでるときに思いつく。
あと、風呂入ってるときとか、夢ン中で思いついたこともあった。
さすがに夢からさめたとき、自分の精神状態を危ないと感じたが…。
95よく言わないって?:03/04/08 02:16
よく言うじゃないですか。

一般に思いつきというのは
人が精出して仕事をしているときに限って現れるものだって。
そんなもんなのかもな〜
もちろん、普段の勉強が大切だろうけど…
97132人目の素数さん:03/04/10 21:49
>>73は、先攻が4の目を出したとき、それがサイコロAだった確率の方が高いから
先攻がサイコロAを振ったと仮定して、Bの戦略は
 「kが2以下で変えないほうがよく、3以上で変えたほうがいい」
という答えを導いたということですか?
98132人目の素数さん:03/04/16 03:06
おもろい問題作ってくりくり。
99132人目の素数さん:03/04/16 23:07
>>30の問題ですが、>>32の回答を見て問1の1行目までは分かったのですが、
それ以降が分かりません。
教えてチャソで申し訳ありませんが、どなたか詳しく説明をお願いしまつ。。。
10099:03/04/16 23:09
99ですが追記です。
数学板に来たのが初めてでよく分からないのですが、
「^」←これは何ですか?
101132人目の素数さん:03/04/16 23:49
>>100
眉毛です。

2^2は2まゆげ2と読みます。
102100:03/04/16 23:51
回答ありがとうございます。
ところでもう一つ質問があるのですが、教えてもらうばかりですいません
「∫」←これって耳ですよね?
103132人目の素数さん:03/04/16 23:52
>>102
違います。Sです。

やばいことを書く時にこれをいれて、検索でヒットしないようにしてるんです。
10499 100:03/04/17 00:19
ドキュで申し訳ない。。。
あおられるだろうとは思ってましたが、教えて下さい(ノД`)゚・。

1行目が加法定理なのは分かったのですが、それがどう2行目につながるのか分かりません…
>>101
眉毛じゃなくて目だと思ってたんだが… → (^_^)

じゃ、目を入れるとこうか。 → (^・_・^)

いかん ^ が目で ・ が鼻の穴に見える。
106山崎渉:03/04/17 08:50
(^^)
107132人目の素数さん:03/04/19 15:33
>>104
べき乗
cos(e^iπ)を求めよ 但しi^2=-1である
(只今酒乱))
>>108 悪問の典型だぞ。それは。
110山崎渉:03/04/20 04:01
   ∧_∧
  (  ^^ )< ぬるぽ(^^)
111132人目の素数さん:03/04/21 01:17
>>105
(^・_・^)
だと^が耳に見えるぞ
>>111
見えませんが?
113132人目の素数さん:03/04/21 01:32
>>112
盲目なのに2ちゃんねらとはすごい!!ばっは!!
逆に見える香具師もすごいと思うぞw
115 ◆EvBfxcIQ32 :03/04/21 15:34
>>104
cos2θ=2cos^2θ-1, sin2θ=2sinθcosθ, sin^2θ=1-cos^2θ を使う
116132人目の素数さん:03/05/01 00:50
  _, ._
( ゚ Д゚)
( ゚д゚)
118132人目の素数さん:03/05/04 00:02
すたれているな・・・。ではありがちな問題だが問題をだそうか。

三角形の内接円の半径をr、外接円の半径をRとする。
このときr/Rの最大値を求めよ。
119132人目の素数さん:03/05/04 00:02
( ゚д゚)
120132人目の素数さん:03/05/06 23:09
2002年度高知大理学部

n,kは1≦k≦nをみたす整数で、
 a(n)=(1+1/n)^n (n=1,2,…)
とおく。
(1)n≧2のとき、2<a(n)<2+納k=2,n](1/k!) が成り立つことを示せ。
(2)n≧2のとき、納k=2,n](1/k!)<1 が成り立つことを示せ。
(3)n≧1のとき、(n+1)^n/n!=a(n)a(n-1)…a(1) が成り立つことを示せ。
(4)n≧3のとき、3(n/3)^n<n!<2(n/2)^n が成り立つことを示せ。
121132人目の素数さん:03/05/07 00:08
(証終)
122132人目の素数さん:03/05/08 10:14
問い
tとsが任意の実数値をとるとき
(t+s)/((t^2+1)(s^2+1))
の最大値を求めよ。
(ヒント:図形の問題に帰着させる)
123動画直リン:03/05/08 10:23
124132人目の素数さん:03/05/08 14:51
>>122
さっぱり思いつかない…
125122 ◆R5P/AyayaA :03/05/08 15:04
>>124
漏れなんかの問題を考えてくれたんだ.ありがとう.
ヒント2:基本対称式をなんとかしてみる
126132人目の素数さん:03/05/10 21:07
>>118
答え:正三角形のとき;最大値r/R=1/2。
>>122
答え:t=s=1/√3のとき;最大値=9/8√3。

どちらも瞬殺だったがもちろんたいした自慢にはならないね。
>>126
自慢になると思いますが
とにかくどうやって考えたか
あと瞬札といってもどのくらい考えたか教えてください
128132人目の素数さん:03/05/11 19:58
問い
3角形の内接円の中心から各頂点までの距離が1,1/2,1/3
で与えられている時、内接円の半径を求めよ。
129132人目の素数さん:03/05/11 20:12
問い
三角形の二つの底角のそれぞれに二等分線を引き、それぞれの線
が対辺と交わる点までの長さが等しいとき、この三角形は
二等辺三角形であることを証明しなさい。ただし三角関数を使っては
ならない。
>>129
ホダイが必要
だがメンドイ
>>129
大学入試で三角関数を使うななんて条件がつくだろうか?
132 ◆R5P/AyayaA :03/05/12 09:23
>>127
恐らく、s=tan(A/2),t=tan(B/2)の置き換えを使ったものと思われ。
これだと瞬殺。

この解法を殺す問題にしてみた。

問い
tとsが任意の実数値をとるとき、Aを以下のように定める。
A=(t+s)/((t^2+1)(s^2+1))

(1)Aの式についてtとsを実数x,yでy=ts,x=t+sとして置き換える。このとき
xとyの存在条件をxとyの式を使って答えよ。またAをxとyで表せ。
(2)xとyの式であるAが(1)の条件を満たして変化するとき、Aの最大値を求めよ。
133たま@都立高3:03/05/12 09:38
>132
こけタソの回答にあった。 まったくわけわからんけど。東大逝きたい!

s+t=p,st=q とおくと,s,tは,uに関する2次方程式:u^2-pu+q=0・・・ア の2解.
よって,アの判別式≧0 より,p^2-4q≧0・・・イ
また,与式=k とおくと,
k=p/{p^2+(q-1)^2} ⇔ k{p^2+(q-1)^2}=p・・・ウ (∵イより,(p,q)≠(0,1))
いま,kの最大値を求めるので,k>0 として考える.(∵s+t=0 のとき,k=0.)
したがって,
ウ ⇔ p^2+(q-1)^2-(1/k)p=0
⇔〔p-{1/(2k)}〕^2+(q-1)^2={1/(2k)}^2・・・エ となる.
ここで,
p-q平面において,領域:イと,円:エが共有点を持つようなkの範囲を考える.
円:エは中心が第一象限にあり,かつ,(0,1)においてy軸と接する円である.
したがって,放物線:q=(1/4)p^2 と円:エが接するときの円:エの半径の値をrとすると,
[円:エの半径]≧r,すなわち,1/(2k)≧r・・・★ が成り立つ.よって,rの値を求めればよい.
134たま@都立高3:03/05/12 09:39
円C:(p-r)^2+(q-1)^2=r^2 (r>0)
放物線D:q=(1/4)p^2
が接するとして,rの値を求める.
CとDの接点をP(r+rcosθ,1+rsinθ) (0≦θ<2π) とおく.
Pにおける円Cの接線は,(rcosθ)(p-r)+(rsinθ)(q-1)=r^2・・・オ
また,Pにおける放物線Dの接線の傾きは,(r+rcosθ)/2・・・カ
直線:オの傾きがカに等しくなるので,r*{(sinθ)(1+cosθ)}=-2cosθ・・・キ
ところで,PはD上の点でもあるから,1+rsinθ={(r+rcosθ)^2}/4・・・ク
キ,クより,cosθ=1/2 となるので,θ=π/3,5π/3 を得る.

θ=π/3 のとき,キより,r<0 となるので不適.
θ=5π/3 のとき,キより,r=4/(3√3) となり,これはクも満たす.
よって,(θ,r)=(5π/3,4/(3√3)) となり,★ ⇔ 1/(2k)≧4/(3√3) であるから,
0<k≦(3√3)/8 となる.

∴ 最大値=(3√3)/8・・・答

(注) 最大値:(3√3)/8 を与える(s,t)は,(p,q)=(2/√3,1/3) であるから,
   u^2-(2/√3)u+(1/3)=0 の2解です.すなわち,s=t=1/√3 です.
135たま@都立高3:03/05/12 09:42
この回答でわけわからんのは
>p-q平面において,領域:イと,円:エが共有点を持つようなkの範囲を考える.
というところなんですが。べつに共有点がなくてもいいんじゃないかと思うんだけど
どうしてなんですか。
それに接点の求め方がぜんぜんわからないですが。はんべつしき=0じゃ解けないですか。
136 ◆R5P/AyayaA :03/05/12 11:24
>>135
こけタソってなんですか?よかったら教えて下さい。

あと、p^2-4q≧0の条件下でk=p/{p^2+(q-1)^2}の最大値を求める問題は
p^2-4q≧0の条件下で〔p-{1/(2k)}〕^2+(q-1)^2={1/(2k)}^2のkが最大、
つまり半径が最小になる、という問題に置き換えられます。
放物線のと円を図で書いてみて下さい。
すると、問題は放物線の下の領域に含まれる条件下で半径が最小になる場合で、
円と放物線が接する場合最小になることがわかります。

で、接する場合は
円C:(p-r)^2+(q-1)^2=r^2 (r>0)
放物線D:q=(1/4)p^2
の2式からqを消去した式
(p-r)^2+((1/4)p^2-1)^2=r^2⇔p^2+8x^2-16rx+16=0
が重解を持ちます。
このとき重解をαとおくと、(p-α)^2でp^2+8x^2-16rx+16は割り切れます。
p^2+8x^2-16rx+16=(p-α)^2*(p^2+2αx+8+3α^2)+(16-8α^2-3α^4)
であることから、この恒等式でα=2√3/3と求まり、r=8√3/9となります。




137 ◆R5P/AyayaA :03/05/12 11:33
>>136
ちょっと訂正
p^2+8x^2-16rx+16=(p-α)^2*(p^2+2αx+8+3α^2)+(4α^3+16α-16r)*p+(16-8α^2-3α^4)
であることから、この恒等式でα=2√3/3と求まり、r=8√3/9となります。
138たま@都立高3:03/05/12 14:17
直リンしないでくれといわれたんですが、これ以上約束やぶったら完全に絶交されると思う。というわけでmail欄

4次方程式 ax^3 + bx + cx +d = 0
が重解を持つときって(x-α)^2で割れる可能性もあるし、(x-α)^3や(x-α)^4で割り切れる可能性もなくないですか。
あと、r = 4/(3√3) =(4√3)/3じゃないですか?
139動画直リン:03/05/12 14:23
140132人目の素数さん:03/05/12 14:26
>>138
(x-α)^2で割れれば、(x-α)^3や(x-α)^4で割れる場合も含んでいるよ。
まず必要条件で(x-α)^2で割れるとして、結果αの値を求めて十分条件を
見れば済むことだと思う。rは書き間違いごめんなさい。
141132人目の素数さん:03/05/12 20:37
自作問題です。東大が好きそう。

xy平面上の 円C: x^2+y^2=1 の周上または内部を通る直線lを考える。
lを軸としてCを回転させてできる立体をKlとするとき、Klとして考えられ
る全ての立体の共通部分の体積を求めよ。
142132人目の素数さん:03/05/16 07:58
>>141
ムズイですが・・・
143132人目の素数さん:03/05/16 22:35
京大の好きな幾何の問題。

四角形ABCDの対角線AC,BDの中点をそれぞれM,Nとする。
AB+CD=x, AD+BC=y, AC+BD=z, MN=w とおくとき、
 x^2+y^2≧z^2+4w^2
を示せ。また、等号が成り立つのはどのような場合か。


これって何か名前がある定理?
偶然発見したんだけど。
初等幾何で発見したものは
名前がついてるか調べるには
幾何学大辞典が有効です

8冊(6冊と2冊使いみたいな
あるので調べるのも億劫でしょう
145132人目の素数さん:03/05/17 15:58
>>143
四角形ABCD→凸四角形ABCD
と訂正しておく。
146132人目の素数さん:03/05/17 17:36
長さが一定の弦ABを持つ円Γにおいて、弧ABを三等分する点のうちでAに近いほうをPとする。
A,Bを通るように円Γの半径を変えたとき、∠PABはある値を超えないことを示せ。

方針が決まれば結構カンタン
147132人目の素数さん:03/05/17 18:03
>>143
中線定理とトレミーの定理組み合わせてゴリゴリ計算したら何とかできた。
等号は□ABCDが円に内接するとき
148132人目の素数さん:03/05/17 18:38
>>141
問題おかしくね?
149132人目の素数さん:03/05/17 20:19
>>141
直線lに垂直で原点を通る直線m に垂直な面PでKlと中心O半径1の球Qをぶった切ると、
必ずKlの断面はQの断面を含んでいるので、共通部分にQが含まれる。
直線mの一部であり原点から見て直線lの反対側のKlに含まれない部分である半直線n
を考えてみると、共通部分には原点からの距離が1を超える部分が含まれないことがわかる。
というわけで共通部分は球Qだけで体積は4/3*pi。
150132人目の素数さん:03/05/17 20:20
彼女を一週間以上一ヶ月以下までにゲットする方法を教えろ。
>>152
3日以内じゃだめなの?
153主催者:03/05/17 20:50
【未経験】ITスクール【未経験】

ITスクールを開催しています。
授業料金は安く、即、実践へ活用できます!
只今、IT業界就職支援セミナーの値下げキャンペーン中。
http://homepage3.nifty.com/it3s/index.htm
154132人目の素数さん:03/05/17 22:16
>>147
正解。

別解として複素数を使う。(トレミーの定理の拡張の証明は複素数が楽だから。)
155132人目の素数さん:03/05/17 22:29
>>154
どうやって思いついたか教えて。
>>149
あんたはスゴイ!
美しい解法だ・・・
sinθ=cos(90−θ)

証明しろ。なかなか難しいぞ。(ニヤリ
>>157
単位円書いて以下略 100点
加法定理より以下略 70点
グラフ書いて以下略 30点

と勝手に予想してみる
ラマヌジャンは157のそれぞれを級数展開してごしごし式変形して確かめたとか。
まぁフォンノイマンの蜂が往復するのと同じようなヨタ話なんだが。
160132人目の素数さん:03/05/21 01:36
高校範囲でいいなら簡単ではないかな。
まず、加法定理を証明する。(数年前に東大に出たね。証明省略。)これを用いて
 {sinθ-cos(90゚-θ)}^2+{cosθ-sin(90゚-θ)}^2
= 2-2sin(θ+90゚-θ)
= 0
ところでsinx,cosxの値域は実数だから
 sinθ-cos(90゚-θ) = 0 かつ cosθ-sinθ = 0
よって示せた。


駄目?
161132人目の素数さん:03/05/21 01:36
162山崎渉:03/05/21 21:57
━―━―━―━―━―━―━―━―━[JR山崎駅(^^)]━―━―━―━―━―━―━―━―━―
163山崎渉:03/05/22 00:09
━―━―━―━―━―━―━―━―━[JR山崎駅(^^)]━―━―━―━―━―━―━―━―━―
164132人目の素数さん:03/05/22 22:10
問題
∠Pが鋭角の平行四辺形PQRSの対角線PRを直径とする円をCとする。
PQの延長とCとの交点をX、PSの延長とCとの交点をYとする。
Cの中心Oは△QRXの外接円と△RSYの外接円の少なくとも一方の円の内部(周も含む)に存在することを証明せよ。
lim[n→∞]a_n=0
であるとき
|a_N|<1をみたすNが存在することを示せ
とかいう問題を出したら高校生はどうするんだろうな
自明でいいんじゃない?高校生なら。


……よくないか。
最近、糞スレ多すぎ。
お陰でこのような良スレがdatオチの危機。

ageますよ?
>>165
高校生なら自明としか答えようがない。
もちろんεδを教えようとする講師が世の中には多くいるわけで,
そういう人はa_nを具体的に与えるのが筋だろう。

こんなふうに↓

a_n=n/(n^2+1)とするとき,次の問いに答えよ。
(1)命題「n>Nならば|a_n|<0.1」を真にするような自然数Nの最小値を求めよ。
(2)任意の正の実数εに対して,命題「n>Nならば|a_n|<ε」を真にするような
自然数Nが存在することを示せ。
169132人目の素数さん:03/05/30 18:16
東大受験生の掲示板もよろしく。

2004年度東京大学受験者@2ch2掲示板
http://www.2ch2.net/bbs/toudaijuken/index2.html
170132人目の素数さん:03/06/05 22:30







171132人目の素数さん:03/06/13 08:32
正四面体ABCDにおいて,ABの中点をM,CDの中点をNとおく。
MNを t:(1-t) に内分する点を通りMNに垂直な平面で
正四面体ABCDを切断して2つの立体に分割するとき,
それら2つの立体の体積比が1:2になるようなtの値(0<t<0.5)をt_1とおく。
また,
球を,その1つの直径をt:(1-t) に内分する点を通りその直径に垂直な平面で
切断して2つの立体に分割するとき,
それらの体積比が1:2になるようなtの値(0<t<0.5)をt_2とおく。

t_1 と t_2 の大小関係を調べよ。
t_1<t_2?
173ぼるじょあ ◆yEbBEcuFOU :03/06/16 17:57
(・3・)エェー α+ β+γ=1 αβ+βγ+γα=1 αβγ=1を満たすとき
Σ[K=1→100](α^100+β^100+γ^100)の値を求めYO!

式をよく見たらα、β、γの具体的な値が簡単に出ちゃうから欠陥問題かNA?
個々の値を求めずに解く方法を考えてNE!
174132人目の素数さん:03/06/16 18:58
(・3・)
175132人目の素数さん:03/06/16 22:59
    ∧_∧            ((
   (  ゚д゚ )          ) )
  /    \          ノ
  | |     | \        ((  ((
  | | /⌒|⌒|ヽ二二つ    )    ) 丿
  ヽ二二Ο./      \ (( ( (・3・)ノ
  (_| |_| |_       \ ∴∵
    .(__)__)       //》||ヾミ\
176132人目の素数さん:03/06/17 23:31
>>171
t_1 = t_2 ?
>>173
Newtonの公式
newotonの公式って、再帰的に求めてくやつだよな。
規則性が見つかれば楽だけど、見つからないと大変てことはない?
>>178
(・3・)エェーボク工房だからNewtonの公式ってのは知らないけDO!
周期は簡単にみつかるYO!

【解】
α、β、γは
x^3-x^2+x-1=0の解だから
S[n}=α^n+β^n+γ^nとおけば
漸化式 S[n+3]-S[n+2]+S[n+1]-S[n]=0・・・(・3・)を満たす
さらに nを一個ずらして S[n+4]-S[n+3]+S[n+2]-S[1]=0・・・(・∀・)
(・3・)+(・∀・)⇔S[n+4]=S[n]
∴S[n]は周期が4と分かる

∴Σ[K=1→100](α^100+β^100+γ^100)
=25(S[1]+S[2]+S[3]+S[4])

ところでS[1]=、S[2]=(α+β+γ)^2-2(αβ+βγ+γα)=-1
S[3]=S[2]-S[1]+S[0}=1、S[4]=S[0]=3

∴Σ[K=1→100](α^100+β^100+γ^100)
=25(S[1]+S[2]+S[3]+S[4])=100
180 ◆BhMath2chk :03/06/19 07:20
>>179
問題が違う。
(・3・)エェーごめんΣ[K=1→100](α^100+β^100+γ^100)が
Σ[K=1→100](α^k+β^k+γ^k)だった
そもそも元の式Kが入ってない。
ご迷惑をおかけしました
182132人目の素数さん:03/06/23 02:58
0<a<bとする。f(x)=x^n(nは自然数)のとき、不等式
 f'((a+b)/2)≦{f(b)-f(a)}/(b-a)
を証明せよ。
183_:03/06/23 03:09
184132人目の素数さん:03/06/23 08:25
>>182
なんか問題間違ってるだろ?
n=2,a=1,b=5のとき
左辺={(1+5)/2}^2=9
右辺=(5^2-1^2)/(5-1)=6
になるよ。
185_:03/06/23 08:36
186184:03/06/23 08:44
>>182
左辺は微分なのか。すまん。
責任もって(?)回答しとくわ。

まず示すべき式を同値変形していく。
(b-a)f'((a+b)/2)≦∫[a,b]f'(x)dx
(b-a)f'((a+b)/2)≦∫[a,(a+b)/2]f'(x)dx + ∫[(a+b)/2,b]f'(x)dx
右辺第二項の積分においてx -> b+a-t と変数変換してtをxと書き直すと
(b-a)f'((a+b)/2)≦∫[a,(a+b)/2]f'(x)dx + ∫[a,(a+b)/2]f'(a+b-x)dx
{(b-a)/2}f'((a+b)/2)≦∫[a,(a+b)/2]{f'(x)+f'(a+b-x)dx}/2dx
したがってこの式を示せばよいのだが、
ここでn=1,2の時は等号が成立が簡単に確かめられ。
n≧3のときはf'(x)の凸性により不等号が成立。
187184:03/06/23 09:02
何度もわるい。
×{(b-a)/2}f'((a+b)/2)≦∫[a,(a+b)/2]{f'(x)+f'(a+b-x)dx}/2dx
○{(b-a)/2}f'((a+b)/2)≦∫[a,(a+b)/2]{f'(x)+f'(a+b-x)}/2dx

分かると思うけど、凸性によりってところは
f(tx+(1-t)y)≦tf(x)+(1-t)f(y)  (凸性の定義)
において t->1/2, x->x, y->a+b-x をいれる。
188_:03/06/23 10:08
189_:03/06/23 13:03
190_:03/06/23 15:16
191_:03/06/23 17:24
192_:03/06/23 18:55
193_:03/06/23 20:20
194132人目の素数さん:03/06/26 01:04
ペル方程式ですが。

 a[n]={(2+√3)^n+(2-√3)^n}/2 (n=1,2,…)として、α[n]を x^2-2a[n]x+1=0 の小さくない方の解とする。
(1) 0以上の整数b[n]があって、α[n]=a[n]+b[n]√3 と書けることを示せ。
(2) x^2-3y^2=1 の整数解は無数に存在することを示せ。
今更だが、(1)は
α[n]=a[n]+b[n]√3(b[n]≧0)と書けることを示せ。
の方が入試問題っぽいな。
知識があれば容易に解けてしまうような問題は、
大学入試問題にふさわしくないというようなことが
書いてある手引書みたいなものが各大学に配られているそうだよ。
197194:03/06/26 02:24
>>196
一応>>194は一般解を与えて知識は使わないようにしてあるつもりだけど、
やっぱりa[n]が整数になるというのが知識にあたるのかな。
(ということはやっぱり(1)はb[n]が整数であることを示すまで問題にしたほうがいいのか)
198196:03/06/26 02:40
ごめん問題文読んでなかった。ペル方程式っていうのに条件反射してしまった。
199132人目の素数さん:03/06/28 13:05
問題です。(2)を(1)なしで((1)の事実を使わずに)解くのもあり。

(1) f(x) = (x+1/2)log(1+1/x) (x>0) は単調減少関数であることを示せ。
(2) nが自然数のとき (n+1/2)logn - log(n!) ≧ n-1 であることを示せ。
200get
201132人目の素数さん:03/06/28 13:21
>>199は(1)普通にいらなかったです…。
202そこで問題です。:03/06/28 14:47
「A君は毎日学校を午後5時に終わると、
ちょうどその時刻に学校に迎えに自宅からきたベンツに乗って帰ります。
ある日学校が早く終わったので、
午後4時に学校を出て自宅のほうに歩き始めました。
途中で迎えのベンツに乗り自宅へ向かったら、
いつもより10分早く帰りつきました。
A君とベンツはそれぞれ一定の速さで進むとするとき、
A君とベンツの速さの比を求めなさい。」

20分を目標にといてね。


1:11じゃなかったっけ?
204伝道師:03/06/28 15:16
205そこで問題です。:03/06/28 15:35
それでは今年の中学入試問題より。

21個のクッキーをA〜Eの5人が、
ある順番で一人1回ずつ好きなだけ食べました。
5人の話は以下の通りです。
A「残っていたうちの3分の2の個数を食べました。」
B「残っていたうちの半分の個数を食べました。」
C「残っていたうちの半分の個数を食べました。」
D「残っていたうちの全部を食べました。」
E「みんなが違う個数を食べました。」

次の問いに答えなさい。
(1)最後に食べたのは誰ですか?
(2)Aは何番目に食べましたか?
(3)Eは何個食べましたか?       (03 鎌倉学園)
  

>>205
他のスレで同じ問題見かけたが
おもしろい〜じゃなかったっけ?
208132人目の素数さん:03/06/28 15:51
>>208
正解。
210132人目の素数さん:03/06/30 02:47
△ABCにおいてAB=b,AC=cとし、外接円の半径をRとする。このとき
 cos{(1-t)A}/b + cos(tA)/c = 1/R
をみたすような実数tが存在することを証明しなさい。

少し簡単すぎるか?
211ちよ:03/06/30 02:50
212132人目の素数さん:03/07/01 06:58
pが素数のとき
(1)7^p-5^p-2^p はpで割り切れるか?
(2)2^(p-1)≡1 (mod p) を示せ。
(2)はp=2で成り立たないが。
>>205
(1)D(2)4番目(3)9個
kana?
>>205
E(21−15),?(6−3),A(3−2),D(1−1),?(0−0)。
E(21−9),?(12−6),A(6−4),D(2−2),?(0−0)。
E(21−9),?(12−6),?(6−3),A(3−2),D(1−1)。
地図上に任意の円を書き込み、その内部に任意に点をとる。
このとき、その点を通る直線で、円との2交点の標高が等しくなるようなものが存在することを証明せよ。
ただし、地形は連続とする。

どっかのサイトにあったのを思い出した。東大文系向きかな。
>>216
正直問題の意味がよく分からん
218132人目の素数さん:03/07/05 02:33
微分可能な周期関数f(x)の周期がTである時、
f'(x)も周期Tの周期関数である事を証明せよ。

少なくとも去年の「π>3.05」よりは良問だと自負するのだけど…笑わないで。
219_:03/07/05 02:34
220132人目の素数さん:03/07/05 02:45
>>218
f(x)=f(x+T)
両辺微分して
f'(x)=f'(x+T)
よって証明された

じゃだめ?
221218:03/07/05 02:49
加法定理の問題みたいにやらせる必要ないか…うむ。アホな事をした。
ついでに周期がT/2とかでない事を示しとけばOK>>220
>>216
文系って中間値の定理習うんだったっけ?
理系相手に出しても簡単すぎなのは確かだが
そこは文系ならではの論証力でカバーだよ。
中間値の定理を用いない解法もあるし。
224132人目の素数さん:03/07/09 23:52
ちょっと簡単かもしれないが、たぶん受験生の半分くらいしか正解できないと思う。(根拠はない)

0または1からなる任意のn文字列を無作為に並び替えるとき、
再び元の文字列に一致する確率を求めよ。
225132人目の素数さん:03/07/09 23:57
>>224
不親切な出題のせいで正答率は低そうだな(w
226132人目の素数さん:03/07/10 00:03
糞問題ですが
a_(n+1)=(k*a_n)^k ,k=n/(n+1)を満たす数列の極限を求めよ。
初項は?
228132人目の素数さん:03/07/10 00:08
あ。a_1=1
です。
229132人目の素数さん:03/07/10 00:10
>>226
君の言うとおり糞問題だね。なんのひねりもない
230132人目の素数さん:03/07/10 00:20
そうか?結構計算力と色々な分野の力が試されると思うんだが
231132人目の素数さん:03/07/10 00:31
>>226
1
232226じゃないけど。:03/07/10 00:37
>>231
不正解。
答えは、1/e でしょ?
233132人目の素数さん:03/07/10 00:38
>>232
正解です。229の言うとおりひねりもございませんが出題しました。
234232:03/07/10 00:42
ごめん。おいら証明できてないよw

もうちょっとでできそうな感じだけど
235132人目の素数さん:03/07/10 00:59
そうでしたか。まあ回答があっているからいいんじゃないですか
236234:03/07/10 01:01
>>233
log x の積分間違えて時間かかたけど解けた。
結構面白い問題だとおもうよ。
237132人目の素数さん:03/07/10 01:02
そうですか。ありがとうございます。
作った甲斐がありました。
238132人目の素数さん:03/07/10 01:11
>>225
>>224のような不親切な出題は京大を意識しているんだけど。
問題文をどのように読み取ったかを明記する必要がある。
239132人目の素数さん:03/07/10 01:37
みんなすごいね。どんな人たちなの?数学の教師とかやっぱ多いんですかね?
漏れ、数学大好きなんでこのスレみてるとなんだかゾクゾクしてきちゃいますよ。
240132人目の素数さん:03/07/10 01:38
へー
241132人目の素数さん:03/07/10 01:40
単位円上の正n角形を考える。
正n角形より正n+1角形の方が面積が大きいことを示せ。
またn→∞としたときπに収束することを示せ
長さ2の線分ABを直径とする円周Cがある。

AB上から1点Pを、C上から1点Qを、それぞれ無作為に選び、
PQを通る直線がCと交わる点のうち、Qとは異なる方をXとする。

このとき、弦QXの長さが√3より大きくなる確率を求めよ。
>>242
長さとか面積が関わる確率って高校でやるの?
244132人目の素数さん:03/07/10 03:23
242はいい問題だ
なんか、ベルトランのパラドックスみたいな状況に陥りそうな問題だな。
246132人目の素数さん:03/07/10 12:53
ていうか作ったヤツも、解答を用意できてない罠
247242:03/07/10 13:18
>>244
サンクス。
>>245
ばれたか。実はその話をモチーフに思いついた問題なんだ。
点の分布が指定されてるから、パラドクスにはならないと思う。
>>246
できてるよ。ヘッポコなので3時間くらいかかったけどね‥‥
原液の手練れ受験生なら30分くらいで行けるんじゃないだろうか。

一応値だけ書いとくと、(1/3) + (log(2+√3))/πになった。
計算間違いしてる可能性もある。
248aaad ◆ozOtJW9BFA :03/07/10 21:27
>>205さん、それ算数オリンピックの漫画に載ってなかったっけ?
249132人目の素数さん:03/07/10 23:03
a,b,c,dは異なる自然数で、a+b+c+d=100,2a+3b=70,c<d,である。
このとき、a*b*c*d
の値を最大にするa,b,c,dの値を求めよ。
250132人目の素数さん:03/07/11 00:13
良くこんなの解けるな。
ホント数学ってできねーわ。
数学って努力じゃできるようにならん気がするよ。
95%センスで残りは論理力?
なぞなぞとかマジカル頭脳パワーとかすぐにひらめくやつと似てる。
251132人目の素数さん:03/07/11 00:21
無数の直線が平行に並んでいる。
今、この平行線の間隔の丁度半分の長さの線分を無造作に
同じ平面上に置いたとき、平行線のいずれかにこの線分が
交わる確立が1/Πになるのを証明せよ。
252132人目の素数さん:03/07/11 00:22
>>251
つまらん
253132人目の素数さん:03/07/11 00:28
>>252
どうやって証明すんの?
254132人目の素数さん:03/07/11 00:30
>>251
それ昔、洛南の入試に出てたね。
>>252
答えてみろよ。
255132人目の素数さん:03/07/11 00:37
>>252晒し上げ
256252じゃないが:03/07/11 00:50
やっぱつまらん。
257132人目の素数さん:03/07/11 00:52
>>251
分母のキゴウは何?
259132人目の素数さん:03/07/11 03:34
>>249
(a,b,c,d)=(20,10,34,36)
260132人目の素数さん:03/07/11 03:43
241の問題は糞だな。
単調増加なんか簡単に言えるしπに収束なんて円になることを考えれば
明らか
a,b,cはいずれも2以上の整数とし、最大公約数は1である。
このとき、命題「a,b,cから互いに素であるような2数がとれる。」は真か偽か。
真なら証明を、偽なら反例をあげよ。
262132人目の素数さん:03/07/11 04:17
>>260
お前はダメだしだけしかできないだろ。
問題作るなり解答するなりすれば?
>>261
2*3、3*5、5*2
264132人目の素数さん:03/07/11 04:25
>>249
259は間違ってた。
(a,b,c,d)=(17,12,35,36)
計算量多くない?
265132人目の素数さん:03/07/11 05:49
一辺の長さが1の立方体をある平面で切断したときの断面の最大値を求めよ。
266132人目の素数さん:03/07/11 12:05
>>249を作った人は、エレガントな解法を死ってるんじゃないかな?
ただの計算問題だった・・・、なんてオチじゃないよね?
ここはエレガントな問題を出すスレだから・・・
267132人目の素数さん:03/07/11 17:02
【問】★
三次元空間上の点aを始点とする単位ベクトルuと、bを始点とする単位ベクトルvとが
同一平面上にないための必要十分条件を求めよ。ただしaとbは異なる。
【問】★★
3×nの方眼紙の升目それぞれにω^2,ω,1のいずれかを配置する。
このとき升目の数の総和が0になるような配置の仕方は何通りあるか。
ただしω=(1+i√3)/2 (iは虚数単位)とし、方眼紙は回転などはさせない。
268132人目の素数さん:03/07/11 17:11
>>251
ビュフォオンの針
三角比と連続事象の確率でおしまい。
269132人目の素数さん:03/07/11 19:06
>>268
きちんと解答しる
>>269
計算がちょっとだるいだけで、ヒネリはないよ。
自分でやるよろし。
>>265
東工大に類題

>>269
数学苦手だが力技で、、、、、

線分の一方端点Aが2直線の間にあるとき、
もう一方の端点Bの取り方によって交わるか交わらないかを決められる。
Aと一番近い直線との距離をxとすると交わる確率Pは、
 P=arccos[x/AB]/π  −@
よってxの範囲は0<=x<=ABであるから、
その範囲で@を積分して終了
272山崎 渉:03/07/12 12:28

 __∧_∧_
 |(  ^^ )| <寝るぽ(^^)
 |\⌒⌒⌒\
 \ |⌒⌒⌒~|         山崎渉
   ~ ̄ ̄ ̄ ̄
273132人目の素数さん:03/07/12 21:17
>その範囲で@を積分して終了
きちんと積分しる
274132人目の素数さん:03/07/12 21:17
B
無修正画像用の板 ロリもあって、管理人は神(ネ申)

http://www.hl-homes.com/
275mathmatica:03/07/12 22:27
なにか新しい問題だしていただけないでしょうか?
最近やっと数学が少しずつわかってきたので。
初めのころに書いてあった幾何の問題とかも面白いですね。
勉強になりました。
276mathmatica:03/07/12 22:31
なんだか予備校で東大は・・・
フィボナッチ数列が大好きだとかいってたんですが。
そんな感じのこともやっぱりやっておかないとだめなのかなあ。
277132人目の素数さん:03/07/13 21:16
どのくらいぶりにここに来ただろ。
僕が作ったんじゃないけど、
大学入ったときにできた友人に出された問題。
彼は自分で作ったと言っていました((2))
今でも印象に残っています。ってここ見てたりして。

(1) 半径rの3つの円が1点のみを共有しているとき、
この点以外の残りの3つの交点が半径rのある円上にあることを証明せよ。
(2) (1)のとき、2つの円の共通部分は3つできるが、
その面積の和の最大値を求めよ。
(図を書けないので、ちょっとわかりにくい説明ですが)
278132人目の素数さん:03/07/15 00:31
p>2の素数のとき

(3^p-3)/(2^p-2)

が整数になるようなpの値は幾つあるか。
lim[x→∞]∫[0,x](1+t^2)dt/(1-t^2+t^4) を求めよ。
280山崎 渉:03/07/15 12:37

 __∧_∧_
 |(  ^^ )| <寝るぽ(^^)
 |\⌒⌒⌒\
 \ |⌒⌒⌒~|         山崎渉
   ~ ̄ ̄ ̄ ̄
281132人目の素数さん:03/07/17 00:18
a,bは自然数、[x]はxを超えない最大の自然数のとき、

[5a+πb]で表せない自然数をすべてあげよ。
1, 2, 3, 4, 5, 6, 7, 9, 10, 12
以上。
15いける?
284282:03/07/17 01:19
いけません。書き間違えました。
285132人目の素数さん:03/07/17 01:47
  /ヘ;;;;;  
  ';=r=‐リ  
  ヽ二/       n  
 ̄     \    ( E) グッジョブ!!
フ     /ヽ ヽ_//
286132人目の素数さん:03/07/17 07:34
241の問題はレベルとかも高すぎず低すぎないいい問題だと思うけどねえ
287286:03/07/17 07:37
286は275さんへのレスです。
288132人目の素数さん:03/07/17 13:12
糸を円になるよう結び、机に置くとさまざまな形ができる。
この形で最も面積が大きくなるのはどのようなときか?
289_:03/07/17 13:13
張り詰めたときだから同心円のときなんっすかねぇ?
感覚的には
291132人目の素数さん:03/07/17 13:14
初めて日本(東京など)で開催中の第44回国際数学オリンピックで、
日本代表の高校生6人は、灘高(兵庫県)2年の西本将樹さんが金メダルを獲得、銀3人、銅2人と全員入賞の好成績をあげた。
数学オリンピック財団が17日発表した。
 82の国と地域から高校生以下458人が参加。
各国代表(6人)の合計点でも日本は9位で、
92年のモスクワ大会8位に次ぐ成績。上位はブルガリア、中国、米国の順。

 銀メダルは、入江慶さん(東京・筑波大付属駒場2年)、
尾高悠志さん(同3年)、大島芳樹さん(同3年)。銅メダルは、
足立潤さん(神奈川・栄光学園3年)、長坂友裕さん(愛知県立岡崎3年)。金は37人、銀は69人、銅は104人に贈られた。

(07/17 11:44)

292132人目の素数さん:03/07/17 13:15
>>290
証明どうする?
293132人目の素数さん:03/07/17 21:49
>>288
> 糸を円になるよう結び、机に置くとさまざまな形ができる。
> この形で最も面積が大きくなるのはどのようなときか?

>>293
証明ってどうやるんだっけ?変分法が必要か?
>>294
http://pascal.edu.gunma-u.ac.jp/~ootake/papers/tousyuu/node3.html
ここの定理5参照、証明付きです
なるほどなぁ
297132人目の素数さん:03/07/21 16:36
なろほどなぁ
298132人目の素数さん:03/07/23 23:29
√5=a+b*√2
を満たす整数a,bが存在しないことを示せ。
存在すると仮定する
両辺2乗して 5=a^2+2b^2+2ab√2
√2が無理数であることに矛盾
>>299
それだと厨房レベルだな。
せめて√2が無理数である証明まで入れれば何とか高校生が解く問題にはなるが。
そもそもこの問題、東大・京大入試問題じゃありえないだろ。
301132人目の素数さん:03/07/24 00:02
√2が無理数てあることの証明ってネタですか?
ネタ、とわ?
303132人目の素数さん:03/07/24 01:11
√2が無理数の証明ってそこまで難しいのか?
中学生でも証明できる。 < むりすー
最近の東大にはそのレベルの問題も出てくる…

つぅか暗記型の学習してる奴が解けないような問題を出したいのなら
π>3.05とかよりもビーカーのような問題を出せやゴルァ
306132人目の素数さん:03/07/24 01:42
x^n+y^n=z^n (nは3以上の自然数)の自明でない整数解は存在しないことを示せ。ただし、谷山・志村予想は正しいと仮定してよいとする。
307132人目の素数さん:03/07/24 01:43
log2<0.699 を証明せよ
308132人目の素数さん:03/07/24 01:48
漸化式a_(n+1)=4a_n(1-a_n) で定義される数列の一般項を求めよ
309_:03/07/24 01:49
310132人目の素数さん:03/07/24 01:51
2より大きい任意の偶数はある二つの素数の和で表せることを示せ。
>>307は意外に面倒?
312_:03/07/24 01:58
313307:03/07/24 02:04
>>311
1/xのグラフの面積の台形近似でいいんだけど
範囲をきつくしたからちょっと面倒…というより無駄に煩雑にしてみた。

0≦x≦1/3において、1/(1-x^2) = 1 + x^2/(1-x^2) ≦ 1 + (9/8)x^2
この両辺を[0,1/3]で積分すればよい。
315132人目の素数さん:03/07/24 03:36
3以上の素数を小さい順に並べた数列
 3, 5, 7, 11, …
を考える。この数列の任意の隣り合った2項の和は
少なくとも3つの素因数をもつことを示せ。
>少なくとも3つの素因数をもつことを示せ。
 
この表現は微妙〜。
連続する3つの整数の和が3の倍数になることを示せ。
n+(n+1)+(n+2)=3n+3=3(n+1)
319132人目の素数さん:03/07/24 04:04
>>315
隣り合った2項をx、y(x<y)とおくと、x、yが奇数だからx+yは2を素因数
に持つ。x+y=2p(pは素数)と仮定すると、2x<x+y<2yだから2x<2p<2y
すなわちx<p<yしかし題意よりこのようなpは存在しない。
よってx+y=2k(kは素数でない自然数)が成り立ちkは2つ以上の素因数を持つ。

320319:03/07/24 04:10
x+y≧8だからp>3、k≧4です。
一応。
321_:03/07/24 04:11
>>308
そういや漸化式a'=4a(1-a)って昔登場したような気がする。
その式は、カオスでよく見かける奴か?
おれはよく知らんが。
324132人目の素数さん:03/07/24 05:32
(1)a,b,c(a≦b≦c)が自然数のとき、abc=a+b+c、を解け。
(2)a,b,c,d(a≦b≦c≦d)が自然数のとき、abcd=a+b+c+d、を解け。
(3)a,b,c,d,e(a≦b≦c≦d≦e)が自然数のとき、abcde=a+b+c+d+e、を解け。
(4)a,b,c,,,y,z(a≦b≦c≦・・・≦y≦z)が自然数のとき、abc…yz=a+b+c+…+y+z、を解け。
325132人目の素数さん:03/07/24 05:41
(a + b + c)^4 と 27(a^2 b^2 + b^2 c^2 + c^2 a^2) の大小を比較せよ
326_:03/07/24 05:44
327132人目の素数さん:03/07/24 05:50
>>325
定まらないようだ…

(a + b + c)^4 ≧ k(a^2 b^2 + b^2 c^2 + c^2 a^2) が
常に成り立つようなkは存在するか?
存在する。例、k=0
329132人目の素数さん:03/07/24 06:07
K>0では?
330132人目の素数さん:03/07/24 06:48
>>324
(1)だけでも教えてください。
331132人目の素数さん:03/07/24 09:23
>>324
(3)になったとたん、
(1,1,1,2,5)、(1,1,2,2,4)、(1,1,1,3,3)
と三つもでてきた。もう無理ぽ・・・
332132人目の素数さん:03/07/24 09:28
2つの自然数x、yに対して、その最小公倍数をA、最大公約数をBとする。
このときxy=ABを証明せよ。

式は有名と思うけど、証明は知らない人が結構いると思う。自分で示そうと思っても
なかなか難しい。
333132人目の素数さん:03/07/24 09:29
>>332
嘘つけ。
少なくとも東大で出るようなレベルじゃないだろ。
334132人目の素数さん:03/07/24 10:00
>>332
一分で出来たんだけど・・。
素因数分解の一意性を証明せよとか、出てきたら
解けないやつ多いだろうなぁ
336132人目の素数さん:03/07/24 11:23
一意性の意味がわからず大学院生やっていますが何か?
337132人目の素数さん:03/07/24 11:27

logx+log(logt)+t^2=√(x-t)
tについて求めよ。

>>332
大学入試には難しすぎる。
339132人目の素数さん:03/07/24 13:42
>>332
初等整数論
340トップエリート街道さん ◆BIG6e4aEMg :03/07/24 15:01
>>327 これならどう?

a, b, cを正の数とするとき、
(a + b + c)^4 ≧16(a^2 b^2 + b^2 c^2 + c^2 a^2)
を証明せよ。
341132人目の素数さん:03/07/24 15:03

期間限定!もうお目にかかれない!

http://alink3.uic.to/user/angeler.html
342132人目の素数さん:03/07/24 16:40
>>324
(1)
1.2.3.ダー!
343supermathmania ◆ViEu89Okng :03/07/24 17:18
中学、高校の知識でも素因数分解の一意性は示せるが、私は大学で初めて素因数分解の一意性を見た。
これを示すにはabが素数pで割りきれるならばaがpで割りきれるかbがpで割りきれることを証明すればよい。
(古くから知られているにも関わらず、これの証明が大変だ。)
344132人目の素数さん:03/07/24 20:35
(5)a,b,c(a≦b≦c)が自然数のとき、abc=a+b+c、の最大値を求めよ。
(6)a,b,c,d(a≦b≦c≦d)が自然数のとき、abcd=a+b+c+d、の最大値を求めよ。
(7)a,b,c,d,e(a≦b≦c≦d≦e)が自然数のとき、abcde=a+b+c+d+e、の最大値を求めよ。
(8)a,b,c,,,y,z(a≦b≦c≦・・・≦y≦z)が自然数のとき、abc…yz=a+b+c+…+y+z、の最大値を求めよ。
345311:03/07/24 23:26
>>307
見事にはめられた?
expの方が良い近似式がでるだろうから…とか考えてやったけど(try and errorで)面倒だった
いくら何でも台形近似はないだろ、とか思い込んだのは大失敗

とはいえ、失敗でも10分程度で解答可能だから甘いね
346受験生:03/07/25 00:13
>>307台形近似じゃできなかったんだけど…どうすればいいの?
347132人目の素数さん:03/07/25 01:55
>>344
意味ない
348132人目の素数さん:03/07/25 02:46
円周率は3.14…で、sqrt(10)=3.16…ですよね。そこで、

  π < sqrt(10) を証明せよ。

というのはどうですか? 区分求積法でも使うのかなあ?
349トップエリート街道さん ◆BIG6e4aEMg :03/07/25 03:05
π^2=6ζ(2)<6(1+1/4+1/9+1/16+1+25+1/5)=5989/600<10^2
>>349
一番右の不等式でミスったねw
つーか、その公式は使っちゃいやん
351トップエリート街道さん ◆BIG6e4aEMg :03/07/25 03:27
>>350
あっ、しまったw
しかももう1箇所打ち間違えている・・・
353132人目の素数さん:03/07/25 03:45
一辺の長さが1の正五角形を線分によって面積を2等分する。
このとき線分の長さのとりうる範囲を求めよ。
354厨房3:03/07/25 04:43
>>324 344
解けそうで駄目ポ
ギブアップ
355132人目の素数さん:03/07/25 05:31
>>354
ほとんど1になるんじゃない?
356132人目の素数さん:03/07/25 05:36
kを素数、xを3以上の自然数とするとき、
√x<k<xとなる
素数kが必ず存在することを示せ。
nを自然数とするとき、
n<k<2nとなる
素数kが存在することを示せ。
358132人目の素数さん:03/07/25 14:39
>>357
おまえつまらなすぎる
>>324
(4)
(a,b,...,v,w,x,y,z)=(1,1,...1,1,1,2,26),(1,1,...1,1,1,6,6),
        (1,1,...1,1,2,2,9),(1,1,...1,2,2,2,4)
の4個ですな。
360132人目の素数さん:03/07/25 17:33
>>348がうまくイカン。

そういや2000年前後の京大の入試問題で e^π > 21 を示すのがあったな。
記憶がアヤシイが、e^xとsinの積の関数を積分してたような…
361_:03/07/25 17:36
>>348
単位円に外接する2^n角形の面積と√10の比較を根性で続ける(w
単位円に外接する正n角形の面積=n*tan(π/n)
π<24*tan(π/24)=24(√6-√3+√2-2)<√10
これは,966528√2>1366873から導ける。(w
364ビッグバン宇宙論は完全に間違いだった!:03/07/25 19:22
科学者よ、恥を知れ!
ビッグバン宇宙論は完全に間違いだった!
科学の原則を無視した、デタラメのインチキ理論だったのだ。
そして、そのビッグバン宇宙論の世界的な浸透は
アメリカ、ユダヤ・キリスト教勢力による世界支配のための思想的な戦略なのだ!
また、ビッグバン宇宙論の思想によって戦争が起こり、
貧富の差がひらき、終末的な絶望感が世界に蔓延しているのだ。
ビッグバン宇宙論は世界の平和を揺るがす、悪の元凶となっているのだ。
ビッグバン宇宙論とは、
「宇宙は『無』からビッグバン(大爆発)によって誕生した」という理論である。
この理論は、ユダヤ・キリスト教の創造神話(神が天地を創造した)そのものである。
ビッグバン宇宙論の実態は、科学理論ではなく宗教思想なのである。
『無』は科学的に証明できるものではなく、
そして、『無からの誕生』も科学では証明できるものではないのだ。
ビッグバン宇宙論が科学の正統であるという思想を、世界中の人々に
浸透させる戦略が成功したことにより、ユダヤ・キリスト教勢力の
世界における優位性が確立されていったのだ。(20世紀に)
そして、その思想的支配の最大の例が、アメリカやイギリスによる
イラク戦争なのだ。
ビッグバン宇宙論の浸透により、世界中に終末思想(世界の終わり)が
蔓延してしまっている。
そのことにより、自己中心的、せつな的、短絡的な考え方が
社会に広がっている。
科学的に間違っているビッグバン宇宙論から脱却しなければならない。
そして、宇宙は無限だということを理解しなければならない。
人間は本当の宇宙観、世界観を構築し、
新しい時代に進んでいかなければならないのだ。
ビッグバン宇宙論が世界を支配している限り、平和な世界にはならないのだ。
そのことを科学者は重く受けとめるべきである。
平和を返せ!!!!!!!!!!!!!!!!!!!!

365厨房3:03/07/25 21:07
>>355 359
すばらしいですね。
(1)の解き方をおしえてください。

実は、これが完璧に(途中の式を含む)出来れば先着1名まで
夏休みの数学の宿題が免除されるんです。
y=ax^2+bx+c を解け
a = (y - bx - c)/x^2
368132人目の素数さん:03/07/25 23:47
p,q は p<q をみたす素数として
 Σ[k=1,q] (kp を q で割った商) = pq/2
が成り立っている。このような p,q を求めよ。
369132人目の素数さん:03/07/25 23:51
(a-x)(a-b)(a-c)・・・(a-y)(a-z)(a-1)(a-2)(a-3)・・・(a-98)(a-99)
a=xの時の展開式を求めよ。
370132人目の素数さん:03/07/25 23:52
>>369
2秒で解けた。
371132人目の素数さん:03/07/25 23:53
>>369
ミス?
372132人目の素数さん:03/07/25 23:55
(x-a)(x-b)(x-c)・・・(x-y)(x-z)(x-1)(x-2)(x-3)・・・(x-98)(x-99)
この式の値を求めよ。
じゃない?
373132人目の素数さん:03/07/25 23:56
なんかむかしあったね。
適当に数字を連呼して(1つだけ0を混ぜる)積を求めよ。
っていうやつ。
>>372
けっきょく
  ……
の最後に(x-x)が入っていて答えが同じヨカーン
375132人目の素数さん:03/07/26 00:05
>>374
ただの謎々なんダローヨ。
376365の厨房3:03/07/26 00:09
みなさんシカトですか…

誰か・・
377132人目の素数さん:03/07/26 00:12
>>376
スレ違いでつ
>>376
本人が解いたわけではないのに宿題免除は、その主旨に反する。
379132人目の素数さん:03/07/26 00:17
>>376
遊休増やしてくれるんなら考えてやるよ。
俺は仕事が忙しい。
380132人目の素数さん:03/07/26 00:22
>>357
>>356より強い条件ですな。
>>356を解くには
>>357をまず解かなくてはならないことは
ないと思うが…
>>376と同じ学校の奴が見ていて、
誰か2ちゃんで聞いてたぞと。
382誰か=:03/07/26 00:38
381
>>368
座標平面上にy=px/qという直線を引いて
A(0,0)B(q,0)C(q,p)で出来る三角形の内部の格子点を数えれば
Σ[k=1,q](>>368の奴)=(p-1)(q-1)/2である事が分かるから
そのようなp,qはない。
>>368
残念。
385368:03/07/26 01:24
>>384>>383の間違えです。
何自分にレスしてるんだ_| ̄|○

3行目のΣ[k=1,q](>>368の奴)=(p-1)(q-1)/2が間違い。
Σ[k=1,q-1](>>368の奴)=(p-1)(q-1)/2が正解。

そんなわけで答えはp=2,q=3でした。
386365の厨房3 芝中:03/07/26 02:51
>>377
承知してます。
でも、こっちの人達の方が頭良さそうと思って…
>>378
その通りなんですが、これでも2時間位考えて
(1)は(1・2・3)は判って、あと、aは1しかあり得ない事までは
判りました。但し証明はできません。
>>381
そうですか… 正直、もう解いちゃった人はいる気がしてます。

もう、諦めちゃおうかな。グスン
>>386
諦めろ!
おまえ自身の将来のためにもな
( ´,_ゝ`) プッ
388132人目の素数さん:03/07/26 05:05
>>360
1999年東大
∫[0,π](e^x)sin^2(x)dx > 8 の証明で出てくる
389132人目の素数さん:03/07/26 08:15
>>386
漏れが証明を用意したる。
390132人目の素数さん:03/07/26 08:56
@の証明
x+y+z=xyz
x≧y≧zの場合
2^3>2+2+2
2^2<2+2+2より、
少なくとも1を1つ含む。
z=1とすると
xy=x+y+1 
因数分解して
(x-1)(y-1)=2 x≧yなので
解はx=3,y=2のみ。
よって解は(x,y,z)=(3,2,1)
Axyzw=x+y+z+w
 x≧y≧z≧wの場合
2^4>2+2+2+2
2^3>2+2+2+1
2^2<2+2+1+1より
少なくとも2つは1を含む
w=z=1とすると、
xy-x-y=3より
(x-1)(y-1)=3
x≧yより、
x=4、y=2のみ
よって、(x,y,z,w)=(4,2,1,1)

391132人目の素数さん:03/07/26 08:57
B(中略)
 の場合
2^5>2+2+2+2+2
2^4>2+2+2+2+1
2^3=2+2+2+1+1
3・2^2>3+2+2+1+1
2^2<2+2+1+1+1
2^3の解は(2,2,2,1,1)
少なくとも他の解は1を3つもち
(x-1)(y-1)=4
x≧yより
他の解は(3,3,1,1,1) (5,2,1,1,1)
一時中断
392132人目の素数さん:03/07/26 09:24
C(中略)
の場合
2^5>2+2+2+2+2+1×21
2^4<2+2+2+2+1×22
よって少なくとも解は1を22もち
1を25、24持つものは
(x-1)(y-1)=25 x≧yなので、(26,1……)(6,6,1……)
23持つものは
xyz=x+y+z+23
z=2の場合        z=3の場合     z=4の場合
2xy=x+y+25       3xy=x+y+26     4^3>4+4+27(x≧y≧z)
因数分解して      因数分解して    なので解が存在しない
(2x-1)(2x-1)=51    (3x-1)(3y-1)=79
x≧yより(9,2,2……)  この整数解は存在しない。
393_:03/07/26 09:28
394132人目の素数さん:03/07/26 09:55
22持つものは
xyzw=x+y+z+w+22
w=2の場合      w=3の場合
2xyz=x+y+z+24    3^4>3+3+3+25(x≧y≧z≧w)
           なので解が存在しない
w=2のうち
z=2の場合      z=3の場合
因数分解して     2×3^3>3+3+3+24(x≧y≧z≧w)
(4x-1)(4y-1)=105   なので解が存在しない
この解は
x=4,y=2のみ。
よって(9,2,2,2……)
これより、Cは4つの解(26,1……)(6,6,1……)(9,2,2……)
(4,2,2,2……)の四つ。
395132人目の素数さん:03/07/26 09:56
×(9,2,2,2)
○(4,2,2,2)
396132人目の素数さん:03/07/26 09:58
早いもん勝ち。
397132人目の素数さん:03/07/26 09:59
>>390
2^3>2+2+2
2^2<2+2+2より、
少なくとも1を1つ含む。

これどういう意味ですか?すいません教えてください。
398132人目の素数さん:03/07/26 10:00
>>394
この解は→この整数解は
399132人目の素数さん:03/07/26 10:01
誤植ですよ。
2^2<2+2+1に直して。
400132人目の素数さん:03/07/26 10:11
厨房!!起きろ!!
ってか早くネットつなげ!!
しかしここの先生もセンスない出題するな・・・
401132人目の素数さん:03/07/26 10:16
>>394
z=3の場合が
要らない気が。
402132人目の素数さん:03/07/26 10:22
芝中って
進学校なの?
403132人目の素数さん:03/07/26 10:39
>>392
2^5>2+2+2+2+2+1×21
2^4<2+2+2+2+1×22
よって少なくとも解は1を22もち

これどういう意味ですか?すいません教えてください。
つーか分からない問題スレいけば?
405132人目の素数さん:03/07/26 10:46
2^5×1^21>2+2+2+2+2+1×21
2^4×1^22<2+2+2+2+2+1×22
って意味。
406132人目の素数さん:03/07/26 10:48
間違えた
2^4×1^22<2+2+2+2+1×22
ですね。
407132人目の素数さん:03/07/26 11:00
×1を25、24持つものは
(x-1)(y-1)=25 x≧yなので、(26,1……)(6,6,1……)
○24持つものは、
(x-1)(y-1)=25 x≧yなので、(26,2……)(6,6,1……)に
訂正。
408132人目の素数さん:03/07/26 11:06
×Cは4つの解(26,1……)(6,6,1……)(9,2,2……)
 (4,2,2,2……)の四つ。
○解の1つを(26,2,1……)に訂正。
409132人目の素数さん:03/07/26 12:12
芝中はくそ
(3)は等号が入ってなければまともな問題なんだが。







まあ、くそってこった。
411365の厨房3 芝中:03/07/26 21:50
390を始め、心有る皆様へ。
ありがとうございました。スッキリしました。
やはり、ここでアガイテ正解でした。
スレを荒らしてしまってごめんなさい。

因みに、例の問題の答は出題後5分で俺の友達が既に
解いていてしまったそうです。(泣)

因みに芝中とは…
すげ-昔からある学校で、300人中
東大=10人前後 早稲田・慶応=100人前後の入学者を出す
進学校ともDQN校とも言えない中途半端な学校です。

>>379 社長さんに、頼んでおきました。
   2日増やしてくれるそうです。
>>409 まー、否定はできません。


   
   
>>411
中学から東大なんて超すげーじゃん!
413365の厨房3 芝中:03/07/27 00:28
412
揚げ足とられちゃった。グスン
414ヤリマンZ:03/07/27 00:56
f(x)g(x)を微分するとどーなるかしらねー奴はいねーだろ! 
しかし今ここですぐやれぼけと言われるとこの板の中にいる低脳どものうち 
何人できることやら
415132人目の素数さん:03/07/27 00:56
★☆★☆★☆★☆★☆★☆★☆★☆★☆★☆★☆★☆★
☆★☆★☆★☆★☆★☆★☆★☆★☆★☆★☆★☆★☆
★☆                       
☆★ 送料激安!!  送料激安!!  送料激安!!
★☆      http://www.get-dvd.com      
☆★  激安アダルトDVDショップ        
★☆    お買い得セール1枚500円〜980円!急げ!
☆★    インターネット初!「きたぐに割引」  
★☆    北海道・東北の皆様は送料も激安!   
☆★      http://www.get-dvd.com      
★☆        スピード発送!        
☆★      http://www.get-dvd.com      
★☆        商品が豊富!         
☆★      http://www.get-dvd.com      
★☆                       
☆★☆★☆★☆★☆★☆★☆★☆★☆★☆★☆★☆★☆
416やりまんZ:03/07/27 01:02
f(x)g(x)を微分するとどーなるかしらねー奴はいねーだろ! 
しかし今ここですぐやれぼけと言われるとこの板の中にいる低脳どものうち 
何人できることやら


つーか、普通出来るでしょ。計算手法は、よくある手だし。
418やまりんZ:03/07/27 01:20
f(x)g(x)を微分するとどーなるかしらねー奴はいねーだろ! 
しかし今ここですぐやれぼけと言われるとこの板の中にいる低脳どものうち 
何人できることやら
419132人目の素数さん:03/07/27 01:28
 lim(Δx→0)f(x+Δx)g(x+Δx)−f(x)g(x)/Δx
=lim(Δx→0)g(x+Δx){f(x+Δx)-f(x)}/Δx−f(x){g(x)−g(x+Δx)}/Δx
=公式
420やりまんz:03/07/27 01:47
>>419だから教科書みんなって
 >>417お前まじ頭悪いよ藁
分からない時は分からない問題スレで素直に
「分かりません教えてください」って言うのが一番早く解決に至ると思うよ。
422132人目の素数さん:03/07/27 05:29
槍まんひどい
これでもちゃんと考えたのに・・・
各桁の数が等しい2桁以上の自然数(33333333など)の中に平方数がないことを証明せよ。
>>423
解けたけどあんまりエレガントに解けなかった。

下二桁が44のときしか平方数にならないことを示してから、
44...4 = 2^2 * 11...1 で、11...1が平方数じゃないから
44...4 平方数にならないって証明なんだけど、
もっとキレイにできそうな気がするね。
y=ax^2+bx+c を解け

>>367 ふつうxの値を求めんだろ
426132人目の素数さん:03/07/28 04:06
>>425
あまい! それはエゴだよ
427_:03/07/28 04:07
>424
どうやって解くんですか?
>>424
平方数の下1桁は1、4、5、6、9、0。
0、5、6は明らかにダメ。
残る11...1、44...4、99...9はいずれも11...1が
平方数になることが必要充分。

平方数の下1桁が1になるためには、元の
数の1桁目が1か9であることが必要だが、
11、19、21、29、…、91、99を2乗すると
いずれも十の位が1にならない。
430429:03/07/28 09:31
失敬、>>428だった。
431423:03/07/29 05:17
(略)
111...1=x^2 とする。このときxは奇数
111...0=x^2-1
111...0=(x+1)(x-1)
左辺は2を因数に1つ、右辺は2を因数に2つ以上持つ
よって矛盾
432132人目の素数さん:03/07/29 23:50
世界の歴史を1000字程度で書け
4421:03/07/30 12:19
おめぇ何がしたいんじゃヴォケェ
443132人目の素数さん:03/07/31 17:48
(1) x,y を0以上の整数として 2x+3y≦(2^x)(3^y) を示せ。
(2) 集合Aは以下の条件をみたしている。
  ・ 自然数の要素からなる
  ・ 要素の和は2004である
このような集合はいくつかあるが、
そのうち要素の積が最大のものを求めよ。
444132人目の素数さん:03/07/31 23:01
>>443むずいな…
集合の要素は、重複が許されないからなあ。
>>443
(1)と(2)関係ないじゃん
難易度もダンチだし。
447132人目の素数さん:03/07/31 23:43
実はあるんじゃないの?関係。
俺は予想すらできない・・・
448132人目の素数さん:03/07/31 23:44
>>445
ごめん。重複ゆるすでおねがい。
>>443
>>445を失念し、(1)を(2)の誘導のつもりで書いたとみた。
450132人目の素数さん:03/07/31 23:44
448=443です。
451_:03/07/31 23:44
452_:03/07/31 23:51
自然数からなる有限列{a_k}(k=1,n)が
Σ[k=1,n]a_k=2004
を満たすとき
Π[k=1,n]a_kを最大にするa_kを求めよ。
ってとこかな。
予想ではn=668, a_k=3(k=1,668)
>>453
できたから出題したんじゃないの?>>453>>443
455443:03/08/01 00:00
>>443>>453のつもりでした。
ちなみに>>453は正解。
2x+3y(x≧0, y≧0) が1以外の自然数をすべてあらわせることを示した上で
(1)と 2^3<3^2 (2+2+2=3+3)とによって示せます。

お詫びをかねて新しい問題を

a,b は 2≦a<b をみたす整数とする。
k=1,2,…,b に対して ka を b で割った商を q[k] とするとき
{q[k]} が等差数列になるための a,b がみたすべき必要十分条件を求めよ。
{a_k}に1が含まれていると、最大にはならないことはすぐ示せる。
a_kが2以上の場合、(1)を使って、a_kを2と3だけの和に分解すると、
積はa_k以上になることがわかるので、積が最大になる時は少なくとも
{a_k}の全ての要素が2か3になることがわかる。
あとは、2^3<3^2より、全て3の時が積最大。
457443:03/08/01 00:02
>>454
>>453は別の方です。補完してくれてありがとうございました。
     ∧_∧  ∧_∧
ピュ.ー (  ・3・) (  ^^ ) <これからも僕たちを応援して下さいね(^^)。
  =〔~∪ ̄ ̄ ̄∪ ̄ ̄〕
  = ◎――――――◎                      山崎渉&ぼるじょあ
459暇人:03/08/02 03:59
>>443 の問題は面白かったので、出題者の意図ではない
「文面通りの設定」で考えてみました。>>443は次の数列の問題に言い換えることが出来ます:

(1) 0 < a_1 < a_2 < ... < a_k となる自然数列 で a_1 + a_2 + ... + a_k = 2004 を満たすものはいくつあるか。

(2)(1)の自然数列 { a_1 , a_2 , ... , a_k }で a_1 * a_2 * ... * a_k が最大となるものを求めよ。


上の(2)を解くのに次の簡単な補題1、2を準備して後は可能な場合をしらみつぶしにMathematicaで調べて解きました
460暇人:03/08/02 04:06
【 補題1 】

(2)で求める数列の隣りあう項の差は2以下である : a_{j+1} - a_{j} <= 2


<証明>

もし a_{j+1} - a_{j} > 2 となれば、a_{j} , a_{j+1} をそれぞれ a_{j} + 1, a_{j+1} - 1

に置き換えると


a_{j} + 1 < a_{j+1} - 1 で (a_{j} + 1)*( a_{j+1} - 1) > a_{j} * a_{j+1}


となり最大性に矛盾する。■


461暇人:03/08/02 04:07
従って求める数列は


33 , 34 , 35 , 37 , 38 , 39 , 40 , 41 , 43 , 45 , 46 , ...


のようなものとなるが、この例のように隣りあう項の差が1の部分の塊


33 , 34 , 35

37 , 38 , 39 , 40 , 41

43

45 , 46 , ...


を以下「ブロック」と呼ぶことにする。
462暇人:03/08/02 04:08
【 補題2 】


(2)で求める数列においてブロックの数は2以下である。


<証明>


(2)で求める数列で


a , a+1 , ... , b-3 , b-2 ,

b , b+1 , ... , c-3 , c-2 ,

c , c+1 , ..


とブロックが3以上あるとする。このとき


a , a+1 , ... , b-3 ,

b-1 , b , b+1 , ... , c-1 , c-2 , c-1,

c+1 , ..


と変形すれば (b-1)(c-1) > (b-2)c なので最大性に矛盾する。■
463暇人:03/08/02 04:10
従ってブロックの数が2である数列でその和が2004であるもののうちで、その積が最大となるものが
求める数列となる。

ブロックの数が2である数列の
最初のブロックの初項を n , 最初と2番目のブロックに含まれている項の数をそれぞれ k , l (エル)
とする。したがって数列は


n , n+1 , ... , n+k-1 ,

n+k+1, ... , n+k+l


という形となる。この和が2004という条件は


(2n + k - 1)k / 2 + ( 2n + 2k + l + 1)l / 2 = 2004


すなわち


2(k + l)n + (k + l)^2l + l-k = 2004 * 2    ・・・・・・・・・(※)


となる。あとはこの不定方程式の自然数解 n , k , l を求めて、得られる数列の中で
積が最大に成るのを求める。

以下はそれを実行するための Mathematica のプログラム:
464暇人:03/08/02 04:15
year=2004;sol={};l=1;

While[2(year-l)/(l+1)-l>0,

dl=Divisors[2(year-l)];j=1;

While[j<=Length[dl],

m=dl[[j]];s=2(year-l)/m+1-m;

If[m>l&& s>0 && EvenQ[s],

n=s/2;k=m-l;

sol=Append[sol,Flatten [{Table[n+i,{i,0,k-1}],Table[n+k+i,{i,1,l}]}]];

];

j++;

];

l++

];

t=Table[Product[sol[[i,j]],{j,1,Length[sol[[i]]]}],{i,1,Length[sol]}];mx=Max[t];sol[[Position[t,mx][[1,1]]]]
465暇人:03/08/02 04:16
上のプログラムの出力は

{2,3,4,5,6,7,8,9,10,12,13,14,15,16,17,18,19,20,21,22,23,24,25,26,27,28,29,30,
31,32,33,34,35,36,37,38,39,40,41,42,43,44,45,46,47,48,49,50,51,52,53,54,55,
56,57,58,59,60,61,62,63}


となりこれが(2)の答え( n = 2 , k = 9 , l = 52 )。ちなみに候補となる数列の数、すなわち不定方程式(※)の解の個数は
Length[sol] より 58 となる。


(1)については以下の Mathematica のプログラムで答えが出るはずだが丸1日動かしても
答えは出ませんでした:


F[1,n_]:=1;

F[k_,n_]:=Sum[F[k-1,n-k*j],{j,1,Floor[(2*n-k(k-1))/(2*k)]}];

n=2004;k=1;s=0;While[k*(k-1)<2*n,s =s+F[k,n];k++];s


(1)、(2)ともに Mathematica に頼らずにエレガントに解答はできないものかなぁ・・・
466暇人:03/08/02 04:54
誤植の訂正
>>462
a , a+1 , ... , b-3 ,
b-1 , b , b+1 , ... , c-1 , c-2 , c-1,
c+1 , ..

a , a+1 , ... , b-3 ,
b-1 , b , b+1 , ... , c-3 , c-2 , c-1,
c+1 , ..
の間違い。また、 で(※)は
2(k + l)n + (k + l)^2 + l-k = 2004 * 2    ・・・・・・・・・(※)
の間違いです。
「l」 と「1」が見にくいなぁ
467132人目の素数さん:03/08/02 20:16
\__  _______
     |/        ,,,,,,, _
             /''''  '';::.
  /二⌒"''ヽ    l ≡   );;;:   / ̄ ̄ ̄ ̄ ̄ ̄ ̄
  〈i   `'ヾ |    ≧〒≦  :;/)  | ツマラン!!
  |こi .iこ ヾl    iー/ i ー'  k.l <  おまいの話は
  l / !.ヽヽ i6.    l ノ‐ヘ   iJ   |   ほんとツマラン!!
.  l,〈+ヽ ノ     U乞 し ノ     \_______
   ヽー '/      `ー ‐
468132人目の素数さん:03/08/03 23:58
a,b,c が三角形の3辺となるとき、次の不等式を証明せよ。

   2(a+b+c)(a^2+b^2+c^2) ≧ (b+c-a)(c+a-b)(a+b-c) + 8abc
469トップエリート街道さん ◆BIG6e4aEMg :03/08/04 00:59
a,b,cが実数でa+b+c≧0なら、
(a+b+c)(a^2+b^2+c^2) ≧ (b+c-a)(c+a-b)(a+b-c) + 8abc
が成り立ってますね。
470468:03/08/04 02:41
まず最初に証明すべき不等式が
   (a+b+c)(a^2+b^2+c^2) ≧ (b+c-a)(c+a-b)(a+b-c) + 8abc
であったことは自分のミスです。申し訳ないです。
本問は公式集に載ってなさそうなマニアックな公式
   (a+b+c)(-a+b+c)(a-b+c) + (a+b+c)(a-b+c)(a+b-c) + (a+b+c)(a+b-c)(-a+b+c) - (-a+b+c)(a-b+c)(a+b-c) = 8abc
の左辺の第4項を右辺に移項して残りの項の (a+b+c) 以外を相加相乗で評価しました。
(ここでいろいろなポカミスをやってしまいましたが)
このとき各因数が正であるように三角形の3辺という条件をつけました。

>>469
問題を精錬してくれてどうもありがとうございました。
a+b+c=0なら0≧0でOK

a+b+c>0ならa+b+c=1としてよい。この条件下で
  (a+b+c)(a^2+b^2+c^2) ≧ (b+c-a)(c+a-b)(a+b-c) + 8abc
⇔a^2+b^2+c^2 ≧ (1-2a)(1-2b)(1-2c) + 8abc
⇔a^2+b^2+c^2 ≧ 4(ab+ba+ca) - 1
⇔a^2+b^2+c^2 ≧ 2(1-(a^2+b^2+c^2)) - 1
⇔a^2+b^2+c^2 ≧ 1/3
よって不等式は正しい。
472これはどう?:03/08/04 03:15
エロサイトからてきとうにみつけてきた問題。

x^2+y^2≦(1-z)^2、0≦z≦1の直円錐と平面A y+z=1/2がある。
この円錐の平面Aより上側の部分の体積をVとするとき、Vを求めよ。
>>465
一般に自然数N≧2についてm=[(√(8N+17)+1)/2]とおくとき
納x∈S]x=Nを満足する中でのΠ[x∈S]xを最大にする自然数の集合Sは
8N+17が平方数でないとき{2,3,4,・・・,[m]}\{m(m+1)/2-N-1}
8N+17が平方数のとき{3,4,・・・,[m]-1,[m]+1}
になるようだ。
あ、N=4は例外。
あ、8N+17が平方数のとき{3,4,・・・,[m]-2,[m]}
>>472
ワロタ。どんなエロサイトだよ。
477132人目の素数さん:03/08/05 20:19
  _, ._
( ゚ Д゚)
478暇人:03/08/06 23:05
>>464のプログラムはブロックが2つの場合しか検証してなく、
ブロックが1の場合を検証してないから不正確でした.
でも最終的な答えはあってるようです.
以下ブロックが1の場合を考察したプログラムです:

year = 2004; sol = {}; l = 1;
While[2(year - l)/(l + 1) - l > 0, dl = Divisors[2(year - l)]; j = 1;
While[j <= Length[dl], m = dl[[j]]; s = 2(year - l)/m + 1 - m;
If[m > l && s > 0 && EvenQ[s], n = s/2; k = m - l;
sol =
Append[sol,
Flatten[{Table[n + i, {i, 0, k - 1}],
Table[n + k + i, {i, 1, l}]}]];];
j++;];
l++];
k = 1;
While[2*year - k(k - 1) > 0,
n = (2*year - k(k - 1))/(2*k);
If[IntegerQ[n], sol = Append[sol, Table[n + i - 1, {i, 1, k}]];];
k++;
];
t = Table[
Product[sol[[i, j]], {j, 1, Length[sol[[i]]]}], {i, 1, Length[sol]}];
mx = Max[t];
sol[[Position[t, mx][[1, 1]]]]


479暇人:03/08/06 23:15
>>473
上記のプログラムでyearの部分を1000000まで変えて検証したら
確かにそうなりますね.どうやって見つけたんですか?感心します!
証明を書かれてなかったんで、自分なりに証明を考えてみました.
本当は暇じゃないんですけど・・・

【 補題3 】
Nを自然数とする. 自然数列 0 < a_1 < ... < a_k で a_1 + ... + a_k = N
のとき a_1 * ... * a_k が最大となるのは N=1,4 を除いて
a_1 = 2 または a_1 = 3  のときだけである.

<証明>
● a_1 = 2n+1 (n=2,3,...) の場合:
a_1 の代わりに n,n+1 と置き換えれば n(n+1) > 2n+1
なので積の最大性に矛盾する.
● a_1 = 2n (n=3,...) の場合:
a_1 の代わりに n-1,n+1 と置き換えれば (n-1)(n+1) > 2n
なので積の最大性に矛盾する.
● a_1 = 1 の場合:
Nは1でないのでa_kは1でない.この時
1 , a_2 , ... , a_{k-1} , a_k
を a_2 , ... , a_{k-1} , a_k + 1 と置き換えれば a_k + 1 > 1*a_k
なので積の最大性に矛盾する.
● a_1 = 4 の場合:
(i) 4,5,... の時:
4,5 の代わりに 2,3,4 と置き換えれば 4*5< 2*3*4 なので積の最大性に矛盾する.
(ii) 4,6,... の時:
4,6 の代わりに 2,3,5 と置き換えれば 4*6< 2*3*5 なので積の最大性に矛盾する.■
480暇人:03/08/06 23:17
>>460の補題1、>>462の補題2、と>>479の補題3より、もとめる数列の形は集合の形で表すと
A){ 2 , 3, ... ,2+k+j } \ { 2+k } (k>0, j ≧ 0 )
B){ 3 , 4, ... , 3+k+j } \ { 3+k } (k>0, j ≧ 0 )
となる.(j=0の場合はブロックが1つの場合を表す)
ここでA)B)のそれぞれのタイプについて集合の要素の和がNとなるものが存在するための必要十分条件を求める.

A)の場合: 
要素の和がNとなる条件は (3+k)k/2+(2k+5+j)j/2=N ⇔ (k+j)^2 + 3(k+j) + 2j =2N となる. 
k+j=m とおくと j = -m(m+3)/2 + N , k = m(m+5)/2 - N となる. 
これより j ≧ 0 ⇔ m(m+3) ≦ 2N ⇔ m ≦ (-3+√(8N+9))/2
であり、また k > 0 ⇔ m(m+5) > 2N ⇔ (-5+√(8N+25))/2 < m 
となるから j ≧ 0 , k > 0 となるための条件は 
α = (-3+√(8N+9))/2 , β = (-5+√(8N+25))/2 とおくと
β < m ≦ α ・・・・・(1)
となる. 以上から『A)のタイプの集合で要素の和がNとなるものが存在する』⇔『(1)をみたす整数が存在する』

B)の場合: 
要素の和がNとなる条件は (5+k)k/2+(2k+7+j)j/2=N ⇔ (k+j)^2 + 5(k+j) + 2j =2N となる.
 k+j=m とおくと j = -m(m+5)/2 + N , k = m(m+7)/2 - N となる.
これより j≧0 ⇔ m(m+5)≦ 2N ⇔ m ≦(-5+√(8N+25))/2
であり、また k > 0 ⇔ m(m+7) > 2N ⇔ (-7+√(8N+49))/2 < m
となるから j ≧ 0 , k > 0 となるための条件は γ= (-7+√(8N+49))/2 とおくと
γ < m ≦ β ・・・・・(2)
となる. 以上から『B)のタイプの集合で要素の和がNとなるものが存在する』⇔『(2)をみたす整数が存在する』

481暇人:03/08/06 23:19
以下 m~ = [α] , k~ = m~(m~+5)/2 - N とおく.
明らかに α-1 < m~ ≦α が成り立ち、m~-2 < γ < m~ , m~-1 < β が
成り立つことが容易にわかる. 以下 β と m~ の大小、及びγ と m~-1 の
大小に応じて(1)、(2)の不等式の自然数解の存在を考察しする.

●β < m~ ⇔ m~(m~+5) > 2N の場合:

(i) γ < m~-1 の時には m~-2 < γ < m~-1 <β < m~ ≦ α となるから
m~ , m' = m~-1 が(1),(2)のただ一つの自然数解. この時A)タイプの集合は 

{ 2 , 3 , ... , m~+2 } \{ k~+2 } 

となりその積は(m~+2)!/(k~+2)となる. B)タイプの集合は
k' = m'(m'+7)/2 - N とおくと, k' = (m~ - 1)(m~ + 6)/2 - N = k~ - 3 より

{ 3 , 4 , ... , m'+3 } \{ k' + 3 } = { 3 , 4 , ... , m~+2 } \{ k~ } 

となる.その積は (m~+2)!/(2k~) となる. k~ ≧ 0 より 2k~ > k~+2 となり
 { 2 , 3 , ... , m~+2 } \{ k~+2 } が積を最大にする.

(ii) m~-1 ≦ γ の時には  m~-1 ≦ γ <β < m~ ≦ α となるから
m~ が(1)のただ一つの自然数解で(2)は自然数解をもたない.
従って{ 2 , 3 , ... , m~+2 } \{ k~+2 } が積を最大にすることになる.
以下のように、(十進表記で)1桁の整数列 {a_n} (n=1,2, ...) を定めます:
(i) a_1=2, a_2=3
(ii) a_1*a_2=6 なので、a_3=6
(iii) a_2*a_3=18 なので、a_4=1, a_5=8
(iv) a_3*a_4=6 なので、a_6=6
(v) a_4*a_5=8 なので、a_7=8
...
このとき、{a_n} に 5, 7, 9 が現れないことを示して下さい。
483暇人:03/08/06 23:20
● m~ ≦ β ⇔ m~(m~+5) ≦ 2N の場合:

(i) γ ≦ m~-1 の時には γ ≦ m~-1< m~ ≦ β となるが、β - γ< 1 なので矛盾する.
よってこの場合は起こりえない.

(ii) m~-1 < γ の時には m~-1 < γ < m~ ≦ β < α <m~+1 
となるから m~ が(2)のただ一つの自然数解で(1)は自然数解をもたない.
この時B)タイプの集合は m~(m~+7)/2-N = k~+m~ より 

{ 3 , 4 , ... , m~+3 } \{ k~ + m~ + 3 } 

となる. 以下 k~=0 または k~=-1 を示す. k~ = m~(m~+5)/2 - N より明らかに k~ ≦ 0 となる.
また m~ > α-1 = (-5+√(8N+9))/2 より k~ = m~(m~+5)/2 - N > (α-1)(α+4)/2 -N = -2 
となり -2 < k~ ≦ 0 をえる. したがって求めるB)タイプの集合は
k~=0 の場合:{ 3 , 4 , ... , m~+2 } で k~=-1 の場合:{ 3 , 4 , ... , m~+3 } \{ m~ + 2 }
となる.
484暇人:03/08/06 23:22
以上のことからNを2以上の4でない自然数とする時、
要素の和がNとなる自然数の集合のうち要素の積が最大となるのは 
m~ = [(-3+√(8N+9))/2] , k~ = m~(m~+5)/2 - N とおいた時、

イ)m~(m~+5) > 2N の場合:{ 2 , 3 , ... , m~+2 } \{ k~+2 }
ロ)m~(m~+5) ≦ 2N かつ k~=0 の場合:{ 3 , 4 , ... , m~+2 }
ハ)m~(m~+5) ≦ 2N かつ k~=-1 の場合:{ 3 , 4 , ... , m~+3 } \{ m~ + 2 }

となる. ところで イ)で形式的に k~=0 とすると ロ)になるので

ハ)m~(m~+5) ≦ 2N かつ k~=-1 の場合:{ 3 , 4 , ... , m~+3 } \{ m~ + 2 }
ハ)以外:{ 2 , 3 , ... , m~+2 } \{ k~+2 }

とまとめることもできる. 実は『ハ) ⇔ 8N+17 が平方数』となる.

ハ) ⇒ 8N+17 が平方数:
k~ = m~(m~+5)/2-N = -1 であったから N = m~(m~+5)/2+1 となる. 従って
8N+17 = (2m~+5)^2 となる.

8N+17 が平方数 ⇒ ハ):
8N+17 は奇数なので 8N+17 = (2t+1)^2 とおける. この時 N = t(t+1)/2-2 となり
容易に m~ = t-2 となることがわかる. この時
m~(m~+5) = t^2+t-6 , 2N = t^2+t-4 なので m~(m~+5) < 2N が成り立ち、また
k~ = (t-2)(t+3)/2 - t(t+1)/2+ 2 = -1 となる.
485132人目の素数さん:03/08/07 00:26
>>482
問題に具体例しかないので以下のように表記を変えた。
あと勝手に少し改題した。(0が現れないことを付け加えた)
[問]
数列 {a_n} を以下で定める。
a_1=2, a_2=3
n≧3 に対して十進法表記で a_(n-1)*a_(n-2) = a_(n)a_(n+1)…
このとき数列 {a_n} に 0, 5, 7, 9 が現れないことを示せ。

次レスに解答を載せるが、なんかもっと巧い方法があるのだろうか。
486132人目の素数さん:03/08/07 00:26
以下 n≧3 で考える。

【9が現れないこと】
 a_(n-1)*a_(n-2) ≦ 9*9 = 72 < 90
よって9が現れるならば、1の位に現れるしかない。
ところが a_1 が偶数で、a_(n-1)*a_(n-2) は高々2桁の整数だから  … (1)
a_n, a_(n-1) の少なくとも一方は偶数である。  … (2)
したがって数列 {a_n} に9は現れない。

【7が現れないこと】
数列 {a_n} に9は現れないから
 a_(n-1)*a_(n-2) ≦ 8*8 = 64 < 70
したがって7が現れるならば、1の位に現れるしかない。
1の位に7が現れるような a_(n-1), a_(n-2) の組み合わせは
(1,7),(7,1) の2通りしか考えられないが、(2)によりこれはありえない。
したがって数列 {a_n} に7は現れない。

【5が現れないこと】
(1)により数列 {a_n} に5が現れるとするならば a_(n-1), a_(n-2) の組み合わせは
(奇数,5),(5,奇数),(6,9),(9,6),(7,8),(8,7) のみであるが、
最初の2つは(2)により、残りの4つは数列 {a_n} に7,9が現れないことによりいずれもありえない。
したがって数列 {a_n} に5は現れない。

【0が現れないこと】
(1)と、最高位に0は現れないことにより
数列 {a_n} に0が現れるとするならば a_(n-1), a_(n-2) の組み合わせは
(偶数,5),(5,偶数) のみである。
これは数列 {a_n} に5は現れないことに矛盾。
したがって数列 {a_n} に0は現れない。
487_:03/08/07 00:27
488_:03/08/07 00:29
489132人目の素数さん:03/08/07 01:26
>>486の訂正
【7が現れないこと】
(1,7),(7,1) の2通りしか考えられないが、(2)によりこれはありえない。

  ↓

(1,7),(7,1),(3,9),(9,3) しか考えられないが、
前2つは(2)より、後2つは数列 {a_n} に9が現れないことより、これはありえない。
490132人目の素数さん:03/08/07 01:40
もう1つあった。
【9が現れないこと】4行目
a_(n-1), a_(n-2) の少なくとも一方は偶数である。  … (2)

まあさっきほど致命的ではないが。一応。
491暇人:03/08/07 01:43
>>482 なかなか傑作な問題ですね。背理法で示します。a_m を5,7,9 の中で最初に出現した数とする。
a_1, ... , a_{m-1} までは1,2,3,4,6,8のいずれかであるので下の表

 |1  2  3  4  6  8
============
1|1  2  3  4  6  8
2|2  4  6  8 12 16
3|3  6  9 12 18 24
4|4  8 12 16 24 32
6|6 12 18 24 36 48
8|8 16 24 32 48 64

より a_m = 9 で a_k=3 , a_{k+1}=3 となる番号 k < m が存在する。
この時
『任意の自然数 n に対して a_1 , ... , a_{k-1}の中に
3 , 1 , 1 , ... , 1 , 3 ( 1 が n 回続く )となる部分列が存在する』(※)
が示され、n > k と取れば矛盾する。

492暇人:03/08/07 01:44
(※)の証明:
n に関する帰納法で示す。
表の二桁の数字はすべて偶数であることより
a_1 , ... , a_{k-1}の部分列{ a , b , c }で
ab = 3 , bc = 3 となるものが存在する。可能な場合は (a,b,c) = (1,3,1) , (3,1,3) である。
もし(a,b,c) = (1,3,1) とすると、同様に表の二桁の数字はすべて偶数であることより
a_1 , ... , a_{k-1}の部分列 { e , f , g , h} で
ef = 1 , fg = 3 , gh = 1 となるものが存在することになるがこれを満たす自然数解は存在しない。
したがって(a,b,c) = (3,1,3) である。これで(※)の n = 1 の場合が示された。
n の場合に(※)が成立するとする。
このとき a_1 , ... , a_{k-1}の部分列{ b_1 , ... , b_{n+3} }で
b_1*b_2 = 3 , b_2*b_3 = 1, ... , b_{n+1}*b_{n+2} = 1 , b_{n+2}*b_{n+3} = 3 ・・・・(1)
となるものが存在する。b_1 = 1 とすると b_2 = 3 b_2*b_3 = 1 となり矛盾。
よってb_1 = 1 で以下関係式(1)より b_2 = ・・・ = b_{n+2} = 1 , b_{n+3} = 3
となり n+1 の場合に(※)が成立することが示された。

493暇人:03/08/07 01:52
細かいことだけど a_1 , ... , a_{k-1}の「部分列」といってるのは正確には 
a_1 , ... , a_{k-1}の隣接する項からなる部分列と言うことです。
494暇人:03/08/07 02:59
>>485
>数列 {a_n} を以下で定める。
>a_1=2, a_2=3
>n≧3 に対して十進法表記で a_(n-1)*a_(n-2) = a_(n)a_(n+1)…
これでは数列が定義できませんよ。
n=3 で漸化式を使うと a_2*a_1 = a_3a_4 (十進法表記) で
a_2*a_1=3*2= 6よりa_3=0,a_4=6となりますが、
n=4 で漸化式を使うと a_3*a_2 = a_4a_5 (十進法表記) で
a_3*a_2= 0* 3=0 よりa_4=0,a_5=0となり、矛盾がおきます。
>>482の数列は
2,3,6,1,8,6,8,4,8,4,8,3,2,...
となりますが a_1*a_2,a_2*a_3,a_3*a_4,...を使って、「かなり先のほう」を
決定していきますので簡単なかたちで表現するのは難しいですね。
仮に>>485の数列に対する証明としてみたばあい
>>486の【9が現れないこと】の証明で(1)から(2)はすぐにいえないのでは?
19,39,59,79が現れないのは自明ではないはず。
じゃあ訂正。

a_n は次のように帰納的に定義される。
(イ) a_1=2, a_2=3
(ロ) a_(n), a_(n+1) が定義されていて a_(n+2) が定義されていないとき
   (i) a_(n)*a_(n+1) が1桁のときは a_(n)*a_(n+1) = a_(n+2)
   (ii) a_(n)*a_(n+1) が2桁以上のときは十進法表記で a_(n)*a_(n+1) = a_(n+2)a_(n+3)…
     すなわち、各桁の数を最高位から順に a_(n+2), a_(n+3), … と1の位まで定義する。

こうすれば>>482の意味になっているかな。


>仮に>>485の数列に対する証明としてみたばあい
数列に項が重複して現れるのでそれは考えないでくれ。
>>486の解答は>>482の意味でやってるから。
nは自然数ね。一応。
ごめん違うね。やっぱ簡単に表わすのは難しいわ。

498暇人:03/08/07 05:20
>>497
人間が読めば>>482の数列の構成法は理解できるけど、これをコンピュータの
プログラムに乗せるには、きちんとした帰納的な定義が必要。
本来の問題を解くのに全然役に立たないけど、一応アルゴリズムを正確に表現する
練習問題として一応書いておくことにします。

k≧1として(k-1)回目の操作を行った時点で定義された数列の項の数をb_kで表します。
k=1では初期状態で
a_1 = 2 , a_2 = 3 , b_1 = 2 ・・・・・・(1)
(k-1)回目の操作後には
a_1 , a_2 , ・・・ , a_{b_k}
まで定義されてますので k 回目の操作では a_k と a_(k+1)を用いて a_{b_k} 以降を
定義することになります。
i)a_k * a_(k+1) < 10 の場合:
b_(k+1) = b_k + 1
a_(b_k + 1) = a_k * a_(k+1)
により b_(k+1) , a_(b_k + 1) を定義します。
ii)10 ≦ a_k * a_(k+1) < 100 の場合:
b_(k+1) = b_k + 2
a_k * a_(k+1) = 10*a_(b_k + 1) + a_(b_k + 2)
1 ≦ a_(b_k + 1) <10 , 0 ≦ a_(b_k + 2) < 10
により b_(k+1) , a_(b_k + 1) , a_(b_k + 2) を定義します。
499132人目の素数さん:03/08/07 05:27
☆美女の○○○を見放題(^^)v!☆
http://endou.kir.jp/yuminet/link.html
http://endou.kir.jp/marimo/link.html
(σ・∀・)σゲッツ!! 500
501132人目の素数さん:03/08/07 10:56
任意の無理数の任意の小数第 n (n≧1)位 に 数字 k (0≦k ≦9)
が現れる確率P(n,k)は,n,kに関わらず1/10ではないことを示せ.
502132人目の素数さん:03/08/07 15:07
    ヽ(`ヽ(`Дヽ(`Д´)`Д´)ノД´)ノ)ノ
  .  ヽ(ヽ(`Дヽ(`Д´)`Д´)ノД´)ノノ
     ヽ(ヽ(`Дヽ(`Д´)`Д´)ノД´)ノノ
     ヽヽ(`Дヽ(`Д´)`Д´)ノД´)ノ
      ヽ(`Дヽ(`Д´)`Д´)Д´)ノ
      ヽ(`Дヽ(`Д´)`Д´)Д´)ノ
        ヽ(`Дヽ(`Д´)`Д´) ´)ノ
         ヽ(ヽ(`Д´)`Д´)´)ノ
          ヽヽ(`Д´)`Д´)´)ノノ
         ヽヽ(`Д´)`Д´)´)ノノ
         ヽ(`Д´)`Д´)´)ノ
         ヽ(`Д´)`Д´)ノ
       ヽ(ヽ(`Д´)´)ノ
      . ヽヽ(`Д´)´ノノ
      ヽ(ヽ(`Д´)ノ
       ヽ(`Д´)ノノ
     ヽ(`Д´)ノ
      ( )
503482:03/08/07 21:57
皆さんすばらしいですね。私が用意した解答は>>486さんとほとんど同じです。
数列をもう1問(こちらの方が簡単だと思いますが、1番あたりでどうか
と言うことで)。

数列 {p_n} (n=1,2,...) を次のように定義します:
p_1=2, p_{n+1} は 1+p_1*p_2*...*p_n の最大の素因数。
このとき、5 はこの数列に現れないことを示してください。
>>503
でけた。
p_1=2、p_2=3なので3以上のnに対し1+p_1*p_2*...*p_nは2,3と互いに素。
もし5が最大素因子なら1+p_1*p_2*...*p_n=5^kとなりよって
p_1*p_2*...*p_n=5^k-1であるが左辺はちょうど1回でしか2でわれないが右辺は4の倍数となり
矛盾である。よって1+p_1*p_2*...*p_nの最大素因子は5ではありえない。
nを任意の自然数、x_1, x_2, ..., x_n > 0 とするとき、
(x_1 * x_2 * ... * x_n)^(1/n) < (x_1 + x_2 + ... + x_n)/n
を証明せよ。

コーシーシュワルツの不等式の証明よりはムズイと思う。
506暇人:03/08/08 02:39
>>505 < は ≦ です。相加平均≧相乗平均 という超有名な不等式ですね。
おそらく100通りぐらいの証明があるでしょう。
507505:03/08/08 02:45
>>506
そうそう。 <= でした。
4通りくらいなら証明の仕方知ってるけど
知らないとなかなか難しいんじゃないかと思うな。

高校のときは証明せずに使ってたように思うのだが、
どうだったかなぁ……。
n=2,3のとき限定で使ってたのかなぁ。
508現役付属中U:03/08/08 03:42
いつもコソーリROMってます。
暇人さんはマジすごいですね。

暇人さんに質問ですが、今年の数学オリンピックの問題って
ご覧になりましたでしょうか?

もし見たのであれば、どのくらい出来るのか正直に教えてください。


509暇人:03/08/08 05:12
>>508

>暇人さんはマジすごいですね。

中学生の方ですかぁ。年の功というか、君らからみればもう相当オジンなわけですし、
それにいちおう昔は数学好き少年でしたんで・・・
普段ルーチーンなことしかしてないんで、たまたま覗いたこのスレで久々に数学の
問題で頭を活性化させてもらってます。

そういえば今年は日本で数オリ開催だったんですね。ふだんテレビとか新聞
あんまし見ないんで忘れてました。おそらく報道してたとしても地味な扱いじ
ゃなかったのかな。

>暇人さんに質問ですが、今年の数学オリンピックの問題って
>ご覧になりましたでしょうか?

というわけなんで見てないんですが、問題って公表されてるんですか
年重ねると、頭さび付いて自由な発想ができなくなるので、数オリで
出題されるような問題はあんまり解けないんじゃないかな。
君らのような若い人がこれからがんばってください。

明日〆切りの書類を書かなきゃならないんで、本当は暇じゃないはず
なんですけどね・・・
510暇人:03/08/08 05:45
>>505 そういえば相加平均≧相乗平均の行列バージョンがあります。
対称な二次の正方行列 A={{a,b},{b,d}}({a,b}が第一行、{b,d}が第二行となる行列です)
が a>0 , det(A) = ad-b^2>0 を満たすとき正定値と呼びます。
A , B が対称な二次の正方行列で正定値の場合、

det((A+B)/2) ≧ √(det(AB))

が成り立つことを示せ。
(実は一般の n 次の正値対称行列についても成り立ちます)
511132人目の素数さん:03/08/08 19:24
>>510
でけた。結構しんどかった。(しかしこれでは一般の n で証明できない・・・)

A={{a,b},{b,c}} , B={{x,y},{y,z}} , a>0, x>0, ac-b^2>0, xz-y^2>0 とする。
det((A+B)/2) = (1/2)det(A+B) は成分計算をすれば分かる。
そこでまず det(A+B) > 0 を示すことにする。

det(A+B) = (a+x)(c+z) - (b+y)^2
   = (ac-b^2) + (xz-y^2) + ax+cz-2by
   > ax+cz-2by
   ≧ 2(√(axcz)-by) (∵ac>b^2≧0, xz>y^2≧0 より相加相乗平均の不等式が成り立つ)
   ≧ 2(√((b^2)(y^2)) - by)
   = 2(|by|-by)
   ≧ 0

つぎに det(AB) > 0 を示す。

det(AB) = (ax+by)(by+cz) - (ay+bz)(bx+cy)
   = (ac-b^2)(xz-y^2)
   > 0

したがって与不等式の両辺を2乗しても同値・・・と思ったが、計算過程をよくよく観察すれば、

det((A+B)/2) = (1/2)det(A+B)
   ≧ ((ac-b^2) + (xz-y^2))/2
   ≧ √((ac-b^2)(xz-y^2))
   = √(det(AB))

であった。等号は ax=cz, by≧0, ac-b^2=xz-y^2 のとき。
どうやら等号成立条件はもう少し簡単に書けそうですね。
512ビッグバン宇宙論は完全に大間違いだった!!!!!!!!:03/08/08 19:25
科学者よ、恥を知れ!
ビッグバン宇宙論は完全に間違いだった!
科学の原則を無視した、デタラメのインチキ理論だったのだ。
そして、そのビッグバン宇宙論の世界的な浸透は
アメリカ、ユダヤ・キリスト教勢力による世界支配のための思想的な戦略なのだ!
また、ビッグバン宇宙論の思想によって戦争が起こり、
貧富の差がひらき、終末的な絶望感が世界に蔓延しているのだ。
ビッグバン宇宙論は世界の平和を揺るがす、悪の元凶となっているのだ。
ビッグバン宇宙論とは、
「宇宙は『無』からビッグバン(大爆発)によって誕生した」という理論である。
この理論は、ユダヤ・キリスト教の創造神話(神が天地を創造した)そのものである。
ビッグバン宇宙論の実態は、科学理論ではなく宗教思想なのである。
『無』は科学的に証明できるものではなく、
そして、『無からの誕生』も科学では証明できるものではないのだ。
ビッグバン宇宙論が科学の正統であるという思想を、世界中の人々に
浸透させる戦略が成功したことにより、ユダヤ・キリスト教勢力の
世界における優位性が確立されていったのだ。(20世紀に)
そして、その思想的支配の最大の例が、アメリカやイギリスによるイラク戦争なのだ。
ビッグバン宇宙論の浸透により、世界中に終末思想(世界の終わり)が蔓延してしまっている。
そのことにより、自己中心的、せつな的、短絡的な考え方が社会に広がっている。
科学的に間違っているビッグバン宇宙論から脱却しなければならない。
そして、宇宙は無限だということを理解しなければならない。
人間は本当の宇宙観、世界観を構築し、
新しい時代に進んでいかなければならないのだ。
ビッグバン宇宙論が世界を支配している限り、平和な世界にはならないのだ。
そのことを科学者は重く受けとめるべきである。
平和の時代へ!!!!!!!!!!!!!!!!!!!!!!!!!!!!!!!!!!!!!!!!!
= (ac-b^2) + (xz-y^2) + ax+cz-2by
   > ax+cz-2by
   ≧ 2(√(axcz)-by) (∵ac>b^2≧0, xz>y^2≧0 より相加相乗平均の不等式が成り立つ)



= (ac-b^2) + (xz-y^2) + az+cx-2by
   > az+cx-2by
   ≧ 2(√(azcx)-by)
          (∵ a>0, x>0, ac>b^2≧0, xz>y^2≧0 より c>0, z>0 だから az>0, cx>0 より相加相乗平均の不等式が成り立つ)

でした。すまぬ。
514132人目の素数さん:03/08/08 20:19
det((A+B)/2) = (1/2)det(A+B)
det((A+B)/2) = (1/2)det(A+B)
det((A+B)/2) = (1/2)det(A+B)
det((A+B)/2) = (1/2)det(A+B)
det((A+B)/2) = (1/2)det(A+B)
det((A+B)/2) = (1/2)det(A+B)
det((A+B)/2) = (1/2)det(A+B)
515132人目の素数さん:03/08/08 22:15
        ∩
 \      ( )                 ∩         /
       ∧_∧| |               (ヨ )∧_∧
     < `Д>|               \<Д´ >
     /⌒   |    \ ボッキアゲニダ/   (  ⌒ヽ
_    < く\ \   \     /    ノ  ノ\\___
     \( ヨ 、 つ              ⊂  )  \   ) ─
       / /ω    ヽ<`Д´>ノ     ω \    ̄
     / / ./       ( ∩ )       ヽ\ \
 /  〈  ̄フ ̄フ.     /ω\      << ̄ 〉  \
516132人目の素数さん:03/08/08 22:21
511 名前:132人目の素数さん :03/08/08 19:24
>>510
でけた。結構しんどかった。(しかしこれでは一般の n で証明できない・・・)

A={{a,b},{b,c}} , B={{x,y},{y,z}} , a>0, x>0, ac-b^2>0, xz-y^2>0 とする。
det((A+B)/2) = (1/2)det(A+B) は成分計算をすれば分かる。

パシャ パシャ  パシャ パシャ パシャ  パシャ パシャ パシャ パシャ パシャ パシャ   
   パシャ パシャ パシャ パシャ パシャ  パシャ パシャ パシャ  パシャ  パシャ パシャ パシャ
 ∧_∧     ∧_∧     ∧_∧  ∧_∧    ∧_∧     ∧_∧    
 (   )】      (   )】    (   )】 【(   )    【(   )    【(   ) < (・∀・)イイヨイイヨー
 /  /┘ .   /  /┘.    /  /┘ └\\    └\\   └\\    スゴク(・∀・) イイ!
ノ ̄ゝ     ノ ̄ゝ      ノ ̄ゝ     ノ ̄ゝ     ノ ̄ゝ     ノ ̄ゝ  
517132人目の素数さん:03/08/08 22:52
ギャハハハハハハ!
det((A+B)/2) = (1/2)det(A+B) は成分計算をすれば分かる。

    /\___/ヽ   ヽ
   /    ::::::::::::::::\ つ
  . |  ,,-‐‐   ‐‐-、 .:::| わ
  |  、_(o)_,:  _(o)_, :::|ぁぁ
.   |    ::<      .::|あぁ
   \  /( [三] )ヽ ::/ああ
   /`ー‐--‐‐―´\ぁあ

det((A+B)/2) = (1/4)det(A+B) だった。
死体処理は適当に任せるので、殺っちゃってくれ。
任意の凸多角形には周囲の長さと面積を同時に2分する直線があることを証明してください。
520132人目の素数さん:03/08/08 23:27
  ∧_∧           ノノノノ  -___
 (  ^^ )←518         (゚∈゚ )  ─_____ ______ ̄
 (     )          丿\ノ⌒\  ____ ___
 | | |          彡/\ /ヽミ __ ___
 (__)_)             ./∨\ノ\  =_
                  .//.\/ヽミ ≡=-
                 ミ丿 -__ ̄___________
 (⌒\ ノノノノ
   \ヽ( ゚∋゚)
    (m   ⌒\
     ノ    / /
     (   ∧ ∧
  ミヘ丿 ∩ ^^ ;)
   (ヽ_ノゝ _ノ

     丿   ,;⌒⌒i.
  ノノノノ⌒ヽ (   ;;;;;)   ______
 (゚∈゚ )  ミ)     ,,:;;;)  |         |
/⌒\/(   ) ヽ| |/ |;,ノ  |  樹海    |
( ミ   ∨∨  | /  .,i  |______|
 ノ  /     | | ,,i; ,, . ,;⌒‖
( \/ヽ ,,,丶, | |,,,;.    ;i,  ‖ヽ
 \ ) ) ..   ,,   ´ヽ (,,  ‖丿.,,,
 ///   ,,   ,,  .. ´ヽ  ‖,,, ..,
`ヾ ヽミ ,,  .、 ヽ .. ヽ丶,.ヽ ‖、,,
>>519
多角形の周上に1点Aを取る。すると周を2等分する反対側の点Bが
一意的に決まる。直線ABで分けられた領域は、片方が1/2以下で
他方が1/2以上である。

点AとBを、「周の長さを2等分」という条件が崩れないように移動させる。
つまりAを時計回りにaだけ動かしたら、Bも時計回りにaだけ動かす。
こうしてAがはじめのBの位置に来るまでの間に、もと1/2以下だった
領域の面積は1/2以上に連続的に変化する。中間値の定理。
522132人目の素数さん:03/08/08 23:53
34分間もの間>>521が↑のような解答を必死で考えてたと思うと涙ぐましいものがある
523132人目の素数さん:03/08/09 00:08
>>522
実は>>521の解法に感動してたんだけど…
なんか不備があるのですか?
522の意図が分からぬ。521の解法を思いつくだけならすぐ出来るだろうし…
(i) F(1,0,0,1)=1
(ii) F(ka,b,kc,d)=k*F(a,b,c,d)
(iii) F(a,b,c,d)=-F(b,a,d,c)
(iv) F(a+e,b,c+f,d)=F(a,b,c,d)+F(e,b,f,d)
を満たす関数F:R^4→Rを全て求めよ。
>>525
スレ違いもいいところだ。
そうかな?なかなか感じいいと思うけど。
528132人目の素数さん:03/08/09 10:03
    ヽ(`ヽ(`Дヽ(`Д´)`Д´)ノД´)ノ)ノ
  .  ヽ(ヽ(`Дヽ(`Д´)`Д´)ノД´)ノノ
     ヽ(ヽ(`Дヽ(`Д´)`Д´)ノД´)ノノ
     ヽヽ(`Дヽ(`Д´)`Д´)ノД´)ノ
      ヽ(`Дヽ(`Д´)`Д´)Д´)ノ
      ヽ(`Дヽ(`Д´)`Д´)Д´)ノ
        ヽ(`Дヽ(`Д´)`Д´) ´)ノ
         ヽ(ヽ(`Д´)`Д´)´)ノ
          ヽヽ(`Д´)`Д´)´)ノノ
         ヽヽ(`Д´)`Д´)´)ノノ
         ヽ(`Д´)`Д´)´)ノ
         ヽ(`Д´)`Д´)ノ
       ヽ(ヽ(`Д´)´)ノ
      . ヽヽ(`Д´)´ノノ
      ヽ(ヽ(`Д´)ノ
       ヽ(`Д´)ノノ
     ヽ(`Д´)ノ
      ( )
529 ◆iicafiaxus :03/08/09 10:05
図が無いと説明しづらいけど

田の字型に並べられた12個の線分を黒と白に塗り分けることを考える。

(1)塗り分け方の総数は何通りあるか。
(2)黒い線分がすべてひとつながりに繋がっているような塗り分け方は何通りあるか。
  ただし黒い線分が一つもない塗り方も認めることにする。
(3)黒い線分も白い線分もそれぞれひとつながりに繋がっているような塗り分け方は
  何通りあるか。黒い線分や白い線分が一つもない塗り方も認めることにする。

なお、回転・裏返しによって重なるものは同一の塗り分け方であるとする。
530132人目の素数さん:03/08/09 10:20
lim[ε→+0]∫[ε,1](xlogx)^n dx
をnの式で表せ。(対数の底はe)
531132人目の素数さん:03/08/09 12:22
>>530
(-1)^n*{n!/(n+1)^(n+1)}
532現役付属中U:03/08/10 02:49
>>509 暇人サンへ…
書類は締め切りに間に合いましたでしょうか?
ご丁寧な返事どうもありがとうございました。

僕は、某付属中に落ちて公立中学に通っている、数学オリンピックの問題
はおろか、このスレの問題すら2割も解けない盆栽2年生です。

でも、クラスの友達には口が裂けても数学が好きなんて、今更言えません。
だって、先月バイク盗んで補導された奴が、いきなり「相加相乗平均の証明って
何通りあるか知っている?」とか眼を輝かせて言い出したら、
友達全員失うのは確定ですからね…

だから、数学板でコソーリ勉強&エロサイトでコソーリオナニー(最近覚えちまった)
が僕の最大の楽しみです。

スレ違いだ黙れって? わかりました。失礼します。
533132人目の素数さん:03/08/10 02:55
>>532
スレ違いだ

黙れ
534現役付属中U:03/08/10 03:05
>>533
お約束、ありがとうございます。
535132人目の素数さん:03/08/10 07:18
    ヽ(`ヽ(`Дヽ(゚∀゚)`Д´)`Д´)ノД´)ノ)ノ
  .  ヽ(ヽ(`Дヽ(`Д´)`Д´)ノД´)ノノ
     ヽ(ヽ(`Дヽ(`Д´)`Д´)ノД´)ノノ
     ヽヽ(`Дヽ(`Д´)`Д´)ノД´)ノ
      ヽ(`Дヽ(`Д´)`Д´)Д´)ノ
      ヽ(`Дヽ(`Д´)`Д´)Д´)ノ
        ヽ(`Дヽ(`Д´)`Д´) ´)ノ
         ヽ(ヽ(`Д´)`Д´)´)ノ
          ヽヽ(`Д´)`Д´)´)ノノ
         ヽヽ(`Д´)`Д´)´)ノノ
         ヽ(`Д´)`Д´)´)ノ
         ヽ(`Д´)`Д´)ノ
       ヽ(ヽ(`Д´)´)ノ
      . ヽヽ(`Д´)´ノノ
      ヽ(ヽ(`Д´)ノ
       ヽ(`Д´)ノノ
     ヽ(`Д´)ノ
      ( )
536132人目の素数さん:03/08/11 13:35
    |┃三               _________
    |┃        /ヘ;;;;;    /
    |┃ ≡      ';=r=‐リ  < 話は全部聞かせて貰ったぞ!
____.|ミ\_____ヽ二/    \
    |┃=___     \     ̄ ̄ ̄ ̄ ̄ ̄ ̄ ̄ ̄
    |┃ ≡   )   人 \ ガラッ
537132人目の素数さん:03/08/11 18:34

 
 
538山崎 渉:03/08/15 18:45
    (⌒V⌒)
   │ ^ ^ │<これからも僕を応援して下さいね(^^)。
  ⊂|    |つ
   (_)(_)                      山崎パン
539132人目の素数さん:03/08/18 21:37
a,bは実数。
(3^a)+(13^b)=17^a
(5^a)+(7^b)=11^b
a<bを示せ。
540132人目の素数さん:03/08/19 20:57
わからんちん
>>539
でけた。
f(x)=17^x-(3^x+13^x)とおく。17^x=tとおくとf=t-(t^(log3/log17+t^(log13/log17))で
こいつの増減表かくとある実数αが存在して
f(x)>0⇔x>α、f(x)=0⇔x=α、f(x)<0⇔x<α
同様にしてg(x)=11^x-(5^x+7^x)とおくとある実数βが存在して
g(x)>0⇔x>β、g(x)=0⇔x=β、g(x)<0⇔x<β
f(1)=1>0より1>α、g(1)=-1より1<β。
a≧bと仮定する。
0=17^a-(3^a+13^b)≧17^a-(3^a+13^a)よりa≦α。
0=11^b-(5^a+7^b)≦11^b-(5^b+13^b)よりb≧β。
以上よりb≧β>1>α≧a。これは仮定に反する。
xy平面上に放物線C:y=x~2をとり、内部に点(0,0)、且つ各頂点がy<x~2に常に存在する正三角形をとった。
そしてこの正三角形の面積をS、Cと正三角形で囲まれる図形で大きい方の面積をTとした。
このときT/Sの最大値を求めよ。
543132人目の素数さん:03/08/29 23:12
座標空間上に長さLの線分ABがある。
ABをxy平面、xz平面、yz平面に正射影した線分の長さをそれぞれ L_1,L_2,L_3 とするとき、
L_1+L_2+L_3 の最大値および最小値をLで表わせ。
544132人目の素数さん:03/09/01 19:02
<543
Z会が好きそうな問題だな。
545132人目の素数さん:03/09/02 19:50
余弦定理を証明しる。
△OABでOA=a↑、OB=b↑とするとAB=(b↑-a↑)  以下小文字はベクトル(↑は省略する)
|b-a|^2=|b|^2-2a・b+|a|^2
a・b=|a||b|cosθだから
AB^2=OA^2+OB^2-2|a||b|cosθ
で良いのでしょうか?それとも内積が余弦定理から出てきてるのでしょうか?
素人高校生の解答でした
>>546
三平方の定理を使って証明するんじゃなかったけ?
548546:03/09/02 20:21
ちょっとググってみたら
内積は余弦定理から定義されたらしい
(ちょっとしか読んでないから勘違いしてるかもしれないけど)
ってことは>>546は×だね
549132人目の素数さん:03/09/02 20:38
>>546
教科書では余弦定理を用いて内積の分配法則を証明しているはず。
だから余弦定理以外で分配法則が示せることをほのめかせば減点されないと思う。
>>543
L_1^2+L_2^2+L_3^2=L^2より最大値は相加相乗平均、
最小値はL_1+L_2がL_1=0の時最小になる事、その条件の元で
L_1+L_2+L_3がL_2=0の時最小になる事を使えばよい。
551132人目の素数さん:03/09/02 22:17
スレ違いだったらスマソ。
初項がa 2項めがb 3項目がc 4項目がa 5項目がb 6項目がc
・・・(周期的にa、b、cが続く)である数列の第n項をa,b,cを用いて表せ
552543:03/09/02 22:29
>>550
OK. ただ、L_1^2+L_2^2+L_3^2=L^2 じゃなくて
L_1^2+L_2^2+L_3^2=2L^2 ですな。
553550:03/09/02 22:47
あぁ、勘違い。試験だったらこれは結構な痛手。
>>551
数列の第n項をAnと汁と
n=3k-2:An=a
n=3k-1:An=b
n=3k :An=c
ただし、kは自然数と汁。
555132人目の素数さん:03/09/02 23:33
ちなみに場合わけはなし。>>554のは×でございます。
>>555
またこの手の問題か・・・どうせsin(2π(n-1)/3)sin(2π(n+1)/3)とかつかえってんだろけど・・・なんだか・・・
勘が鋭いですねぇ。やはりそれは見当はつきそうですものね
線型代数を知っている人間には通用しないな
(=゚ω゚)ノ age
560sage:03/09/03 16:12
>>545
円に内接する三角形ABCを考える。
a=c*cosB+b*cosC
b=a*cosC+c*cosA
c=b*cosA+a*cosB
それぞれの両辺にa,-b,-cをかけて
a^2=ca*cosB+ab*cosC
-b^2=-ab*cosC-bc*cosA
-c^2=-bc*cosA-ca*cosB
これらをたし合わせると
a^2-b^2-c^2=-2bc*cosA
従って
a^2=b^2+c^2-2bc*cosA
561560:03/09/03 16:15
「円に内接する」が不要やった。当たり前やん
562大介:03/09/03 18:46
調和数列ってなんですか?教えてください
609 名前:大介 :03/09/03 18:34
調和数列ってなんですか? おしえてください

611 名前:132人目の素数さん :03/09/03 18:37
gugure
564ごめんなさい。。。ビッグバン宇宙論は間違いでした。:03/09/03 19:14
科学者よ、恥を知れ!!!
ビッグバン宇宙論は完全に間違いだった!
科学の原則を無視した、デタラメのインチキ理論だったのだ。
そして、そのビッグバン宇宙論の世界的な浸透は
アメリカ、ユダヤ・キリスト教勢力による世界支配のための思想戦略なのだ!
また、ビッグバン宇宙論の思想によって戦争が起こり、
貧富の差がひらき、終末的な絶望感が世界に蔓延しているのだ。
ビッグバン宇宙論は世界の平和を揺るがす、悪の元凶となっているのだ。
ビッグバン宇宙論とは、
「宇宙は『無』からビッグバン(大爆発)によって誕生した」という理論である。
この理論は、ユダヤ・キリスト教の創造神話(神が天地を創造した)そのものである。
ビッグバン宇宙論の実態は、科学理論ではなく宗教思想なのである。
「真空」には時間も空間も存在していて『無』ではない。
『無』は文字通り、存在するものではないのだ。だから、
『無』は科学的に証明できるものではない。
そして、『無からの誕生』も科学で証明できるものではないのだ。
だから、ビッグバン宇宙論が仮説である可能性は、0%なのだ。
ビッグバン論は完全に間違いであり、宇宙は時間も空間も無限なのである。
ビッグバン宇宙論が科学の正統であるという思想を、世界中の人々に
浸透させる戦略が成功したことにより、ユダヤ・キリスト教勢力の
世界における優位性が確立されていったのだ。(20世紀に)
そして、その思想的支配の最たるものが、アメリカやイギリスによる
イラク戦争なのだ。
ビッグバン宇宙論の浸透により、世界中に終末思想(世界の終わり)が蔓延してしまっている。
そのことにより、自己中心的、せつな的、短絡的な考え方が社会に広がっている。
科学的に間違っているビッグバン宇宙論から脱却しなければならない。
そして、宇宙は無限だということを理解しなければならない。
人間は本当の宇宙観、世界観を構築し、新しい時代に進んでいかなければならないのだ。
ビッグバン宇宙論が世界を支配している限り、平和な世界にはならないのだ。
そのことを科学者は重く受けとめるべきである。
さらばビッグバン宇宙論!!!!!!
565132人目の素数さん:03/09/05 00:40
座標平面上の単位円Cの内部は鏡になっているものとする。
いま、C上の点P_1からC上の点P_2にむかって光線を発射した。
P_1を出た光線はP_2で反射し、さらにC上の点A_1, A_2, A_3, …で反射しながら進む。
A_1, A_2, A_3, …がすべて有理点(各座標がいずれも有理数であるような点)となるための必要十分条件は
P_1, P_2が共に有理点であることを証明せよ。
まあ実際に大学の助手になったら試験作成委員とかに選ばれる可能性もなきにしもあらず
色々時間取られてかなりうざいらしいが
>>565
■P_1, P_2が共に有理点⇒A_1, A_2, A_3, …がすべて有理点

P_1, P_2が共に有理点ならば、A_1が有理点となることを示せばよい。
原点OとP2を結ぶ直線Lは傾きが有理数で切片は0。
(A1は、Lに関してP1と対称な点である。)

従ってP1を通り、Lに垂直な直線をMとすると、Mも係数が
全て有理数の式となるので、L、Mの交点Qは有理点である。
QはP1とA1の中点なので、A1は有理点。

■A_1, A_2, A_3, …がすべて有理点⇒P_1, P_2が共に有理点

光線を逆に辿って同じ議論をすればよい。
568565:03/09/06 01:04
>>567
いいね。私が用意していた解答は
複素数平面で P_1(cos(θ_1)+isin(θ_1)), P_2(cos(θ_2)+isin(θ_2)), A_n(a_n) とおけば
a_0 = cos(θ_2)+isin(θ_2), a_(-1) = cos(θ_1)+isin(θ_1) と定義してよく、
ド・モアブルの定理より導ける。
でした。
569132人目の素数さん:03/09/06 02:20
四角形ABCDにおいて、△ABD∽△BCDかつAB:AC:CD=4:5:1である。
この時、四角形の対角線の交点は対角線BDをどのような比に内分するか?

コピペですがこんなのはどうでしょう。
570132人目の素数さん:03/09/06 02:22
>>569
ちょっとかしこい中学生レベルの問題しか出せないキミは無価値
571132人目の素数さん:03/09/06 02:23
>>570
はいはい
572132人目の素数さん:03/09/06 02:26
>>570
東大でも高校入試にでてもおかしくない問題もときどきでますが
この問題は誘導がないとむずかしいと思います。
573132人目の素数さん:03/09/06 02:40
自然数全体からなる集合の任意の部分集合は最小の元を持つ事を示せ。
574132人目の素数さん:03/09/06 02:43
最小の元って高校生はわからないんじゃないか?
そこはそれ、いわゆる大小関係を順序関係として与えれば
576132人目の素数さん:03/09/06 05:02
tan2004°の小数第1位を求めよ。

どうよ? いかにも来年出そうじゃないか?
577132人目の素数さん:03/09/06 05:54
>>576
tan60°とtan36°を使ってtan24°を出すの?
578132人目の素数さん:03/09/06 06:39
579132人目の素数さん:03/09/06 09:36
>>577
お好きなように。
>最小の元って高校生はわからないんじゃないか?

そんなことは無いと信じたい・・・
>>573
反例
{Φ}
582132人目の素数さん:03/09/06 14:27
>>581
それは2^(2^N)の元。問題文は2^Nの元でって聞いてる。
だから反例は{Φ}じゃなくてただのΦ
583132人目の素数さん:03/09/06 15:19
>>577に言われて気付いた。tan2004°は簡単に求められるので tan2005°にしよう。
どうやら出題を1年勇み足したようだ(w
584132人目の素数さん:03/09/06 16:50
>>583
tan2005°=tan25°
tan(45°/2)=√2-1<tan25°<2/3tan(45°/2)+1/3tan30°=(√3+6√2-6)/9<0.47
よって小数第1位は4
>>584
tan25°<2/3tan(45°/2)+1/3tan30°に猛烈に感動した。
以下作問者による解答。

tan15°=2-√3<tan25°<tan30°<0.6 より tan25°の小数第1位は2,3,4,5のいずれか。
3倍角の公式 tan3θ={3tanθ-(tanθ)^3}/{1-3(tanθ)^2} から
tanθ_i=i/10 (i=2,3,4,5) として tan3θ_i をそれぞれ計算すると
0.…, 1.…, 2.…, 5.5 となるので tan3θ_4<tan75°=2+√3<tan3θ_5
したがって (0°<)α<25°<β(<30°)   ∴ 0.4<tan25°<0.5
よって tan2005°=tan25°の小数第1位は4となる。
5行目訂正
α→θ_4, β→θ_5
587supermathmania ◆b1q0oeBbVc :03/09/06 19:12
test
すまん、584よ、tan25°<2/3tan(45°/2)+1/3tan30°のところがよく
わからないので詳しく教えてください。だめだめでごめんなさい。。
589132人目の素数さん:03/09/07 00:01
>>588
補間法とy=tanxが0<x<90°で下に凸であることから
590132人目の素数さん:03/09/09 20:00
3辺の長さがいずれも素数であるような三角形の面積は整数とはならないことを示せ。
591便乗:03/09/09 20:09
s,を整数p,q,rを素数とする。
この時
(p+q+r)(p+q-r)(p-q+r)(-p+q+r)=16s^2
は解を持たないことを示せ。
>>591
p=5,q=2,r=3,s=0は解。
593132人目の素数さん:03/09/09 21:47
>>592
この際s≠0としようね。
594ヌーン:03/09/11 08:20
無限級数 Σ[n=k,∞](-1)^(n-1)/2n-1 の和を求めよ.
595132人目の素数さん:03/09/11 12:56
赤本で東大の問題みたら中学3年レベルの数学ができれば解ける問題が載ってた。
はやりのか?
はやりのか??
597132人目の素数さん:03/09/11 13:04
はやりなのか=流行なのかな?

もしかして方言?
はやりのかははやりなのかなのか
599132人目の素数さん:03/09/17 08:15
|cosx-Σ[l=0,n](-1)^lx^2x/(2l)!|≦|x|^2n+2/(2l+2)!
を示し、
cosx=Σ[l=0,∞](-1)^lx^2x/(2l)!
を証明せよ。


 高校生では三角函数の無限級数展開範囲外でしたっけ?
600132人目の素数さん:03/09/17 09:25
>>599
お前が範囲外
601132人目の素数さん:03/09/21 23:57
だいぶ廃れてきたな。では問題を。

nを自然数とする。
実力が互角な(2n+1)チームによる総当たり戦を行うとき、
どのチームもn勝n敗になる確率を求めよ。
ただし引き分けはないものとする。
602132人目の素数さん:03/09/22 00:00

     ∧_∧∩ / ̄ ̄ ̄ ̄ ̄ ̄ ̄ ̄ ̄ ̄ ̄
    ( ´Д`)/< 先生! 自然数に0は入りますか?
 _ / /   /   \___________
\⊂ノ ̄ ̄ ̄ ̄\
 ||\        \
 ||\|| ̄ ̄ ̄ ̄ ̄||
 ||  || ̄ ̄ ̄ ̄ ̄||
    .||          ||
603132人目の素数さん:03/09/22 00:06
高校範囲ということで、入れない方向でおながいします。
>>602
入りますよ。って中学の時の先生が言ってた。
恐ろしくバカな教師だ。
>>602
入れるかどうかは個人の自由。
>>601
2{(2n)!/(2^n・n!・(n-1)!)}^(2n+1)÷2^C[2n+1,2] になった。あってる?
607606:03/09/22 00:41
しまった。まちがった。無視して下さい。
>>601
2{(2n)!/(n!n!2^n)}^(2n+1)÷2^C[2n+1,2]
これでどう?
609601:03/09/22 01:21
>>608
自分も答えに自信がないんだけど、多分違う。
2{(2n)!/(n!n!2^n)}^(2n+1) がn=2で整数でないし。
610608:03/09/22 01:31
あ、ホントだ。逝ってくる
>>601
もしかして2^(-2n)?
612611:03/09/22 03:29
ちがった。逝ってきます。
613132人目の素数さん:03/09/22 05:17
>>605
     ∧_∧∩ / ̄ ̄ ̄ ̄ ̄ ̄ ̄ ̄ ̄ ̄ ̄
    ( ´Д`)/< 先生! おまんこにてぃむぽは入りますか?
 _ / /   /   \___________
\⊂ノ ̄ ̄ ̄ ̄\
 ||\        \
 ||\|| ̄ ̄ ̄ ̄ ̄||
 ||  || ̄ ̄ ̄ ̄ ̄||
    .||          ||
614132人目の素数さん:03/09/22 09:52
おいお前ら

答えが2chになる東大問題考えれ!!
>>614
答えが2abになる問題持ってきてaとbをcとhに変えれば終わり
616132人目の素数さん:03/09/22 14:32
>>590
ほんとうなのか?
617590:03/09/22 15:30
>>616
本当です。
>>614
どちらかというと物理の答えに出てきそうな感じ。
620132人目の素数さん:03/09/22 17:40
>>590
1/2bcsinA 1/2acsinB 1/2absinC 全部整数にならなきゃいけなくて、
しかもa,b,c素数だから、少なくとも2つの辺は2。
三辺を2,2,aとすると、面積は(4-a^2/4)^(1/2)*a*1/2だけど、
aが2だと整数にならないから、(4-a^2/4)^(1/2)は2の倍数じゃなきゃいけないけど、
aは奇数だから無理。

間違ってるかな・・・
621132人目の素数さん:03/09/22 18:00
>1/2bcsinA 1/2acsinB 1/2absinC 全部整数にならなきゃいけなくて、
>しかもa,b,c素数だから、少なくとも2つの辺は2。
が嘘くさい。論理が飛躍しすぎ。
622132人目の素数さん:03/09/22 18:16
>>621
一応、一つ目から、b or cが2の倍数 以下同様 で
偶数素数は2のみだから、少なくとも2つは2
とやっていったのですが、ダメですか?・・・
623132人目の素数さん:03/09/22 18:32
たとえば
sinA=2/5 のとき、b=5,c=11
とかでも整数になるよ。
>>623
ああ、そうですね・・・。
簡単だと思ったw
625132人目の素数さん:03/09/22 21:57
>>624
難しく考えすぎなんだよ。
>>591がほとんど答えを言っているようなもの。
626132人目の素数さん:03/09/22 22:05
>>591のヒントなしだと試験にならないのでは?
627132人目の素数さん:03/09/22 23:07
いや、与えられた情報が辺の長さのみで
面積の話なんだからヘロンの公式は当然だろ。
628132人目の素数さん:03/09/22 23:08
もしかして今ヘロンの公式って教科書載ってない?
だったら(1)にヘロンの公式の証明が必要かも…。
629626:03/09/22 23:17
>>627
ごもっともだけど、>>591自体軽くはないから、やるには度胸が必要(w
理IVの試験ならともかく

>>628
そんな感じで、あらぬ方向に目を向けて時間を潰さないための措置が
ないと可哀想な気が

時間倍ならノーヒントでOKとも思うけど
そうか、すまなかった。
でも>>590>>591よりはかなり簡単なんだよ。じつは。
>>591でも相当簡単になってるけど。これ以上に簡単にしめせるん?
いや、三角形の成立条件を考えるとp,q,rがそれほど
散らばらないってこと。大して変わんない。

もうここまできたら良いか。以下解答。

a,b,cを題意の三角形の3辺として面積はヘロンの公式より
{(a+b+c)(-a+b+c)(a-b+c)(a+b-c)}^(1/2)/4
であたえられる。これが整数となるには
(a+b+c)(-a+b+c)(a-b+c)(a+b-c)
が偶数であることが必要。そのためにはa,b,cの1つまたは3つが
偶数であることが必要十分。
1つが偶数のとき、それは2である。残りの2つをq,rとする。
もし q≠r とすると |q-r|>2 となり三角形の成立条件を満たさない。
したがって3つが偶数の場合も含めて {a,b,c} = {2,p,p} (p素数) とかける。
すると面積は (p^2-1)^(1/2) となって整数でない。(∵p≧2)
訂正

もし q≠r とすると |q-r|>2 となり三角形の成立条件を満たさない。

もし q≠r とすると |q-r|≧2 となり三角形の成立条件を満たさない。
634132人目の素数さん:03/09/23 00:15
なるほど
635626:03/09/23 00:22
>>633
漏れが解いたのと同じ感じ。
回答はさりげなく短いけど、(凡人には)try&error必須なんで結構辛いね。

E★★★★くらい?
636川口:03/09/23 02:04
東京大学よりも難しい問題出してるとこだってあるのに、なんでこのスレ立ち上げたの
「理系で数学が得意な高校生が25〜50分で解ける問題」
ってのは小ネタとしては丁度いいのですよ
638132人目の素数さん:03/09/23 13:37
             __   /⌒ヽ
          _,.-ニ二ヽ〃´ ̄ ̄`ヽ、
 (⌒⌒)    / _,.-‐''"´`ヽ/⌒ヽ、 `ヽ、
  \/   〃 /  _,.-‐- ‐- 、  ヽ \ヾ、
      ,-‐/  / / /     \  ヽ ヽ
     / /   /_//_/l/|l l /|_i,,,_ヽ  ヽ、!
   /  /イ  ハ/l/__l | ヾN/ _!」,,i/ト   lヽ、  アイアイアイフル〜♪
   `ー--‐|/| イ/ |` ゙̄''     〃::゚!i!|    l ヽ
     __  レト_lヾi、   __ '  ヾ;;ツ,l!.レ' !. l  \
     ヽ、ヽ (ヽ ヽ|\  `‐'   `/)|  レi |`ー-一'
      (⌒ヽ`ヽ `レ|>、,.-r‐ニ´ ̄´|ハ| レ'
      { `二)-、ト、_i´ ̄`ii´ ̄`!___  \
      ,へー'____ハ \,.-、/ ̄ } ̄ヾ、__/ハ
      //ハ、____,イ >{___}<  〉  ` ̄´ }
       {∨  〉   l /l| |i\_/-‐'-、/  ,ノ
      \ /i   |´  |l !|  L-‐-、/ ̄
       `ー|   |  || ||   |   |
639132人目の素数さん:03/09/24 00:27
>>601答えおしえてくれage
640[[[[ BIGFROG ]]]]:03/09/24 17:47

http://www.doze.com/members/bigfrog/

東京大学同窓生・在校生の為のBBS・チャットルーム。
641132人目の素数さん:03/09/24 22:13
>>601答えおしえてくれage ぱ〜とつ〜
(2n)!/2^{n(2n+1)}
643132人目の素数さん:03/09/24 23:48
>>642
それどうやって証明するんですか?
Π[1≦k≦n](2k C k)/ 2^{n(2n+1)}
645132人目の素数さん:03/09/25 01:08
>>644
なんでそんな小出しにかくんですか?答えスパッと書いてよ。
このスレ終わったな
647132人目の素数さん:03/09/25 01:28
というか>>642>>644じゃないの?>>644計算しても>>642にならないもん。
計算機でやったら違う値になった。どっちが正しいの?
ちなみに計算プログラムと結果
def combination(i,j)
if(i==0||j==0||j==i)
return(1)
else
return(combination(i-1,j)+combination(i-1,j-1))
end
end

def factor(n)
if (n==0)
return(1)
else
return(n*factor(n-1))
end
end

def func1(n)
r=1
for i in 1..n do
r=r*combination(2*i,i)
end
return(r)
end

for i in 1..10
print func1(i)," "
print factor(2*i)
puts
end
−結果−
2 2
12 24
240 720
16800 40320
4233600 3628800
3911846400 479001600
13425456844800 87178291200
172785629592576000 20922789888000
8400837310791045120000 6402373705728000
1552105098192510332190720000 2432902008176640000
再帰使ってcombinationの計算させると遅いよ。
>>650
そうなん?どうするのがいいの?
>>644はn=2で合わないような気がするのだが。
まあプログラムの実行速度の問題はさておき>>642とか>>644は論述部分が
ないからどっちが正しいのか判定できないね。かといって>>601の組み合わせ
かぞえあげるプログラムはそんなに速攻ではつくれないし。
明日休みだからプログラム組んでみよ。
>>652
そうだねn=2みたら>>642の方があってそう。
>>651
この方が速いはず。

def c(n, i)
 return 0 if i < 0 or n < i
 return 1 if i == 0
 return c(n, n-i) if 2*i > n
 c = 1; i.times{|j| c = c*(n-j)/(j+1)}
 return c
end
>>655
thx. すなおに(m+n)!/m!n!つかったほうがいいってことね。そりゃそうか。多謝。
間違ってるかもしれんがPCで数え上げたらこうなった。
2
24
2640=2^4*3*5*11
3230080=2^7*5*7^2*103
これから考えると単項式では表せないっぽい?
658132人目の素数さん:03/09/25 03:02
大田豚江は、道頓堀への飛び込みを煽り死者を出しただけでなく、
府庁舎内に億単位の金をかけて女子便所を作り、
数千万の金を四天王寺ワッソという、
朝鮮人の捏造祭り(11/3開催)につぎ込み、
朝鮮人のテーマパークを関空横に大金をかけて作ろうとし(2005年春)、
大阪と縁もゆかりもない阪神タイガースのパレードを
御堂筋に誘致しようとしているキチガイだ。

創価学会パワー、炸裂ですか?
府民を冒涜するのもたいがいにせい。
こいつは一体、
な に を か ん が え て い る ん だ 。
>>657
ソースは長い?
660657:03/09/25 03:42
DelphiPascalです。
function c(m: Integer): Integer;
const
MAX = 100;
var
x: array[0..MAX{2*m}] of Integer;//loop counters
a: array[0..MAX{2*m}] of Integer;//勝敗表

procedure loop(i: Integer);
var j,k: Integer;
begin
if i > 2*m then begin
// 全試合終わった
for j := 0 to 2*m do begin
if a[j] <> m then Exit;
end;
Inc(Result);
Exit;
end else begin
// i対kとの試合
k := x[i];
if x[i] < i-1 then begin
Inc(x[i]);

if a[i] < m then begin//枝刈り
//i勝ち, k負け
Inc(a[i]);
loop(i);
Dec(a[i]);
end;
661657:03/09/25 03:43
if a[k] < m then begin
//k勝ち, i負け
Inc(a[k]);
loop(i);
Dec(a[k]);
end;

Dec(x[i]);
end else begin
if a[i] < m then begin
//i勝ち, k負け
Inc(a[i]);
loop(i+1);
Dec(a[i]);
end;
if a[k] < m then begin
//k勝ち, i負け
Inc(a[k]);
loop(i+1);
Dec(a[k]);
end;
end;
end;
end;

begin
FillChar(x, SizeOf(x), 0);
FillChar(a, SizeOf(a), 0);
Result := 0;
loop(1);
end;
>>660-661
乙。コードにあやまりがないかどうかパッとはわかんないけどあってるとすると
いままであがってる解答は全部まちがってるってことだね。
出題者の解答はどうなってるんだろ?
663657:03/09/25 06:22
考えずに作った変なコードだったので、
コードを見直した。
ついでに標準Pascalにし、またグローバルを使って
Cに変換しやすいようにしてみた。
数え上げの結果は変わらなかった。

var k: Integer; //チーム数は2*k+1
  w: array[0..10{2*k}] of Integer; //勝敗表
function loop(i, j: Integer): Integer;
var res: Integer;
begin
 if j > 2*k then
  loop := 1// すべてのチームが勝ち数kで終わった
 else // i対jの試合(i<j)
  if i < j then begin
   res := 0;
   if w[i] < k then begin//勝ち数k以下が必要
    w[i] := w[i] + 1; //i勝ち, j負け
    res := res + loop(i+1, j);
    w[i] := w[i] - 1
   end;
   if w[j] < k then begin//勝ち数k以下が必要
    w[j] := w[j] + 1; //j勝ち, i負け
    res := res + loop(i+1, j);
    w[j] := w[j] - 1
   end;
   loop := res
  end else
   loop := loop(0, j+1)
end;
664657:03/09/25 06:23
function count(n: Integer): Integer;
var i: Integer;
begin
 k := n;
 for i := 0 to 2*k do w[i] := 0;
 count := loop(0,1)
end;
とりあえず
a[1]=1,a[2]=1,a[n+2]=a[n+1]+a[n]
(フィボナッチの数列)
これの一般項a[n]を求めよ
東大京大の問題にはふさわしくないかもしれんが
高校までの知識でだれか解いてみ。
俺は高1で解けたから、おまいらなら余裕で解けるはず。
P={(i,j)|(i,j)∈{1,2,...,2n+1}^2(i≠j)}とする時
{i(1),i(2)),(i(2),i(3)),(i(3),i(4))....(i(m),i(1))という形の
集合をPの鎖といいます。
Pを鎖で分割する方法、すなわちPのどの元も漏れなくただ一つの鎖にただ一回
現れるような、鎖の集合の和で表す方法はどの位あるでしょうか?
出来ればnの式で表して下さい・
>>663-664
乙です。オレもいまBCCでくんで>>657と同じ答えになった。n=5計算してみようとしたら
うんともすんともいわなくなった。オレの愛機ではこのへんが限界のようだ。
というわけで外出の解答>>642>>644はいづれもまちがってるくさい。どっちか出題者なのかな?
そうだとしたら出題ミスでファイナルアンサー?>>601
668132人目の素数さん:03/09/25 17:58
>>666
出題者?もってる答え>>657と同じになる?
669132人目の素数さん:03/09/25 18:07
>>665
そんな超既出問題だすな低脳
670132人目の素数さん:03/09/25 18:09
>>665
教科書にのってる
671132人目の素数さん:03/09/25 18:13
とりあえず
a[1]=1,a[2]=1,a[n+2]=a[n+1]+a[n]
(フィボナッチの数列)
これの一般項a[n]を求めよ
東大京大の問題にはふさわしくないかもしれんが
高校までの知識でだれか解いてみ。
俺は中3で解けたから、おまいらなら余裕で解けるはず。
>>671
No-Hintは出題範囲違反に抵触する。この問題が現実に出題される
ことは有り得ない。
>>672
そう?3項間関係の漸化式なんてNo-Hintででまくってそうな気するけど?
やさしすぎて出ない可能性は高いけど。
>>674
精々特性方程式が自然数、或いは有理数の解を持つ場合だけ。
以前は特性方程式を使って解く方法を教えていたようだが、
この方法自体古い方法なので教えられなくなった。復活した
可能性は薄いので、特性方程式が無理数の場合は誘導付の筈
しかし見え透いているので確かに出題率は低い。
フィボナッチをだすなら、リュカ数との混合問題にした方がいいだろ。
∫x^x dx を求めよ (挑戦問題)

※挑戦問題 とは?
受けに来る者達の学力からでは、
到底解く事ができないであろう問題だが、
その難題をいかにして解こうと努力したかの経緯を
見る為の問題。他にも呼び方はある。
こういう問題出す大学、もうないんだろうな
今ならそんな問題出したら、問題に不備があるとか
突っ込まれるだろうなぁ。解けないし。

昔はそんなものを出したこともあったのか?
678132人目の素数さん:03/09/26 00:35
自然数a,b,cが
1/a + 1/b = 1/c
を満たすとき、a,b,cが満たすべき条件を求めよ。
679132人目の素数さん:03/09/26 02:44
>>677
回りからバカにされているのに気付かない人だね
101010・・・0101となる101以上の数のうち素数となるものを全て求めよ。
101
次の各命題が正しい場合は証明し、誤りである場合は反例を挙げよ。

(1) a, b を実数とする。a+b, ab がともに有理数となるための
  必要十分条件は、a, bがともに有理数であることである。

(2) a, b を有理数とする。a+b, ab がともに整数となるための
  必要十分条件は、a, bがともに整数であることである。

# ちとヌル過ぎかな?まあゴッツアン問題ってことで。
a + b, ab は a, b を解に持つ二次方程式の係数だから……。
mを正の実数の変数とするとき、座標平面上に曲線C:y=(e^x)^tanx(0≦x≦π/4),直線l:y=mx+1がある。
このとき、曲線C・直線lによって囲まれる部分と曲線C・直線l・x=e^π/4によって囲まれる部分の面積の総和が
最小となるmを求めよ。
もう厭きたこのタイプ
なら解いて下さい。
687132人目の素数さん:03/10/04 22:44
a,b,c,d,x,yはいずれも実数で xy≠0 とする。数列 {x[n]}, {y[n]} が
 x[1] = x , y[1] = y
 x[n+1] = ax[n] + by[n] , y[n+1] = cx[n] + dy[n] (n=1,2,…)
をみたしている。
次の命題が真となるためにa,b,c,dがみたすべき条件を求めよ。

(命題)
「{x[n]}, {y[n]} がいずれも等差数列ならば
 行列([a,b][c,d])はE(2次の単位行列)の実数倍ではない。」
「はみだし切り法」とかいうのが昔使えてたらしいけどそれが使えれば瞬殺の問題でしょ?
それを今の受験数学でつかえる範囲で書きなおす作業をするだけ。
そもそも「はみだし切り法」が今の受験数学でつかえないのかどうかも微妙なんだけど。
−解答−
f(x)=e^(xtanx)とおく。m<f'(0)で面積は単調減少、m>(f(1)-f(0))/(1-0)で単調増大なので
f'(0)≦m≦(f(1)-f(0))/(1-0)においてSは最小値をとる。
直線の仰角をθ、仰角がθのときの面積の和をS(θ)とする。
直線と曲線の交点でx>0の部分にある方のx座標をa(θ)とする。
a(θ)は微分可能(略)。
十分小さいh>0について
S(θ+h)-S(θ)≦a(θ+h)・a(θ+h)・(tan(θ+h)-tanθ)-(1/2)・(tan(θ+h)-tanθ)
S(θ+h)-S(θ)≧a(θ)・a(θ)・(tan(θ+h)-tanθ)-(1/2)・(tan(θ+h)-tanθ)
S(θ-h)-S(θ)≦(1/2)・(tanθ-tan(θ-h))-a(θ)・a(θ)・(tanθ-tan(θ-h))
S(θ-h)-S(θ)≧(1/2)・(tanθ-tan(θ-h))-a(θ+h)・a(θ+h)・(tanθ-tan(θ-h))
(図をかけばしめせる。詳細略)
よってS(θ)は微分可能でS'(θ)=(a^2-(1/2))(1/cos^2(θ))。
よってa=1/(√2)となるときが最小で傾きは(e^((1/√2)tan(1/√2))-1)/(1/√2)。
689132人目の素数さん:03/10/04 23:47
(誤)
「{x[n]}, {y[n]} がいずれも等差数列ならば
 行列([a,b][c,d])はE(2次の単位行列)の実数倍ではない。」
(正)
「{x[n]}, {y[n]} がいずれも公差が.0でない等差数列ならば
 行列([a,b][c,d])はE(2次の単位行列)の実数倍ではない。」
690浅田満:03/10/04 23:48
必死だな(w
>>688
はみ出し法は其の区域内で其の関数が連続で微分可能、一次導関数が0以上かつ二次導関数が解を持たない(つまり負又は正)
でないと、その1/√2:1-(1/√2)に内分する点が面積最小だとは言えない。
恐らく二次導関数は1つ解を持ってCとLが3回まざわるときが解だと思われます。
>>694のタイプの問題なんどもなんども貼りつける香具師がいるんだがマニアなのかな?
693132人目の素数さん:03/10/05 02:30
694に期待
りんごが3つ、みかんが3つ、マスカットが4つあります。
果物は全部合わせていくつあるでしょう。
ただし、解答は「微分・積分」「行列」「複素数」のいずれかを使って論拠の整ったものにしなさい。
次の(1)〜(3)を計算せよ。
(1)lim[x→+0](x^0)
(2)lim[x→+0](0^x)
(3)lim[x→+0](x^x)
>>694マニアハケーン
lim[x -> +0] f(x) = 0
lim[x -> +0] g(x) = 0
lim[x -> +0] f(x)^g(x) = π
となる f(x), g(x) を構成せよ。
f(x)=(1/π)^(1/x)、g(x)=-x
次の(i)(ii)をみたす数列 {a(n)} の一般項 a(n) を求めよ。
(i) a(1)=1, a(2)=1
(ii) n*a(n+2) = {(n+1)^2}*a(n+1) + {(n^2)+n}*a(n)
700132人目の素数さん:03/10/08 22:27
>>699の答えおしえてくれage
701132人目の素数さん:03/10/08 22:54
僕も
702699:03/10/08 23:27
悪い。問題の条件間違えてた。
駄問でスマン。

(ii) n*a(n+2) = {(n+1)^2}*a(n+1) - {n(n+1)}*a(n)
                   ~~
a(n)=(n-1)! (n≧3)かな?
あたり。0!=1 として n≧1 でもいけます。
補足トリビア

(ii) ⇔ a(n+2) - (n+1)*a(n+1) = {(n+1)/n}{a(n+1)-n*a(n)}

この変形を見抜いたらすごい。
706132人目の素数さん:03/10/09 14:44
>>705
そうかなぁ
707132人目の素数さん:03/10/09 15:14
ん結局
>>601って
(1/2)^{(3n^2+n)/2}
と出たけどOK?
708132人目の素数さん:03/10/09 15:27
中核派と核マルの戦いなんて悲惨そのもの。
いい歳した団塊の世代のオヤジが未だに襲撃しあったりしている。

お前らの人生って一体なんだったの?って感じ。
>>707
>>657と一致しないからまちがってると思う。
710132人目の素数さん:03/10/09 21:57
2^a+3^b=5^c をみたす自然数の組(a,b,c)をすべて求めよ。
(a,b,c)=(4,2,2),(1,1,1)ですか?
712132人目の素数さん:03/10/10 14:52
>>710
採点しる!

>>711
それ以外にないなら証明しる!
713132人目の素数さん:03/10/10 22:58

    |┃三   人      _________
    |┃   (_ )    /
    |┃ ≡ (__) <  オヤジ!冷やしうんこ一丁!!!
____.|ミ\__( ・∀・)  \
    |┃=__    \    ̄ ̄ ̄ ̄ ̄ ̄ ̄ ̄ ̄
    |┃ ≡ )  人 \ ガラッ
下図のような3×3のマスがある。
┌─┬─┬─┐
│1 │ 2│ 3│
├─┼─┼─┤
│4 │ 5│ 6│
├─┼─┼─┤
│7 │ 8│ 9│
└─┴─┴─┘
時刻0においてA君は1のマスに、B君は9のマスにいるとする。
時刻が1増えるごとにA君、B君は隣のマスにランダムに移動する。
例えば2のマスにいるときは、1,3,5のマスに移動する確率がそれぞれ1/3である。
さて、時刻tにA君とB君が同じマスにいる確率をP(t)として
P(2), P(3), P(4) を求めよ。
1/6 2/9 35/162
センターで誘導で出そう
716132人目の素数さん:03/10/11 01:17
>>714はいい問題だと思うけど本当に自分で考えたのかな?
717714:03/10/11 21:00
>>715
正解。

>>716
thx. 一応自作です。

追加問題:
時刻tまでに少なくとも1回A君とB君が出会う確率をQ(t)として
lim[t→∞]Q(t)=1 を証明せよ。
718Galois:03/10/12 04:27
<<717
すごくいい問題だなあ(ただ717はまだ解いていない)!!
特に本当に717が示せるなら、確率論の2次元ランダムウォークの
再帰性を条件付ながら示したことになる!!
寝て起きたら解いてみます。ここのスレ面白い問題結構あるね。
<<30もよいと思う。どこかの入試ででるかも?
719132人目の素数さん:03/10/13 01:30
1. 方程式 x^3-kx+1 = 0 ・・・(a) を考える。
  (1) 方程式(a)の持つ解の個数を求めよ。
    (2) 方程式(a)はkが十分大きいとき異なる3つの解を持つ。その解を小さい方からα(1)、
      α(2)、α(3)とおく。このとき、n=1,2,3に対して極限
         lim[k→∞] α(n)/x^t 
     が0でない定数に収束するように実数tの値を定めよ。

2. 最高次数が2以上である多項式f(x)に対して、関数g(x)を
         g(x) = f(x)・f(1/x)
   と定める。t = x+(1/x) とおくと、g(x)はtの多項式として表せることを
   示せ。

3. 楕円C1: (x^2/4) + y^2 = 1
   楕円C2: x^2 + (y^2/4) = 1
   を考える。C1上の点(2,0)に点Pが、C2上の点(0,2)に点Qがあり、Pでの
   C1の接線をl、QでのC2の接線をmとすると、点Pと点Qは常にlとmが直交す
   るように、反時計回りに連続的に動く。

(1) PとQの交点をRとする。Rが動いてできる曲線Crは、xとyの多項式
     f(x,y)を用いて
         f(x,y) = 0
     の形で表される。f(x,y)を求めよ。
   (2) Crは直線 y=x に関し対称であることを示せ。
   (3) 傾きが-1である直線で、Crと異なる2点で交わるものを考える。
     そのような直線のうち、次の条件を満たすものを一つ求めよ。
   「CrがLから切り取る長さが最大である」
720132人目の素数さん:03/10/13 01:46
t>3であればA、Bともにどの場所にいてもよいので、A,Bが重ならないような配置を考える。
この際、正方形は点対称な図形であるので、時刻Tにおいて
(1、2)にいる確率をAt、(1、3)をBt,(1、5)をCt,(1、8)をDt,(1、9)をEt、(2、4)をFt,(2、5)をGt,(2、8)をHt
としてA、Bが出会わないという条件下で8個の漸化式を立てる。
っと・・・これ以降は書くと死ねるので割愛・・・
ただ一応lim[t→∞]Q(t)=1にはなりそうです。計算途中ですが(ぉ

上手い解法例教えて下さい(汗
721717:03/10/13 02:09
ヒント。
マンドクセーのでtが奇数のときのみを考えてみる。
するとA君B君ともに偶数番号のマスにいる罠。
A君B君がどの偶数番号のマスにいても次の偶数時刻において出会う可能性がある。
t→∞で
sが奇数の時 A,Bが同じマスにいる確率は1/4
sが偶数の時 A,Bが同じマスにいる確率は2/9
よってt→∞でA,Bが少なくとも一回出会う確率はほぼ lim[t→∞]1-{(3/4)^(t/2)}*{(7/9)^(t/2)}=1

こんなんじゃだめ?w
723717:03/10/13 19:42
実験用のHTMLを作ったのでよかったら使ってみて。(JavaScript使用)
ttp://up.isp.2ch.net/up/867dc3ea7a7f.zip
724132人目の素数さん:03/10/17 22:42
xy平面上に(x/a)^2 +(y/b)^2 =1の楕円Cが存在する。(a,b実数)
この楕円Cは常に周の長さが1である時、楕円Cの存在しうる領域の面積をもとめよ。

>>724を受験してきますた。

ちょっと酒入っているので解けるか不安でしたが…
23:50開始

楕円の酋長?こりゃ鬼門だパス…と思ったが、今回は一問しかない(´・ω・`) (0.5分)
誘拐領域なのは明らかだな(1分)
絵をいっぱい描いてあそぶ フジテレビマークも描いたよ(3分)
手の運動のため、条件をxで表してみる…だから何?(5分)
パラメータ表示してもだめだろうな、やっぱり(7分)
友達から電話がかかってきたので相手をする(20分)
おっと、時間が…というか、制限時間何分?
あ、平成少女図鑑 SHO-003 中村由季ダウソ完了(32分)

やーめた
漏れの入学時に出なくて良かった
>>725
ワロタ
724解けるの?(高校範囲内で)
>>724
愚問。
ってか解けないだろ。
>>724
これオレも解けない気がするんだけど。また出題ミス?
楕円の周の長さは楕円積分という積分で表されるだけで、実値は近似値しか出せない。
だから、特殊な方法でも使わないと解けないのは明らか。
各項が正である数列{a(n)}が、任意の正整数nに対して
 {a(1) + a(2) +・・・+ a(n)}^2 = a(1)^3 + a(2)^3 +・・・+a(n)^3
を満たすとき、{a(n)}の一般項を求めよ。
>>731
a(n)=n
733132人目の素数さん:03/10/19 10:03
半径1の円Cと、円Cの中心と距離が1/2の直線Lが存在する。
円Cを直線Lについて回転した立体の体積を求めよ。

簡単かな?
>>732
それに限ることを示してもらわんと・・・
帰納法で示せる。
Σ[i<n]a(i)=aとおいて{a+a(n)}^2=a^2+a(n)^3を解くと正の解は一つしかない。
>>731
俺高3だけどその問題見たことあるぞ
熊本女子大とかの問題だったっけ
737132人目の素数さん[733]:03/10/19 18:09
パップス・ギュルダンの定理によると回転体の体積は
 (回転図形の面積)*(回転図形の重心の移動距離)
と表すことが出来る。
さて、半径1の円Cと、円Cの中心と距離がcosθ(0≦θ≦π/2)の直線Lが存在し、
CをLについて回転したとき、上定理に従えば、どこに回転図形の重心があると考えるのか妥当か。
その重心と直線との距離をθで表せ。

こんな入試絶対出ないけど・・・
a_1=a_2=1 a_(n+2)=a_(n+1)+a_n を満たす数列{a_n}について
ミスった

a_1=a_2=1、a_(n+2)=a_(n+1)+a_n を満たす数列{a_n}について
{b_n}を b_n=納k=1→n]{(a_k)^(a_k)} で定義する。
b_11111! の下2桁を求めよ。
>>739
でけた。メンドイので合同式つかわせてもらってまず自然数a,bに対し
a≡b (mod 100)⇒a^a≡b^b (mod 25)
実際a≡b (mod 100)と仮定する。5|aならa^a≡b^b≡0 (mod 100)
(a,5)=1なら Z/25Zの乗法群が位数20であることから
a≡b (mod 20)より a^a≡a^b (mod 25)。
またa≡b (mod 25)より a^b≡b^b (mod 25)。
同様にして a≡b (mod 4)⇒a^a≡b^b (mod 4)
以上よりa≡b (mod 100)⇒a^a≡b^b (mod 100)
さらにa(n)≡a(n+300) (mod 100)より
納k=p+1→p+300]{(a_k)^(a_k)}≡納k=p+301→p+600]{(a_k)^(a_k)} (∀p≧0)
よって11111!/300=mとおくとき
b_(11111!)≡(b_300)・m (mod 100)
一方m≡0 (mod 100)であるのでb_(11111!)≡0 (mod 100)
・・・受験数学でつかっていいかどうかあやしいテクつかいまくりだが・・・
>>736
俺もやったことある。有名問題だろ、たぶん。
742( ´∀`) ◆jUUvOZ/9.Y :03/10/20 02:19
…前スレからの滞在者いる??
もしいたらみんなでどの問題が良問か言い合って
その中からいくつか絞って2ch数学版問題をつくりませぬか??

って、もう誰か言ったかな??
言い出しっぺの法則
>>740
あんたすげぇ。その問題、実は
ttp://www.tokyo-s.jp/susanowo/2001_012/index.html
のやつなんだけどね。


あなた予備校の講師とか?
>>742
(・∀・)イイ!!ねそれ
mathemaniaの問題を集めたスレならあった。
747132人目の素数さん:03/10/21 20:42
>>742
前スレが立ったときから見てますよん
あと、☆2ちゃんねらーず編・大学入試数学問題集☆
http://science.2ch.net/test/read.cgi/math/988730706/ も
リアルで出版するってこと?おもしろそう
ぜひ問題集の傾向としては定石でぱっと解けるんじゃなくて
式の図形的意味を考えた、こんな見方があったのかって解法を重視していきたいな
あと大学入試の範囲におさまらず
大学で習う数学の範囲だけどみんなが興味を持ちそうな分野の紹介とか入れたい
748132人目の素数さん[733]:03/10/23 01:39
(1)正十二角形の隣り合う面のなす角をθ゜とすると、110<θ<120となることを証明せよ。
(2)θと115の大小を調べよ。

まず自分の解答が怪しいので解答してくれると助かります(ぉぃ
(2)はできる・・・のかな?
>>744
>あなた予備校の講師とか?

いんや、名も無き引き篭もりだが何か?
こんな問題、たいしたことないぜ。
予備校の講師に憧れているのかしらんが、レベルの低い奴ばっかりだぜ。
口の悪い2チャネラーの方が、よっぽど頭いいと思うぞ。
12面体?
751132人目の素数さん[733]:03/10/23 02:18
>>750
さようでございます(汗)
ただ問題ミスのようなので正十二面体の隣り合う面のなす角の余弦を求めよ。
とします。
752132人目の素数さん[733]:03/10/23 02:26
何度も失礼します(汗汗)
>>748の(2)は解けるのであれば、宜しくお願いします(ぉぃ
作成いておいて何ですが、高校履修範囲内では解けそうにないので・・・
753132人目の素数さん:03/10/23 02:56
cosθ=-1/√5であってる?
754132人目の素数さん:03/10/23 03:02
n個の整数を掛け合わせたときの下一桁がkになる確率P(k,n)を求めよ。
>>749
ほんと引き篭もりなの?なんかもったいない存在だなぁ
予備校の講師とかになって金かせげばいいじゃないですか
>>719
1.の(2)だけどlim[k→∞] α(n)/x^t じゃなくてlim[k→∞] α(n)/k^tじゃない?
っていうか回答を・・・ 
757132人目の素数さん[733]:03/10/23 21:48
>>753
あっています。
758132人目の素数さん:03/10/23 22:06
xy平面上にf(x)>0で連続な関数f(x)が存在する。
このxy平面上にx=0,x=a(a>0),x軸,y=f(x)で囲まれる面積をn等分するように,直線x=x_i
(i=1,2・・・n-1、且つx_i<x_i+1)をとった。
このとき下の等式が成立することを証明せよ。

lim[n→∞]Σ[0〜n-1]f(x_i)/n
=∫[0〜a]{f(x)}^2 dx/∫[0〜a]f(x) dx
>>754
P(0,n)=(10^n-2*8^n+7^n)/10^n

0以外はムズそうなのでパス。
0も簡単な割にはひどく面倒だった。
>>754
下一桁は0〜9まで同様に確からしいと仮定していいの?
761132人目の素数さん:03/10/24 00:36
n、mを自然数、pを3以上の素数とし、
n<m    p<m   が成り立っている。
また、複素数α、β、γにおいて、
|α|:|β|:|γ|=p:n:m
α/β=(p/n)i
α+β+γ=0
が成り立つとき、nが4の倍数であることを示せ。

結構楽に解けるはず
762132人目の素数さん:03/10/24 01:51
|α|:|β|:|γ|=p:n:m ....(1)
α/β=(p/n)i ....(2)
α+β+γ=0 ....(3)

(2),(3)より、
α=(p/n)βi ....(4)
γ=-((p/n)i +1)β....(5)

(4),(5)を(1)の左辺に代入して整理すると、
(1)の左辺=|α|:|β|:|γ|=p:n:√(p^2+n^2)....(6)
となる。
(6)と(1)の右辺より、∴m=√(p^2+n^2).....(7)
(7)を変形すると、
p^2=m^2-n^2=(m-n)(m+n)....(8)
pは素数であるから、
m-n=1....(9) と m+n=p^2....(10) が言える。
(9),(19)より、mを消去すると、
2n+1=p^2....(11)
ここでpは3以上の素数で、奇数であるから
p=2k+1(k:整数) とおくと、(11)は
2n+1=4k^2+4k+1....(12) となる。
∴n=2k(k+1).....(13) 
ここでk(k+1)は偶数であるから、k(k+1)=2q(q:整数) とおくと、
∴n=4q [答].

763132人目の素数さん:03/10/24 02:10
>>762
合っています
764759:03/10/24 05:49
>>754
この問題、かなり面白い結果になりそうだ。特にkが奇数の時。

面白い問題スレ行きだろこりゃ。
765759:03/10/24 06:39
kが5以外の奇数のとき、でけた。

mod 10 で考えてイパーン性を失わない。0〜9の数字からなるランダムな列を考え、
先頭から1つ取ったとき、それがいくつかによって場合を分けて漸化式を立てる。

たとえば全部かけて1になるケースは、先頭の1つが
・1だった場合、残り全部の積が1
・3だった場合、残り全部の積が7
・7だった場合、残り全部の積が3
・9だった場合、残り全部の積が9
となればよい。これらを3,7,9についても同様に考えることにより、
          先頭が1  3   7    9
P(n+1,1) = (1/10) (P(n,1)+P(n,7)+P(n,3)+P(n,9))
P(n+1,3) = (1/10) (P(n,3)+P(n,1)+P(n,9)+P(n,7))
P(n+1,7) = (1/10) (P(n,7)+P(n,9)+P(n,1)+P(n,3))
P(n+1,9) = (1/10) (P(n,9)+P(n,3)+P(n,7)+P(n,1))

また仮定より P(1,k)=1/10

これらを解いて(対称形なのであっさり解ける)、
P(n,1)=P(n,3)=P(n,7)=P(n,9)= (4^(n-1)) / 10^n
766759:03/10/24 06:46
なんか位数10の巡回群の構造がモロに出てきてるので、
群の知識を使えばもっと簡単に解けるのかも?と思う。
俺には無理。
767759:03/10/24 10:08
P(n,0) = (10^n+4^n-8^n-5^n) / 10^n
P(n,1) = P(n,3) = P(n,7) = P(n,9) = 4^(n-1) / 10^n
P(n,5) = (5^n-4^n) / 10^n
P(n,2) = P(n,4) = P(n,6) = P(n,8) = (2^n - 1)4^(n-1) / 10^n

面白かったけど疲れた。計算ミスの嵐で地獄を見た。
>>754
確かに面白そうな問題だけど、題意不明瞭。
n個の中に重複あり?正負の記載がないのはわざと?(それも問題のうち??)
平凡な工房(そうでないヤシごめん)相手なら、ちょっと問題を捏造してこんな感じ。

Nを正の整数とする。
N以下のn個の正の(もしくは、負でない)整数を掛け合わせたときの下一桁がkになる
確率をP(N, k,n)とする。
lim[N->∞] P(N, k,n) を求めよ。

と言いつつ、これを解こうとして挫折(w
>>759が前提にしていることは却下なので、不等式評価の分だけ難しそう。
不明瞭を引きずってしまった。

Nを正の整数とする。
N以下の正の(もしくは、負でない)整数からをn個(重複あり/なし)を任意に選び、これらを
掛け合わせたときに下一桁がkになる確率をP(N, k,n)とする。
lim[N->∞] P(N, k,n) を求めよ。

()部は出題者の気分次第(答えに影響はなさげだが)。
負まで考えると二重limだし、あんまり面白くないだろうからこんな感じにしてみた。
770132人目の素数さん:03/10/25 00:04
>整数からをn個(重複あり/なし)を任意に選び
えーと、どの数も等確率で選ぶんだよね?
>>740
10行目「さらに〜」の式はどうやって示すの?

>>756
出しっぱなしな問題多いよな…。
772769:03/10/25 00:22
>>770
>>整数からをn個(重複あり/なし)を任意に選び
>えーと、どの数も等確率で選ぶんだよね?
そうだよ。
ただ、引用するなら

>N以下の正の(もしくは、負でない)整数からをn個(重複あり/なし)を任意に選び
こうして欲しかったけど。

これで等確率で選ぶことと同値と伝わったかどうかは、文才が無いから微妙かも。
773769:03/10/25 00:35
落ち着いて読んでみた。
全然日本語になっていないよ、ごめんな ∧||∧
こう言いたかった。

Nを正の整数とする。
N以下の正の(もしくは、負でない)整数からn個(重複あり/なし)を任意に選び、これらを
掛け合わせたときに下一桁がkになる確率をP(N, k,n)とする。
lim[N->∞] P(N, k,n) を求めよ。
>>771
まずm≡n (mod 6)⇒a(m)≡a(n) (mod 4)
は実際やってみりゃわかる。
n             │0 1 2 3 4 5 6 7 ・・・
─────────┼────────
an(を4でわった余り) │0 1 1 2 3 1 0 1 ・・・
m≡n (mod 100)⇒a(m)≡a(n) (mod 25)もa(n)を25でわったあまりを102個ならべてみればわかるけど
それがいやなら以下のようにしてもしめせる。
 
α=(1-√5)/2、β=(1+√5)/2とおく。a(n)=(β^n-α^n)/(β-α)=(β^n-α^n)/√5
R=Z[α]とおいて各r∈Rにたいしv(r)=max{n|r/(√5)^n∈R}とおく。これはRの加法付値になる。
m-n∈100Nと仮定する。このとき(√5)(a(m)-a(n))∈(β^100-1)R+(α^100-1)R。
まずv(α^100-1)≧5をしめす。
(2^100)(α^100-1)
=C[100,1]√5・2^99+C[100,2]5・2^98+C[100,3]5√5・2^97+C[100,4]25・2^96+C[100,1]25√5・2^95+・・・
25|C[100,1]、25|C[100,2]、25|C[100,3]、25|C[100,4]、から前の4項はvの値が5以上。
それ以降の項のvのあたいはあきらかに5以上。よってv((2^100)(α^100-1))≧5。
一方でv(2^100)=0 (∵2RはRの素イデアル)
∴v(α^100-1)≧5。示せた。
同様にしてv(β^100-1)≧5。∴v(√5)(a(m)-a(n))≧5。
∴v(a(m)-a(n))≧4。つまりa(m)-a(n)=25rとなるr∈Rが存在するが25R∩Z=25Zゆえa(m)-a(n)∈25Z。
 
以上からm≡n (mod 300)⇒a(m)≡a(n) (mod 100)
775132人目の素数さん:03/10/25 23:07
f(x)=ax^2+bx+cとf(x)=dx+kの共有点の個数を調べよ
>>756
おれ出題者じゃないけど1.の(2)なら
α(n)は方程式x^2+1/x=nの最小解でn>1に対しそれはx<-1の領域にある。
この領域においてx^2-1<x^2+1/x<x^2なので
x^2-1=nの最小解x=-√(n+1)はα(n)より小さく
x^2=nの最小解x=-√(n)はα(n)より大きい。つまり
-√(n+1)<α(n)<-√(n)
よってlim[n→∞]α(n)/n^(t)が0でない定数に収束する⇔t=1/2
だと思う。
AとBが戦ったときAが勝つ確率は常にp(1/2<p<1)であり、引き分けはないものとする。
AがBより先にn連勝する確率をP(n)とするとき、P(n)<P(n+1)を証明せよ。
ただしAとBは十分回戦うものとする。
>>775
さすがにそれは簡単すぎるだろ。
>>758
これどうやって解くの?全然方針たたないんだけど
>>779
大体方針立ててみた。

∫[0,a]f(x)dx=Aとおく。
∫[x_(i-1),x_i]f(x)=A/n (i=1,.,n-1)
よりΣ=Σ[i=1,n-1]として
Σf(x_i)/n=(1/A)Σf(x_i)∫[x_(i-1),x_i]f(x)dx
=(1/A)Σ∫[x_(i-1),x_i]f(x_i)f(x)dx
ここで|f^2(x)-f(x_i)f(x)|
<=(max[x_(i-1),x_i]|f(x))×|f(x)-f(x_i)|
<=(max[0,a]|f(x)|)|×|f(x)-f(x_i)|
=M×(max[x_(i-1),x_i]|f(x)-f(x_i)|)
ここで|x_(i-1)-x_i|->0(一様、n->∞)を使うと(*)
∫[x_(i-1),x_i]f(x_i)f(x)dx->∫[x_(i-1),x_i]{f(x)}^2dx(iに関して一様)
Σをとって
lim(n->∞)Σf(x_i)/n=∫{f(x)}^2dx/A
(*)の部分はもしかして単にf(x)が連続だけじゃ成り立たないかも知れない。

もちろnこのままの解答じゃ(*)の部分の証明は大学入試レベルじゃないし、
別の証明も無さそうだからこの問題は入試問題としては全く不適切だと思う。
>>777
これ事象が無限にでるから入試問題にはでないのでは?
―解答―
まず非負整数tを固定する。
今Bがb勝し(0≦a≦n-1)次にBがb1勝、がb1勝、・・・、Bがbt勝、Aがbt勝、・・・、
そして最後にAがn勝する(1≦ai,bi≦n-1)確率は
q^a・p^(a1)・q^(b1)・・・p^(at)・q^(bt)・p^n
よって0≦a≦n-1、1≦ai,bi≦n-1の範囲でこれらを足し合わせると
(1+q+・・・+q^(n-1))・(p+・・・+p^(n-1))・・・(q+・・・+q^(n-1))・p^n
=((1-q^n)/(1-q))・(1-p^(n-1))^t・(1-p^(n-1))^t・p^n
これをt=0から∞までたしあわせるとAが先にn連勝する確率は
(p^(n-1)-q・(pq)^(n-1))/(p^(n-1)+q^(n-1)-(pq)^(n-1))
関数f(x)をf(x)=(p^x-q・(pq)^x)/(p^x+q^x-(pq)^x)とおく。これがx≧0の範囲で単調増大
であることをしめせばよい。さらにy=(q^x-p・(pq)^x)/(p^x-q・(pq)^x)とおけば
f(x)=1/(1+y)、y≧0。1/(1+y)はy≧0において単調減少なのでyがx≧0の範囲で
単調減少であることをしめせばよい。y=(p/q)(((1/p)^(x+1)-1)/((1/q)^(x+1)-1))
であるのでz=(1/q)^(x+1)とおくとy=(q/p)(z^(logp/logq)-1)/(z-1)。
0<logp/logq<1であるのでyはzに関して単調減少。さらに1/q>1なのでzはxに関して
単調増大。∴yはxに関して単調減少。∴fはxに関して単調増大。□
(x-a)(x-b)(x-c)・・・・・(x-z)を計算せよ
>>782
0
784758:03/10/26 21:38
>>780
(*)を解消するには、逆関数をとり、変換して解き直すか、中間値の定理を利用して解くかすれば良いです。
(私も最初考えたとき、悩んでしまいました)

易しくはないかもしれませんが、C(orD)****ぐらいでは?
>>758
この問題高校の知識でとけると思う。
 
F(x)=∫[0,x]f(t)dtとおく。F(a)=bとおく。f(t)>0よりF(x)は単調増大。
よって逆関数G(y):[0,b]→[0,a]がとれる。このとき
I=lim[n→∞]Σ[0〜n-1]f(x_i)/n
=lim[n→∞]Σ[k:0〜n-1]f(G(bk/n))/n
=∫[0,1]f(G(bt)dt
=(1/b)∫[0,b]f(G(u))du
ここでG(u)=vと置換するとu=F(v)よりdu=F'(u)du=f(u)du、u:0〜aだから
I=(1/b)∫[0,a]f(v)^2dv
=∫[0〜a]{f(x)}^2 dx/∫[0〜a]f(x) dx
786758:03/10/27 00:23
>>785
正解です。ただ受験生がこの手の問題で逆関数という発想が出来るかというのは別問題でしょうか・・・
平均値の定理(上では血迷って中間値と書いていました)(汗)は、n等分→平均と読みとれれば比較的に楽になります。
連続関数って概念が高校数学には無いから、具体的な関数を与えた方がいいね。
y=(sinx)^2って奴だと今年の東大後期に少し似てるのがある。
788132人目の素数さん:03/10/27 05:20
>>774
レスありがとう。なるほどそういうやり方があるのかあ。
2つだけ教えてください。
r,s∈Rとしてv(rs)=v(r)+v(s)が成り立つのはどう示すの?
あと14行目からの二項展開の計算合ってる?α^100-1<0だよね。

教えて君かつややスレ違いすいません。
>>780
f(t)>0であることを使っていない。
F(x)=∫[0,x]f(y)dy,F(a)=Aとおく.F(x)は単調増加
F(xi)=Ai/n(i=1..n)
A/n=F(x(i+1))-F(x(i))で左辺でn->∞で0に近づくことと、F(x)が単調増加
であることを合わせると|x(i+1)-x(i)|->0
(逆関数の連続性を使っているが、この辺りは高校範囲外だと思う)
Σ[i:1〜n]f(xi)/n=ΣF'(xi)(F(x(i+1))-F(xi))
=ΣF'(xi)F'(θ(i))(x(i+1)-x(i)) x(i)<θ(i)<x(i+1)(∵平均値の定理)
x(i)<=x<=x(i+1)に対し
f(x)>0だから
min[x(i),x(i+1)]f^2(x)<=f(xi)f(θ(i))<=max[x(i),x(i+1)]f^2(x)より
Σmin[x(i),x(i+1)]f^2(x)(x(i+1)-x(i))<=ΣF'(xi)F'(θ(i))(x(i+1)-x(i))<=
Σmax[x(i),x(i+1)]f^2(x)(x(i+1)-x(i))
左辺と右辺は積分の定義よりそれぞれ∫[0,a]f^2(x)dxに近づいていく。
∴AΣf(xi)/n=∫[0,a]f^2(x)dx
790789:03/10/27 13:28
lim(n->∞)AΣf(xi)/n=∫[0,a]f^2(x)dx の間違いです。
あとmin[x(i),x[i+1]f^2(x)(x(i+1)-x(i))
とかは、{min[x(i),x[i+1]f^2(x)}(x(i+1)-x(i))
とか読み替えて下さい。
お前馬鹿じゃねーの?とかいわれそうですけど・・・
758さんの問題、Σ[0〜n-1]f(x_i)/n は
Σ[i=0〜n-1]f(x_i)/nってことでいいんですよね?
i=1からなのにi=0〜n-1でも・・・いいんですか?
x_0
x_0=0ってことですかね?
なんかすっごい馬鹿っぽい書き込みになっちゃいましたね・・・
スレ汚してスイマセン。マジで。
自分を愚かと表現するのに馬鹿と気安く使うな。馬や鹿に失礼也。
馬さん鹿さんごめんなさい
>>788
>r,s∈Rとしてv(rs)=v(r)+v(s)が成り立つのはどう示すの?
 
素イデアル分解の一意性をつかう。以下<x>=(xで生成されるイデアル)とするとき
v(r)=m⇔∃I:Rのイデアル <x>=<√5>^m・I I+<√5>=R
証明はさほど難しくない。
 
v(r)=m、v(s)=n、p=(√5)Rとするとき
(r)=(rで生成される素イデアル)=p^m・I、p+I=R
(s)=(sで生成される素イデアル)=p^n・J、p+J=R
となるI,Jが存在する。このとき
(rs)=(rsで生成される素イデアル)=p^(m+n)・IJ、p+IJ=R
 
>あと14行目からの二項展開の計算合ってる?α^100-1<0だよね。
 
これはまちがってる。スマ。ただしくは
((2α)^100-1)
=C[100,1](-√5)・2^99+C[100,2]5・2^98+C[100,3](-5√5)・2^97+C[100,4]25・2^96+C[100,1](-25√5)・2^95+・・・
 
ちなみにもっと初等的な方法もあるだろうと思う。しかしこの問題についてはa(n)を
25でわった余りを102個ならべるという究極の初等的方法があるのでわからなければ
そっちでやったほうが早いと思う。
つーかあんた頭良すぎ
俺工房だからイデアルとか分かんないけど
すげえ頭良さそうに見える

25でわった余りを102個ならべるという究極の初等的方法があるので

な、、、なにこれ
799132人目の素数さん:03/10/28 22:25
次の命題の真偽を判定せよ。
『aを実数として、どのような自然数nに対してもa^nが自然数にならないとき、
任意の正数εに対してa^m - [a^m] < ε となる自然数mが存在する』

ただし、[x]はxを超えない最大の整数を表す。
出題形式をもってわざとらしく難しくするのは、試験としていかがなものか
f(1)=f(2)=1 、 f(n+2)=f(n+1)+f(n)
α=(1+√5)/2
と定義する。

自然数nに対して
f(2n+1) = Σ[k=1,n] [ (α^n)*f(n) + 1/2 ]   ← Σの中ガウス記号ね。
が成立する事を示せ。

ただし、[x]はxを超えない最大の自然数であるとする。
数オリ系になってきた。
>>801
>f(2n+1) = Σ[k=1,n] [ (α^n)*f(n) + 1/2 ]   ← Σの中ガウス記号ね。

じゃなくて

f(2n+1) = Σ[k=1,n] [ (α^k)*f(k) + 1/2 ]   ← Σの中ガウス記号ね。

の間違いだった。
>>801
できた。
α=(1+√5)/2、β=(1-√5)/2とおく。
f(n)=(α^n-β^n)/√5
一方k≧2にたいし0<β^(2k)/√5+1/2-(-1)^k/√5<1であるから
[α^(2k)/√5+1/2-(-1)^k/√5]=(α^n-β^n)/√5+[β^(2k)/√5+1/2-(-1)^k/√5]=f(2k)
k=1のときは1<β^(2k)/√5+1/2-(-1)^k/√5<2であるから
[α^(2k)/√5+1/2-(-1)^k/√5]=(α^n-β^n)/√5+[β^(2k)/√5+1/2-(-1)^k/√5]=f(2k)+1
よって
Σ[k=1,n] [ (α^k)*f(k) + 1/2 ]
=Σ[k=1,n] [ (α^ (2k)-(-1)^(2k))/√5+ 1/2 ]
=1+納k=,n]f(2k)
よって1+納k=,n]f(2k)=f(2n+1)を示せばよいがこれは帰納法から容易ゆえry
>>799
偽である。
α=2+√2、β=2-√2とおく。α、βはt^2-4t+2=0の解である。t[k]=α^k+β^kとおく。
漸化式t[k]=4t[k-1]-2t[k-2]、t[0]=2、t[1]=4よりt[k]はすべて整数。
よって[α^k]=α^k+β^k+[-β^k]=α^k+β^k-1でありよってα^k-[α^k]=1-β^k。
これは√2-1より小さくなくことはない。
面積1の正方形を正三角形で完全に覆い尽くすとき、正三角形の面積の最小値を求めよ。



・・・だめ?
面積1の楕円にしろ
nが6で割って3余る自然数の時、
n^2 + 2^n
は素数である事を示せ。

また、上の命題の逆は成り立つか?
27^2+2^27 = 134218457 = 73 × 521 × 3529
すまん。。。逆だった。
n^2 + 2^n が素数の時、nは6で割って3余る事を示せ。
また、上の命題の逆は成り立つか?


っていう問題だった。
逆が成り立たないのは、既に証明済みという事で・・・
811132人目の素数さん:03/10/29 08:03
n=1
>>798
これがフィボナッチ数列を25でわったあまりを第0項〜第101項までならべたもの。
100項目、101項目が0項目、1項目と等しくなるので以下周期100で繰り返しになる。
 
0 1 1 2 3 5 8 13 21 9
5 14 19 8 2 10 12 22 9 6
15 21 11 7 18 0 18 18 11 4
15 19 9 3 12 15 2 17 19 11
5 16 21 12 8 20 3 23 1 24
0 24 24 23 22 20 17 12 4 16
20 11 6 17 23 15 13 3 16 19
10 4 14 18 7 0 7 7 14 21
10 6 16 22 13 10 23 8 6 14
20 9 4 13 17 5 22 2 24 1
0 1
813132人目の素数さん:03/10/31 05:40
814132人目の素数さん:03/11/01 13:39
5(x^2)-3(y^2)=1
整数解があるなら例を示し、ないなら証明しなさい。

ってかこれってもしかして入試よりレベルひくい?
815132人目の素数さん:03/11/01 14:11
全然ひくい。
東大入試なら、ちと易しすぎるな。
整数問題に慣れていたら解く方針もすぐ立つし。
ナァーンダ・・・(´・Д・`)
>>814
略解だけど・・・
整数解が存在するので有れば、x^2≡2(mod3)で有ることが必要。
しかし、m∈Zとすると,(3m)^2≡0(mod3)  (3m+1)^2≡1(mod3) (3m+2)^2≡1(mod3)であるので
x^2≡2(mod3)となる整数xは存在しない。
以上より与式は整数解を持たない. Q.E.D.

こんな感じで終わってしまうから、もう少し煩雑にすれば解きにくくなるかも。
(ただ悪問になりかねないけど)
はぁ?
5x^2=3y^2+1に整数解が無いなんて...
mod 5で考えれば3y^2+1=0(mod5).
(y+5k)^2-y^2=0 mod 5だから
y=0,1,2,3,4について3y^2+1≠0 mod 5だけ示せば十分
それぞれ1,4,3,3,4だから全然だめじゃん。
819より818の方がエレガント。

mod 3 で -1 は平方非剰余、の一言で瞬殺したいところ。
(まあ、実際の答案は 818 のように丁寧に書くにしても)
見かけ倒しの問題を一つ。
実数α、β、γ、δが次の三つの条件を全て満たすとき、その値を求めよ。

・0<α<β<γ<δ
・この4つの実数は全て 4(x^4) - a(x^3) + b(x^2) - cx + 5 = 0 の解である
・(α/2) + (β/4) + (γ/5) + (δ/8) = 1 を満たす。


まぁ、答えは・・・
正の実数だから、微分して・・・とかやってると求められないから意外と難しいかも。。。
んなわけないかな。
解と係数の関係じゃ上手くいかないようだ
うまくいくっちゅうの。てか閉条件がαβγδ=5/4、(α/2) + (β/4) + (γ/5) + (δ/8) = 1
しかないのに値がきまるってことからだいたい推測できる。
>>823
あ、ホントだ。4忘れてた
やっぱ、瞬殺されたか。
相加平均・相乗平均だっけ?あれ関係の不等式を使うと瞬札です。
ちなみに解法は?
821の解法(のつもり)です。
αβγδ=5/4・・・(A)
(α/2) + (β/4) + (γ/5) + (δ/8) = 1・・・(B)
このとき、0<α<β<γ<δより(B)に(A)を代入したものの左辺について相加相乗平均の大小を考えると、
(γ=5/4αβδとして代入)
(Bの左辺)=4・(α/2*β/4*1/4αβδ*δ/8)^1/4≧1      
となり(B)は、上式の等号成立時にのみ成立する。
よって、 α/2=β/4=γ/5=δ/8=1/4より(α,β.γ.δ)=(1/2,1,5/4,2)となる。


おまえらきもいな
829132人目の素数さん:03/11/04 01:33
あげ
830132人目の素数さん:03/11/07 15:39
みなさんネタぎれですか?
α>0, β>0, α+β<π のとき
(sinα-sinβ)^2+{sin(α+β)}^2<2(sinα+sinβ)sin(α+β)
が成り立つことを示せ。
832132人目の素数さん:03/11/08 00:03
xyz空間においてyz平面上の双曲線y^2-z^2/2=1をz軸のまわりに
1回転してできる回転体Qと2平面z=y+1及びz=y−1によって囲まれる
立体図形をKとする。
(1)回転体Q上の点をP(x、y、z)とする時、x^2+y^2を
zで表せ。
(2)平面z=y+t(−1≦t≦1)をαとし、回転体Qの方程式と平面αの方程式から
zを消去することによって、平面αによるKの切り口のxy平面上への正射影の周の方程式
および正射影の面積を求めよ。
(3)平面αによるKの切り口の面積S(t)を求めよ。
(4)Kの体積Vを求めよ。

>>831
それどっかの過去問じゃないかい?
それこそ東大の
>>833
え、まじ? 普通に自作のつもりなんだけど。
835132人目の素数さん:03/11/08 01:23
Y=(logX)^(logX)^(logX)
をXで微分せよ。
>>835
それぐらい根性でやれ
0≦x≦1においてL:y=x,C:y=x2,点P(t,t)を考える。(0<t<1)
この点Pからx軸,y軸と平行な直線をひきCとの交点をとり、次にこの交点からy軸に平行な直線をひき、さらにLとの交点をとる。
この作業を限りなく行い、[それらの交点]と[その交点ともっとも近い交点]を結んだ階段状の線とLによって囲まれる面積は、どのような値に近づいていくか。
>x軸,y軸と平行な直線をひき
ムリポ
x-y座標平面上に凸図形がある。この凸図形を
(cosθ,sinθ)に平行な二本の直線で挟み込み、その時の距離をf(θ)と置く。
このとき、∫[0,π] f(θ)dθを求めよ。

必要なら、凸図形の周の長さをL、面積をSとして用いて良い。
---

言うまでもなく、面積は使わない。
840132人目の素数さん:03/11/08 02:14
平面上にあるどのような形の鈍角三角形または直角三角形も、有限個の線分を用いて
六つ以下の鋭角三角形に分割する事はできない事を示せ。

また、平面上にあるどのような形の鈍角三角形または直角三角形も、有限個の線分を用いて
七つの鋭角三角形に分割に分割する事ができる事を示せ。


ここで、鈍角三角形とは三角形の内角のうち一つが90°を超える物を言い。
直角三角形とは三角形の内角のうち一つが90°になる物を言う。
鋭角三角形とは、鈍角三角形でも直角三角形でもない三角形である。
>>840
数学の部屋になかった?
842132人目の素数さん:03/11/08 02:23
数学セミナーの問題だよ。 っていうか、ピーターフランクルの幾何学の本にも載ってる。
843132人目の素数さん:03/11/08 02:31
>>839
ちょい、日本語がおかしいので訂正。

x-y座標平面上に凸図形がある。この凸図形を
(cosθ,sinθ)に平行な二本の直線で挟み込み、その時の二直線の距離をf(θ)と置く。
このとき、∫[0,π] f(θ)dθを求めよ。

必要なら、凸図形の周の長さをL、面積をSとして用いて良い。
うぅむ。次の入試までもたないか
>(cosθ,sinθ)に平行な
「(0,0)と(cosθ,sinθ)を結んだ直線に平行な」
って意味だよね?
>>845
そうだね。ごめん。
847132人目の素数さん:03/11/08 02:42
2004系統の問題作ってみようかな
三角形ABCの内部に点Pを置く。
PからBCに下ろした垂線の足をD。CAに下ろした垂線の足をE。ABに下ろした垂線の足をFとする。
このとき、不等式
PA+PB+PC≧2(PD+PE+PF)
を示し、等号成立条件を求めよ。
f(0)=f(1)=0。f(x)>0 (0<x<1の時)
f(x)を二階微分可能な関数とするとき、
∫[0,1] | f''(x)/f(x) | dx > 4
を示せ



  過去問にあるかも・・・
>>849
変分法の問題?
>>849
その問題知ってる。
変分法は使わないで解けるやね。
たくさん出たんだから、誰か解いてくれ。。。

俺は一問も解けない。
853132人目の素数さん:03/11/08 19:22
>>849
答えかヒントくれあげ
>>853
ヒント
1.条件を満たす関数は、0<x<1の範囲で最大値を持つ。
2.Webのどこかに同じ問題がある。しかも日本語。
>>854
降参。答え教えてたも。
856132人目の素数さん:03/11/08 20:17
あげ
こういう問題はノーヒントで是非自分で解答を見つけることね。
>>857
できたひと?出題者?
>>849の問題、
f(0)=f(1)=0なのに、[0,1]でf''(x)/f(x)を考えても良いんでしょーうか。

積分のことをしっかり理解してないことが見え見えかも(´・ω・`)
860132人目の素数さん:03/11/08 20:42
広義リーマン積分でいけ
>>859
そうそう。だからこの問題スレちがいといえばスレちがい。
これ積分値有限確定値になる例すらおもいつかないんだけど。
もしかしてかならず積分値∞とかになるのかな?
862851:03/11/08 20:50
849は数学発想ゼミナールって言う本の中で取り上げられているよ。
この本ほとんどの問題が解答がついてないんだけど、
この問題は例題として載っていて解答もちゃんとついている。
863132人目の素数さん:03/11/08 20:51
んじゃ、a,bを0<a<b<1として、どのようにa,bを選んでも
∫[a,b] f(x)dx > 4
が成立する事を示せ。

ほい。これなら、問題あるまい。
>>862
もう降参。たのむ。答えおしえて。
>>863
それではますます文句でると思うぞ。
>>865
にゃぜ? f(x)は上のような条件を満たす問題だよ。
>んじゃ、a,bを0<a<b<1として、どのようにa,bを選んでも
>∫[a,b] f(x)dx > 4
 
だってこれじゃ任意の0<a<b<1で所与の不等式が成立するとよめてしまうけど
b→a+0のとき左辺→0だとおもうけど。
すまん。ぼけてた。
869132人目の素数さん:03/11/08 20:57
>>863は忘れて・・・おおぼけ
まぁ、>>849はどうでもいい
>>848>>839の解答お願い
871132人目の素数さん:03/11/08 21:17
>>849
f(0)=f(1)=0 f(x)>0 (0<x<1)より、f(x)はx=Aにおいて最大値Bをとる。
平均値の定理から、
(f(A)-f(0))/(A-0)=f'(C)  ただし、0<A<C
(f(1)-f(A))/(1-A)=f'(D)  ただし、A<D<1
が成立する。これを整理して
B/A=f'(C) -B/(1-A)=f'(D)

 ∫[0,1] |f''(x)/f(x)|dx
>∫[C,D] |f''(x)/f(x)|dx
≧∫[C,D] |f''(x)|dx /B
=( f'(D)-f'(C) )/B
=1/(A(1-A))

ほい。
すばらしい・・・
>>871
それで示せてるの?
A(1-A)で??
874132人目の素数さん:03/11/08 22:07
お前には無理だから心配いらん
875132人目の素数さん:03/11/08 22:09
>>871の続き
0<A<1 より、
0<1-A<1が成立。

相加平均・相乗平均の不等式より
1=A+1-A≧2√(A(1-A))
両辺二乗して、整理すれば、
1/(A(1-A))≧4

だが何か?
>>873
単純な二次関数の逆数
∫[C,D] |g'(x)|dx
=g(D)-g(C)
????????
lim[n->∞] Σ[k=0,n] 1/(k!)
が収束する事を示せ。
また、その収束値をαとするときαは無理数である事を示せ。
879132人目の素数さん:03/11/08 22:25
お前には無理だから心配いらん
下からの評価がほしいだけなんだから
≧∫[C,D] |f''(x)|dx /B
≧|∫[C,D] f''(x)dx |/B
=|f'(D)-f'(C) |/B
と修正すればいいのでは?
881132人目の素数さん:03/11/08 22:27
あぁ、分かったよ>>877
≧∫[C,D] |f''(x)|dx /B
≧∫[C,D] f''(x)dx /B
=( f'(D)-f'(C) )/B

でいいんだろ
882132人目の素数さん:03/11/08 22:29
よく考えたら
eが収束する証明って
1+1/2+1/4+1/8+・・・が収束するから自明じゃん
有界で単調な数列だから・・
無理数はまあ普通に教科書に載ってるwww
wwww
wwwwwwwwwwwwwwwwwwwwwwwwwwwwwwwwwwwwwwwwwwwwwwwwwwwwwwwwww

wwwwwwwwww
wwwwwwwwwwwwwwwwwwwwwwwwwwwwwwwwwwwwwwwwwwwwwwwwwwwwwwwwww

wwwwwwwwww
883132人目の素数さん:03/11/08 22:31
>>878
有名まんだい
解析がいごんに出てる
884132人目の素数さん:03/11/08 22:32
「eが収束する」って何だよ
>>882
証明は載っていない。
wの連打(笑
887132人目の素数さん:03/11/08 22:58
しかも>>878って別にeについて直接聞いてる問題じゃないんだよな。
Σ[n=0,∞] 1/(n!)
について聞いてるんだよね。もちろん、この値はeだけど、高校生なら知らなくてもおかしくない。
出題としては、なんら不思議のない形式だと思う。
888132人目の素数さん:03/11/08 23:04
[x]をxを超えない最大の整数とする。

Σ[k=1,n^2] ( [(k^2)/(n^2)] + [n*√(k)] )

を求めなさい。



あぁ、出題溜まってきたな。
>>878
簡単な問題の解答はささと書いておこう。(背理法を使うので入試問題には不向きかと...)
α=lim[n→∞]Σ[k=0,n]1/(k!)が存在すること
Sn=Σ[k=0,n]1/(k!)とおく。
n>m>1に対しSn-Sm-1=1/m!+1/(m+1)!+....+1/n!
<1/m!(1+1/(m+1)+1/{(m+1)(m+2)}+1/{(m+1)(m+2)(m+3)}+.....
<1/m!(1+1/m+1/m^2+....)
=1/m!(m/(m-1)<1/(m-1)!→0(m→∞)より{Sn}は収束する。
αが有理数とすると適当な自然数mがあってl(自然数)>mに対し
l!α=lim[n→∞]Σ[k=0,n](l!)/(k!)は自然数
が成立すべき。しかしlを十分大きく取るとこれは成り立たない。
実際l!α=lim[n→∞]{Σ[k=0,l]l!/k!+Σ[k=l+1,n]l!/k!}でΣ[k=0,l]l!/l!は自然数だが
1/(l+1)≦lim[n→∞]Σ[k=l+1,n]l!/k!<1/(l+1){1+1/(l+2)+1/{(l+2)(l+3)}+...}<1/l
だから自然数ではなく、結局l!αも自然数ではない。
原点Oのxy平面上に点A(a,0)点B(b,0)点M((a+b)/2,0)そして直線L:y=mxがある。
(0<a<b、m≧0)φ
この時、点P(p,mp)(p>0)をとり、∠APB=θとする。次の問に答えよ。
(1)θが最大になる時,pの値をa,b,mで表せ。
以下の問いでのpは(1)で求めた値を適用する。
(2)∠OPMがπ/2になる時の∠POM=φとする。tanφをa,bで表せ。
(3)mを0〜tanφまで動かしたときに線分MPが通過する領域の面積Sをa,b,φで表せ。
   またlim[b→a+0]S/(b-a)^nが発散,又は0に収束しないためのnの値を求め、その時の極限値を求めよ。

自作。かなり簡単になってしまったり、不備があったりするかもしれませんが、講評いただければ幸いです。
891890:03/11/09 15:45
↑直線Lはほとんど関係ありません(汗)
ヒント
>>839  多角形で考える
>>840  鈍角または直角三角形の最大の内角をAとして、Aを分割する線分は三角形のどこで止まるでしょう?
>>888  f(x)=(x^2)/(n^2)の逆関数ってなんだと思う?
893132人目の素数さん:03/11/10 00:01
あえげ
>>888は試験に出てもおかしくなさげ
ノーヒントだとC☆☆、ヒント付きならA☆辺りかな
>>894
俺は解けんな
894解法教えて
>>894 解けたの?
>>888は普通に[××]=aとなる××の数数えるんじゃないの?
>>897
意味分からない。[××]って何さ?

x-y平面を考えて
普通に0≦x,y≦n^2内の格子点とy=x^2の上の格子点を数えるんじゃないのか?
>>898
答えお願いします。
>>888  うーん。解いてくれた人の数が少ない。簡単だと思ったんだが

x-y座標平面で考える。
関数f(x)=(x^2)/(n^2)を考える。この逆関数はg(x)=n*√(x)である。
Σ[k=1,n^2] ( [(k^2)/(n^2)] + [n*√(k)] )
のシグマの中身に注目する。
[(k^2)/(n^2)] + [n*√(k)]
前半部分は、x=k、0≦y≦f(k)の範囲にある格子点の総数である。
後半部分は、x=k、0≦y≦g(k)の範囲にある格子点の総数である。
結局、
Σ[k=1,n^2] [(k^2)/(n^2)]
は0≦y≦f(x) 0≦x≦n^2、の範囲にある格子点の総数である。同じように
Σ[k=1,n^2] [n*√(k)]
は0≦y≦g(x) 0≦x≦n^2、の範囲にある格子点の総数である。
ここで、gはfの逆関数なので結局
Σ[k=1,n^2] ( [(k^2)/(n^2)] + [n*√(k)] )
は0≦x,y≦n^2内部の格子点とy=f(x)上の格子点をあわせた物になる。

これから先は簡単じゃね?
間違った

[(k^2)/(n^2)] + [n*√(k)]
前半部分は、x=k、1≦y≦f(k)の範囲にある格子点の総数である。
後半部分は、x=k、1≦y≦g(k)の範囲にある格子点の総数である。
結局、
Σ[k=1,n^2] [(k^2)/(n^2)]
は1≦y≦f(x) 1≦x≦n^2、の範囲にある格子点の総数である。同じように
Σ[k=1,n^2] [n*√(k)]
は1≦y≦g(x) 1≦x≦n^2、の範囲にある格子点の総数である。
ここで、gはfの逆関数なので結局
Σ[k=1,n^2] ( [(k^2)/(n^2)] + [n*√(k)] )
は1≦x,y≦n^2内部の格子点とy=f(x) (1≦x≦n^2) 上の格子点をあわせた物になる。
902897:03/11/10 01:07
オレは[k^2/n^2]=uとなるuの数をa(u)としたとき前の項=盃a(u)になった。
でa(u)を計算すると0≦u≦n^2-1についてa(u)=[n√(u+1)]-[n√(u+1)]±b(n)
b(n)=1 (uが平方数のとき) -1 (u+1が平方数のとき) 0 (それ以外のとき)
になった。そこで納u=1,n^2-1]u[n√(u+1)]-[n√(u)]を計算すると
納u=1,n^2-1]u[n√(u+1)]-[n√(u)]
=納u=1,n^2-1]納v=u,n^2-1][n√(v+1)]-[n√(v)]
=納u=1,n^2-1](n^2-[n√u])
=納u=1,n^2](n^2-[n√u])
=n^4-(所与の式の第2項)
となる。巴(n)は簡単に計算できるので結局もとめる和はn^4+納u=1,n^2-1]ub(u)+n^2と
なった。盃b(u)=納v=1,n-1]v^2b(v^2)+納v=2,n](v^2-1)b(v^2-1)なので
さらっと計算できた。
ぐはぁ、ダメだ。思いっきり間違ってる。>>900-901は無視しろ。
と思ったらあってた。 ・・・何考えてたんだか。
結局答えは何にせよ。n^4 + n

さぁ、次行ってみよう。
>>900
出題者なの?なのに解答無視しろとは?
いや、>>900=904=出題者なので、察してくれ。
なんか勘違いしてた。

>>901はあってると思う。
以前、この方法で解けたと思ってて(実際に合ってるわけだが)
ついさっき見たら、>>901の続きを計算するとn^4+n^2になってしまった。


いや、なるわけ無いんだけどね、n^4+nが正解。>>901をきちんと計算すると
ちゃんとn^4+nになる。


n^4+n^2になってしまったのは俺がぼけてたから。
さて、出題者がいるうちに>>839を聞いておこう。
教えてください。
やっぱ簡単ですか?

(・・・他の奴は結構難しいなぁ)
このスレ・・なつか( ゚д゚)スィ…

>>910
pを変数とし,a,b,mを与えられた実数の定数として考える.
で,θの取りうる値が鋭角に限られるならば,最大値を与えるpはp=√{(ab)/(m^2+1)}かと。
もし,θが鈍角にもなりうるのならば,θの最大値は限りなく180度に近くなるので,
最大値が存在しないことになるというか・・。
たとえば,a=1,b=1000000000000,m=0.000000000000001 のときなんかはθは鈍角になるだろうし。
定数の与え方次第でいくらでもθは変化するので,各々に応じた場合わけが必要?
>>911
問題設定が微妙でしたね・・・
まず、a,bは定数、そしてpは変数、mは(1)で定数(として)、(2)ではある条件におけるmを、(3)ではmを動かしています。

つまりは、
(1)点Pを直線L上で動かして、求まったpでの点を定点として固定。
(2)それを動かしていって、tanφを求める。
(3)そして直線Lを傾き0〜tanφまで動かして面積を求め、極限値を求める。
といった手順です。
ただ直線Lとx軸が重なるときは、(1)で定めた点Pを求めるべき点とします。

座標系ではなく、図形的に明示した方がよかったですね・・・
>>890
(1)の場合、△ABPが直線y=mxに接した場合にθは最大になる
OP^2=OA*OBより、p^2(1+m^2)=ab p=√( ab/(1+m^2) )

(2) Pを通りy=mxに垂直な線分が点Mを通る事から
-mp=(-1/m)*( (a+b)/2 - p ) この式にpを代入して解けば
m=( b-a )/( 2√(ab) )
tanφ=m=( b-a )/( 2√(ab) )

(3)m>0の場合を考える。また、簡単化のためp(a,b,m)=√( ab/(1+m^2) )とする。
 MPの傾きは(-mp)/( (a+b)/2 - p )と表される。これをmの式と見なし微分すればその導関数は
( 4ab - 2(a+b)p(a,b,m) )/( (1+m^2)( a+b - 2p(a,b,m) )^2 )
となり、分母は明らかに正である。この分子をさらにmの関数と見なして再度微分すれば
2m(a+b)p(a,b,m)/(1+m^2)
これは常に正の値をとる。故に、( 4ab - 2(a+b)p(a,b,m) )は増加関数。さらにm=( b-a )/( 2√(ab) )のとき
( 4ab - 2(a+b)p(a,b,m) )=0より、結局、( 4ab - 2(a+b)p(a,b,m) )/( (1+m^2)( a+b - 2p(a,b,m) )^2 )<0 (m>0)が成立する。
よって、MPの傾きはmについて減少関数。
ここでMが定点である事に注目し、線分MPが通る点(x,y)について考える。
yを固定して、xの最大・最小を考えると、MPの傾きが減少関数である事から、
xはMPの傾きが最大の時に最小値をとり、最小の時に最大値をとる事が分かる。
また、yを固定したとき、xが最小値となる点は点Pの軌跡上の点である。
また、yを固定した時、xが最大値となる点はm=tanφとなるときの線分MP上の点である。

点Pの軌跡はmをパラメータとして、( √( ab/(1+m^2) ) , m√( ab/(1+m^2) ) )  (0<m<( b-a )/( 2√(ab) ))
√( ab/(1+m^2) )=xとすれば、( x , √(ab-x^2))   √(ab)<x<2ab/(a+b) である。
これは、半径√(ab)、中心点O、・・・・


arctan使わないかこれ? 間違ってる?・・・・
いや、こんな複雑な計算してたら、間違うだろうな俺。
つーわけで、間違いを教えてくれ。ほぼ確実に間違ってる。
ごめん、φを見落としてた。
続き
これは、半径√(ab)、中心点O、中心角φの扇形の弧になる。
結局、求める面積は△OPM(m=tanφの時) − 上の扇形の面積
=(b-a)(√(ab))/4 - abφ/2

次に、
S=(b-a)(√(ab))/4 - abφ/2
=( tanφ-φ )/2
より、 S/( (tanφ)^3 ) -> 1/6 (φ->0) が成立する。
tanφ=( b-a )/( 2√(ab) ) なので、n=3。
極限値は1/(24a^3)

ふー・・・間違ってる可能性高いなぁ。
>>914
S=ab(tanφ-φ)/2ですので、惜しいと言っちゃ惜しいですw
また、S/( (tanφ)^3 ) -> 1/6 (φ->0)の過程を明記した方がグッドです。
(n=3でのみ0以外で収束することも言及しなければなりませんし)
答えは・・・そちらの値をちょっと弄くったモノになります。
xy直交座標平面上の有理点を各頂点とする正多角形は
正方形以外に存在しないことを示せ。
917132人目の素数さん:03/11/11 22:17
>>839>>843
答えおしえてくれage
918132人目の素数さん:03/11/12 00:31
>>917
主題者がこない限り、俺も分からんが、凸多角形の場合に限って解答。

長さLのある線分αを、(0,0)と(cosθ,sinθ)を結ぶ線分に平行な二直線で挟んだ時、その二直線の距離をd(θ)とする。
線分αと二直線の成す角をφとすれば、明らかに、d(θ)=L*sinφが成立し、
∫[0,π] | d(θ) | dθ
=L*∫[0,π] | sinφ | dθ
=L*∫[a,a+π] | sinφ | dφ
=2L
が成立する。

このため、凸多面体を平行な二直線で挟むときを考えれば、結局その二直線間の距離f(θ)は
凸多面体を構成するn個の辺の長さをL(i)として、2f(θ)=Σ[i=1,n] L(i)sin(φ(i)) が成立する。
 ∫[0,π] 2f(θ)dθ
=Σ[i=1,n] L(i)*∫[0,π]sin(φ(i)) dθ
=2Σ[i=1,n] L(i)
=2L  (周の長さをLとする)

よって、
∫[0,π] f(θ)dθ=L

----
補足。 もし仮に、凸図形の周をn等分して、そのn等分した点を元に凸n角形を作り、
この凸n角形の極限(n->∞)を考える事で、一般凸図形に対して
∫[0,π] f(θ)dθ=L
を言っても良いのであれば、それで一般の図形に対する答えになる。
しかし・・・俺にはそこまでできない。
919132人目の素数さん:03/11/12 00:33
失敬
∫[0,π] | d(θ) | dθ
=L*∫[0,π] | sinφ | dθ
=L*∫[a,a+π] | sinφ | dφ
=2L

ではなく、

∫[0,π] d(θ) dθ
=L*∫[0,π] | sinφ | dθ
=L*∫[a,a+π] | sinφ | dφ
=2L

だな。
920132人目の素数さん:03/11/12 00:33
また間違えた。

 ∫[0,π] 2f(θ)dθ
=Σ[i=1,n] L(i)*∫[0,π]sin(φ(i)) dθ
=2Σ[i=1,n] L(i)
=2L  (周の長さをLとする)

ではなく

 ∫[0,π] 2f(θ)dθ
=Σ[i=1,n] L(i)*∫[0,π] | sin(φ(i)) | dθ
=2Σ[i=1,n] L(i)
=2L  (周の長さをLとする)

だな。
ある平面にランダムに点が奇数個ある。それぞれ最小距離の点との距離は全て異なる。
この点が最小距離の他の点にむかってそれぞれ等しい距離で動く。
二つが重なった時、それは一つの点となる。
どれほどの時間がたっても、点の数が変わった瞬間には
すべての点において最小距離の点との距離は等しくないことを証明せよ
>>916
正n角形の頂点を( x(i) , y(i) )と表記する。
仮にx(i)、y(i)ともに全てのiについて有理数であれば、
適当な自然数をかけてやる事で、全て整数に変える事ができ、
また、全ての点を平行移動させる事によって
Σ[i=1,n] x(i) = 0   x(i)は全て整数
Σ[i=1,n] y(i) = 0   y(i)は全て整数
を満たすように正n角形を変形できる。無論、変形後も正n角形の形は崩れない。
よって、あるnに対し、全ての頂点が有理数であるような正n角形が存在したならば、
全ての頂点が整数になり、上の条件を満たすような正n角形が存在する。
以下、全ての頂点が正n角形の頂点( x(i) , y(i) )が上の条件を満たすときのみを考える。

条件を満たす正n角形のうち、一辺の長さが最小の物を考え、その長さをL(n)と置く。 (各頂点が整数なので最小の物が存在。)
正n角形の中心O。隣り合う二点をA,Bとすれば、AB=L(n)であり、OA=OB、∠AOB=2π/n。である。
そのため、OA*sin(π/n)=L(n)/2が成立し n≧7の時、OA>L(n)が成り立つ。
今、( x(i) , y(i) )が上の条件を満たすとき、
( x(i)-x(i-1) , y(i)-y(i-1) )を頂点とする正n角形も上の条件を満たす。 ただし、x(0)=x(n)と循環するように考える。
ここでn≧7の時、OA>L(n)だったのだから、
正n角形( x(i) , y(i) )よりも正n角形( x(i)-x(i-1) , y(i)-y(i-1) )の方が一辺の長さが小さくなる。
ところが、これは上で考えた最小性に矛盾する。 従ってn≧7の時は条件を満たさない。よって、n=3,4,5,6のどれか。
また、OA*sin(π/n)=L(n)/2 、OA、L(n)が整数である事より、sin(π/n)は有理数。
ところが、n=3,4,5,6のうち、これを満たすのはn=4のみ。

よって条件を満たす正n角形は正方形のみ。
そして、正方形の場合、確かに条件を満たしている。
923922:03/11/12 01:11
スマン。大嘘だ。ここまでミスったのはある意味スゲー。

訂正
よって、n=3,4,5,6のどれか。
また、A=(a,b) B=(b,c)として、△OABの面積は|ad-bc|/2なので、a,b,c,dが有理数ならば△ABCの面積も有理数。
ここで、正n角形X(i)=( x(i) , y(i) )は三角形OX(i)X(i+1)をn個あわせた物なので、その面積は有理数となる。
正n角形の一辺の長さをL(n)とすれば、その面積は(L(n)^2)/(2tan(π/n))となる。L(n)^2は有理数なので、
結局tan(π/n)が有理数である必要がある。

よって、n=3,4,5,6のうち、これを満たすのはn=4のみ。。。


なんてボケだ
924132人目の素数さん:03/11/12 01:19
>>848
解答お願い
実数から実数への写像fが次の条件を満たすとき、fを求めよ。
任意の実数x,yについてf(x+y)=f(x)+f(y)
f(1)=1
任意の実数a,bに対し、区間[a,b]において、f(x)は有界である。
>>925
Qを有理数全体
1)f(x)=x (x∈Q) 証明(ry
2)xを無理数とする。任意のε>0に対し次の条件を満たす整数M,Nの組
が無限個ある.|Mx-N|<ε証明(ry
3)ある無理数xがあってf(x)≠xとする。
任意のε>0に対し|Mx-N|<εを満たす整数(M,N)の組が無限個ある。
f(Mx-N)=f(Mx)-N=Mf(x)-N
Mf(x)-N-(Mx-N)=M(f(x)-x)であるがMを十分大に取ると右辺は幾らでも
(絶対値が)大きくなる。Mx-N>-εより結局f(Mx-N)=Mf(x)-Nは幾らでも
絶対値が大きくなりえる。
これは[-ε,ε]でfが非有界であることを意味する。従ってf(x)=xでなけれ
ばならない。
927132人目の素数さん:03/11/12 03:51
nを自然数とする。
x(1),x(2),…,x(n+1),x(n+2)についての連立方程式、
x(n+1)^2 = Σ[k=1,n] ( x(k) )^2
x(n+2)^2 = Π[k=1,n+1] x(k)
が自然数解を持つようなnが存在するのならば、それを一つ求めよ。
また、そのようなnが一つも存在しないのならば、その事を示せ。
一辺の長さaの正方形がx-y座標平面に置かれている。
この正方形の内部、および周上に格子点が一つも存在しないとき、
aの満たす条件を求めよ。
無限数列a(1),a(2),…,a(n),…が全て正の実数であるとき
b(n)=( ( Σ[k=1,n] a(k) )/n ) - ( Π[k=1,n] a(k) )^(1/n)
と定義する。任意の自然数nに対して
b(n)≦b(n+1)
を示し、等号成立条件を求めよ。
nは自然数であるとする
x(k)=cos(kπ/n)
として、f(x)をn-1次以下のxについての一変数多項式とする。
Σ[k=0,n-1] ((-1)^k)*(f(x(k+1))-f(x(k)))
を条件を満たす全てのfに対して求めよ。


さーて、そろそろ解くか。
931132人目の素数さん:03/11/12 15:32
>>922
4行目〜6行目がぁゃιぃんだが。証明しる。
>>931
その通りだ良く気づいた。

正n角形のn個の頂点を( x(i) , y(i) )とする。
X = Σ[i=1,n] x(i)
とすれば、Xは有理数。また、両辺からXを引けば
0 = Σ[i=1,n] ( x(i) - X/n )
が成立して、 x(i) - X/nも有理数となる。ここでx(i) -> x(i) -X/nという変換は平行移動を意味している。
従って、正n角形のn個の頂点を( x(i) -X/n , y(i) )と変形しても、形は崩れない。
同様に、Y = Σ[i=1,n] x(i) とおき、( x(i) -X/n , y(i) - Y/n ) を考えれば、これは正n角形を成す。
ここで、x(i) -X/n , y(i) - Y/n は共に有理数なので、適当な自然数Mをかける事により整数に変形できる。
適当な変数Mをかける事は図形的には拡大を意味するので、これも形を崩さない。・・・以下略。
>>918 の採点希望。
>>918
おお、すばらしい。
>>934
いや、だから。これ工房レベルじゃないと思うのよ。
自分でも間違ってるって書いたじゃん>>918

>>918を書いた本人が間違ってると思ってるのに、すばらしいって言われても・・・
>>918ってまちがってるの?その後訂正は入ってるみたいだけど。
 
>いや、だから。これ工房レベルじゃないと思うのよ。
 
それはそうだろうね。だんだん問題だす側のタガが外れてきた気はする・・・
>>918の補足以下を見れば分かると思う。
あれはあくまで、凸多角形に対してのみ証明を行った物。
問題は凸図形に対して聞いているので、凸多角形に対してのみの解答だと不十分だと思う。
それと、
http://coolee.at.infoseek.co.jp/mondai3.html#15
に凸多角形から凸図形に変形する際、f(θ)の一様収束性を使った証明が載ってたけど、
やっぱり、工房レベルじゃないよねぇ。
つまり凸図形についてはただしいんでしょ?だったらもう解決でいいんじゃないの?
そもそも工房レベルの問題じゃないんだし数学科の標準的な実力があれば
たとえば境界が区分的にC1の場合とかに拡張するのはすぐできるだろうし。
(というか一般的に「周長をもつ図形」といって説明もなんもない場合には
境界は区分的にC1は仮定していいだろうし。)
>>938
そだね。解決という事にしとこう。

頑張ったぞ俺@化学専攻/数学離れ歴10年
乙、そしてgood job!
941132人目の素数さん:03/11/13 02:32
A君とB君とC君が相撲をする。
始めにA君とB君が勝負し、勝った方がC君と戦う。
以下、だれかが2連勝するまで、「前の試合の勝者が、前の試合で戦わなかった者と勝負する。」
を繰り返す。

A君がB君に勝つ確率が 1/2
C君がA君に勝つ確率が p
C君がB君に勝つ確率が p

であるとき、優勝確率は3人とも等しいという。
このときpを求めよ。
>>926
1),2)を信じるとしても、3)で帰謬法を使っているから大減点。
東大入試では帰謬法・背理法は減点対象.0点も有り得る,
特に背理法の場合、そればかりでなく他の問題があってても減点になるぞ。
3)の正しい解答は
|f(x)|<L |x|<ε<Lの時
M|f(x)-x|=M|f(x)-N/M+N/M-x|<=|Mf(x)-N|+|Mx-N|=|f(Mx-N)|+|Mx-N|<2L
|f(x)-x|<2L/Mで、xが無理数でもMは幾らでも大きくなり得るから
|f(x)-x|=0∴f(x)=x
これで、1),2)を出題者が信じればf(x)=xであると出題者は納得する。

帰謬法は問題文で明示されている前提を組み合わせると結果が矛盾するこ
とを指摘する方法。まぁ、出題者の主張の矛盾を指摘する方法だから嫌わ
れて当然だな。
背理法は問題文で明示されている前提を組み合わせて、問題文に明示され
ていないが一般的により強く信じられている原理への背理を指摘する方法
で、これは出題者が異端者であることを暗に指摘し、改宗を促す洗脳行為
と見なされるから当然嫌われる。(俺が採点者だったら0点付ける)
ちなみに京大ではむしろ背理法や帰謬法を使った解答のほうが評価は高い
というウワサ。
1),2)を省略せずに書けばの話だが。1)2)3)と分けずに一つの文章で纏め上げて
出題者の提示した前提の矛盾を導くか出題者の背教行為を指摘すればよい。
(背理法や帰謬法・帰納法を用いずに)
√2が無理数であることを示せ
946132人目の素数さん:03/11/13 21:25
粗悪燃料(北朝鮮産)投下あげ
947132人目の素数さん:03/11/13 22:04
縦読み?
>>943
お前が採点者になることはありえないから安心だ。
>>848
やっと解けた。 高校レベルだったよ。良問かも。
∠CABを単純にAと略す。同様に、∠ABC=B、∠BCA=Cと記す。
条件より、PF⊥AB、PE⊥CAが成立するため、四角形AFPEは円に内接し
その直径の長さはAPである。また、四角形AFPEの外接円は三角形AFEの外接円でもあるので、
その直径は正弦定理を用いて、EF/sin(A)となる。よって、PA=EF/sin(A)、EF=PA*sin(A)が成立する。
ここで、点Eから直線BCに下ろした垂線の足をE'とし、同様に点FからBCに下ろした垂線の足をF'と定義する。
明らかにEF≧E'F'が成立する。よって、PA*sin(A)≧E'F'。  [eq.1]

また、DF'=PF*sin(B) DE'=PE*sin(C) E'F'=PF*sin(B)+PE*sin(C)が成立する。 [eq.2]
[eq.1]と[eq.2]をあわせれば、
PA*sin(A)≧PE*sin(C) + PF/sin(B)
よって、
PA + PB + PC
≧( (sin(C))/(sin(B)) + (sin(B))/(sin(C)) )PD
+ ( (sin(A))/(sin(C)) + (sin(C))/(sin(A)) )PE
+ ( (sin(B))/(sin(A)) + (sin(A))/(sin(B)) )PF
≧2(PD+PE+PF)
951132人目の素数さん:03/11/14 14:11
>>929
う〜ん、n*b(n)≦(n+1)*b(n+1) なら示せそうなんだが…。むずい…。
952132人目の素数さん:03/11/15 01:05
>>951
ごめん、出題者の俺が間違っただけ。
元ネタは
http://www.pp.iij4u.or.jp/~yonemura/pdf/78s.pdf
アナタの方法で正解です。

ちなみに、>>927も間違っててn≧2の条件が抜けてた。これがないとメチャ簡単。
あとは>>929の元ネタも
http://www.pp.iij4u.or.jp/~yonemura/pdf/71s.pdf
>>930は数学セミナーからチェビシフの多項式でございます。
953132人目の素数さん:03/11/15 04:11
座標平面上に円C:x^2+y^2=1と,2点A(2,2),B(a,0) (aは定数)があり,
円C上を動く点をPとする。ただし,点PはA,B,Pが△ABPをなすように動く。
(1) a=0のとき,△ABPの面積の最大値を求めよ。
(2) △ABPの面積の最大値をaを用いて表せ。
(3) △ABPの面積の最小値が存在しないためのaに関する必要十分条件を求めよ。
954132人目の素数さん:03/11/15 04:29
四面体をある平面によって0より大きい体積を持つ二つの領域に分割する。
このときの断面積をSとすれば、Sは最大値を持たない事を示せ。
955132人目の素数さん:03/11/15 22:02
正六面体を2つの平面によって0より大きい同体積を持つ3つの領域に分割する。
このときの断面積の合計をSとすれば、Sは六面体の表面積よりも小なる事を示せ。
956132人目の素数さん:03/11/15 22:13
てめぇが示せ バカ!
957132人目の素数さん:03/11/15 22:19
本当に難易度が高くなった。せめて、元ネタだけでも希望する
元ネタっていっても少なくとも>>953以降はオリジナル問題に見えるけど。
>>953以降ぐらいの問題だったらホントに受験問題でもでても不思議じゃないぐらい
のレベルにみえるけど。
959132人目の素数さん:03/11/15 22:49
>>958
解いてくれ・・・頼む。
>>959
どれ?
961132人目の素数さん:03/11/15 23:02
>>960・・・スマン、全部解けた。
954はつまり体積が0になれないから最大値が定義されないってこと
955は1つの断面の面積の最大値を求めれば、糸冬

ここに出される問題って大抵元ネタありなのか?(自作ばかりだと思っていたから、流石だなと思っていたが)
a_n+1=4(a_n)^3-3(a_n)となる数列{a_n}が7個の数値しか持たない時の、一般項a_nを求めよ。
964132人目の素数さん:03/11/16 20:44
1〜2004の自然数のうち、倍数にぞろ目を持たない数の個数を求めよ。
965132人目の素数さん:03/11/16 21:08
愚問。
966132人目の素数さん:03/11/16 21:43
>>964 1804個
>>966
おれの計算だと、1175個だったよ。

10と互いに素な自然数ってぞろ目が倍数にならない?
968964:03/11/16 22:50
>>966 >>967
どちらも間違いです。
一応入試ならば、求めるべき条件の明示と証明も必要になるから、問題としては形になっているかなと思うのだが
969967:03/11/16 23:18
計算しなおした。340個?
>>964
ぞろ目とは?
971964:03/11/16 23:52
>>969
正解。一応理由と軽い証明法もあったらもっと良かったけど。

ぞろ目・・・すべての桁の数が同じ数字
972132人目の素数さん:03/11/17 00:00
全ての桁の数が同じか・・・なぁんだ、ワンペアがあればいいのかと思った。

2〜4008で
2、22、222、2222
4、44、444
 ……
8、88、888

あれ???あわねー!!!
倍数にゾロ目を〜
ってあるけど、倍数って何よ?何の倍数よ?
>>964に対する俺的な疑問。

1,2,0,4の数字が出ているが、当然5進数で考えるんだよね?
倍数って言う言葉があるけど、何の倍数?
ぞろ目を「持たない」っていう言葉があるけど、途中にぞろ目があればいいの?
  例えば、1223とか・・・ それとも、1323みたいなのも含む? それとも111みたいなのだけ?
>>969>>971
なんで?>>967ってまちがってるの?10と互いに素な香具師だけで
もっと多いとおもうんだけど。
976132人目の素数さん:03/11/17 00:37
自然対数eが円周率πよりも小さいことを証明せよ。
>>976
愚問じゃないか?そもそもeもπもろくに定義もできない状態で大小比較なんて。
んで、次スレは?
979964:03/11/17 08:09
>>973 >>974
問題を解釈すると・・・
1〜2004の中の自然数の一つをkとする。
kを整数倍した時に、ぞろ目になるモノが1つでも含まれていれば集合Aに、そうでないモノをBとする。
では、Bの要素はいくつになるか?
ということです。

またぞろ目というのは、「各位の数がすべて一致」することを意味している(つもりです)

>>975
結果から言うと、10の倍数,16の倍数,25の倍数が求めるべき条件です。
余事象は10と互いに素からもっと絞り込めます。
>>978
たてられませんでした。>>981頼む。
できればテンプレ文の最後に「それ以上の難題はスレ違いです。」
くらいを付け加えてほしい。
>結果から言うと、10の倍数,16の倍数,25の倍数が求めるべき条件です。
>余事象は10と互いに素からもっと絞り込めます。
 
わかった。Aの方をもとめてた。Bの数か。
982pig:03/11/18 16:49
体積がVの粘土の塊がある。これで直円錐形のやじりを作るときやじりの表面積の最小値を求めよ。
983132人目の素数さん:03/11/18 22:01
               /  /     /   |      \ ヽ
               / /  /   / /    ||  |  i  ヽ i
              i /  / /  / / /    ||  ||  |│ |ノス
               |//  / /___, -一ァ|  /! |ト、|│ | | く」
                |,-‐¬  ̄---┘'7 |!  ハ! |,、-┼十|! | | |
          , -‐ ''"  し' '´_ /,ィ二l |ト、/!ヽト、\_ヽ!|!l | ハ |
       ,r/      __   ,イ|リ ヾハ! ヽ!  ,ィ⌒ヾミリノ!/リ  |
      / ||ヽ  -'     / ̄ )` __      |ヒノ:} '` ,イ/ |  | 
    ,r '   ヾ、  ,-、____ , イ ̄,r==-      ==-'  レ' /|  |   
  / ヽ    `ーソ  ' | |ト、,ヘ ′""          "" / / || |   
. /    \_  /  | ハ ヽ`゙'ヘ       ' '__. ィ  / / | |  |  
           /   / / |  ヽ 川\    ヾ三ニ‐'′//! |  | |  |    次スレをよろしくであります
        /    / / 八  \川| |`ト- ..  __ , イ‐ァヘ |  | ||  |!
      /    / / /  \  \ 「`ー- 、    /  .〉  ト、|  ヽ、
     ,イ    /-─=¬ニヘ、_  \   厂\ 厂ヽ /!|   | `ー=ヘ
 -‐  ̄ /─ '  ̄     ├- ヽ\  \ノ\ \ 人 ハ!ヽ ||  |-┤ ヽ
      /          /!‐-- | |\   ト、_`ヽ oヽ  ト、!  ||  |‐┤- ヽ
  // 〉      __ /  ├‐-  ||  | 川-‐  | |  厂7! ハ!  ├:┤  ̄ヽ
  / / ー ─    ̄       ├‐- リ  || ハ!ヘ   | |  ト┤|/′ ヾ,┤   ゙i_
  ‐ '              〉‐-    | / /\ .|o | /ヽ/(′    ∨     \
‐--─ ──-r、___-、    /ー_     {(   '´>、! /ヽ/       |\       \
★東大入試作問者になったつもりのスレ★ (第一問)
http://science.2ch.net/test/read.cgi/math/1000592003/l50
★東大入試作問者になったつもりのスレ★ (第二問)
http://science.2ch.net/test/read.cgi/math/1046165076/l50
984132人目の素数さん:03/11/18 22:04
タイトルは 「★東大入試作問者になったつもりのスレ★ (第三問)」 でいい?
985132人目の素数さん:03/11/18 22:07
立てられませんでした…
986132人目の素数さん:03/11/19 00:53
              ☆ チン

        ☆ チン  〃 ∧_∧   / ̄ ̄ ̄ ̄ ̄ ̄ ̄ ̄ ̄ ̄ ̄ ̄
          ヽ ___\(\・∀・)<ねぇ、新スレまだぁ〜?
             \_/⊂ ⊂_)_ \____________
           / ̄ ̄ ̄ ̄ ̄ ̄ ̄/|
        |  ̄  ̄ ̄ ̄ ̄ ̄ ̄:| :|
        |            |/
         ̄ ̄ ̄ ̄ ̄ ̄ ̄ ̄
987132人目の素数さん:03/11/19 00:54
                Λ_Λ ぁぁ-
     ∧_∧    (・∀・;)
    ( ・∀・) ⊂ ⊂_ )    ______
   ⊂   /| ≡≡≡≡≡≡/       /|
     |  _/ ≡≡≡≡≡≡| ̄ ̄ ̄ ̄ ̄|  |
    (__)彡   ≡≡≡≡≡≡|.愛媛みかん|/

           ボコッ
                    シーン
         =≡= ∧_∧
          /   (・∀・ ) <静かにしてます
        〆   ┌  |    | .∈≡∋
         ||  γ ⌒ヽヽコノ   ||
         || .|   |:::|∪〓  ||
        ./|\人 _.ノノ _||_. /|\
             ウズウズ…

         =≡=  ∧_∧   I'm ready.......
          / \ (・∀・ )/
        〆     ⊂    つ∈≡∋
         ||  γ ⌒ヽヽコノ   ||
         || .|   |:::|∪〓  ||
        ./|\人 _.ノノ _||_. /|\
                _∧_∧_∧_∧_∧_∧_∧_∧_
     デケデケ      |                         |
        ドコドコ   < ○○○まだーーーーーーーー!!? >
   ☆      ドムドム |_ _  _ _ _ _ _ _ _ _|
        ☆   ダダダダ! ∨  ∨ ∨ ∨ ∨ ∨ ∨ ∨ ∨
  ドシャーン!  ヽ         オラオラッ!!    ♪
         =≡= ∧_∧     ☆
      ♪   / 〃(・∀・ #)    / シャンシャン
    ♪   〆  ┌\と\と.ヾ∈≡∋ゞ
         ||  γ ⌒ヽヽコ ノ  ||
         || ΣΣ  .|:::|∪〓  ||   ♪
        ./|\人 _.ノノ _||_. /|\
         ドチドチ!
        スコココバシッスコバドドドンスコバンスコ  _∧_∧_∧_∧_∧_∧_
            从 `ヾ/゛/'  "\' /".    |                     |
        ≡≪≡ゞシ彡 ∧_∧ 〃ミ≡从≡=< 新スレまだぁー?       |
.          '=巛≡从ミ.(・∀・# )彡/ノ≡》〉≡ |_  _  _ _ _ _ __ _|
...        《゛=!|l|》リl⌒! I⌒I I⌒I I⌒I从=≡|l≫,
          《 l|!|!l!((つT(つ) ((つT(つ)) !|l!|l;》;
       《 l|!| ̄| ̄γ ⌒ ヽ γ ⌒ ヽ三ll≡|l》;
..        《l|!|  | ((TAMA))((TAMA))||l|||l 》;
      ≡丿-へ/人 _ 人 人 _ 人//へヾ
          ドドドドドドドドドドドドドドドドドドド
9931:03/11/19 08:54
なんで京大の文字を消したんじゃーーーマジで許せねーーーーーー
東大の入試は解いたことないから分からんけど、
聞いたところによると、入試問題は京大のほうが難しいらしい。
兄弟の問題の方がしつこいというか、ねちっこい印象がある。
9961:03/11/19 12:12
京大と東大の比較とかが問題なのではなく
スレタイに京大が入ってない事の方が問題なのだ。
9971:03/11/19 12:13
何故抜かしたんじゃーーー
9981:03/11/19 12:14
もうね。許せませんよこれは。呪いますね
9991:03/11/19 12:14
お前を一生呪ってやるーーー
10001:03/11/19 12:15
さらば愛しき東大京大スレ
10011001
このスレッドは1000を超えました。
もう書けないので、新しいスレッドを立ててくださいです。。。